Download as pdf or txt
Download as pdf or txt
You are on page 1of 112

Electric Field and Charge

Important Formulae and Notes

▶ Charge quantisation, q = ne E = intensity of electric field at a given point


where e = charge on one electron θ = angle made by component of electric field intensity
n = 1, 2, 3, ... (any natural number) with normal to the given surface.
▶ Electrostatic Force of attraction or repulsion between ▶ Application of Gauss’s theorem
two charges q1 , q2 , separated by distance r
(a) Electric intensity at a point outside a charged
1 q1 q2 sphere
F = q q
4πϵm r2 E= or E =
4πϵr 2 4πϵ0 Kr2
Where, ϵm = absolute permitivity of the medium where K = dielectric constant
ϵm ϵ0 = permittivity of free space
▶ Relative Permitivity ϵr or dielectric constant K =
ϵo (b) If point is on the surface of charged sphere, i.e. r
where, ϵ0 = permitivity of the free space
1 q
1 = R, then E =
▶ = 9 × 109 Nm2 C−2 4πϵ0 K R2
4πϵ0
(c) If σ is the surface charge density of the charged
ϵ0 = 8.854187817 × 10−12 C2m2/N
sphere A, then
▶ When two identical conductors having charges q1 and charge q
σ= = −

m
q2 are put to contact and then separated, then each surface area 4πR2
q1 + q2 σR2
have a charge equal to . then, E=
2 ϵr2
▶ If three identical charges (q) are kept at three corners (d) For the point near surface of sphere, r = R then,
of an equilateral triangle of side d then the resultant
force on any one of the charge F = 9 × 109 ×

a square of side d then the resultant force on


√ 2
3q
d2
▶ If four identical charges (q) are kept at four corners of
ra E=
σR2
ϵR

cylinder
2
=
σ
ϵ
(e) Electric intensity at a point outside a charged

1
 any one
q
G
E= or E =
2 √
1 2πϵr 2πϵ0 Kr

q
of the charge is given by F = 9 × 10 × 3 9
2+
d 2 (f) Electric intensity at a point near a charged con-
σ σ
▶ Electric Field intensity (E) at a distance r from charge ductor E = =
1 ϵ ϵ0 K
u
Q
Q, E = . (g) Energy per unit volume of the medium sur-
4πKϵ0 r2
rounded by charged conductor i.e. energy den-
▶ If a charge q is placed in an electric field, it experiences
σ2
Ed

dU
a force, F = qE sity, =
dV 2ϵ0 K
▶ Electric dipole moment (p) = q × 2l (h) Energy density in terms of electric intensity is
▶ Electric flux ϕ = E. ⃗ = E ds cos θ
⃗ ds dU 1
= ϵ0 KE 2
where ds = small surface area dV 2

Multiple Choice Questions

1. Two small identical spheres having charges +10 µC d) execute oscillatory but not SHM
and -90 µC attract each other with a force of F new-
ton. If they are kept in contact and then separated by 3. The electric charge in accelerated motion produces
the same distance, the new force between them is
a) an electric field only
a) F/6 b) 16F c) 16F/9 d) 9F b) a magnetic field only
c) EM radiation only
2. Two equal negative charges q are fixed at the point d) all of the above
(0, a) and (0, −a) on the y-axis. A positive charge Q
is released from rest at the point (2a, 0) on the x-axis. 4. Two equal and opposite charges are placed at a certain
The charge distance. The force between them is F . If 25% of one
a) execute SHM about the origin charge is transferred to other, then the force between
b) move to the origin and remain at rest them is
c) move to infinity a) F b) 9F/16 c) 15F/16 d) 4F/5
Electric Field and Charge

5. If a proton, a deutron and an α particle are kept in


the same electric field
a) proton and deutron will have the same
acceleration a) b)
b) deutron and α particle will have the same
acceleration
c) α particle will have the minimum acceleration
d) proton will have the minimum acceleration c) d)

6. A charged particle of mass m and charge q is released 12. The electric field due to a uniformly non-conducting
from rest in an electric field of constant magnitude E. solid sphere of radius R is a function of the distance
The kinetic energy of the particle after time t is from its centre is represented graphically by
2E 2 t2 E 2 q 2 t2 Eq 2 m Eqm
a) b) c) d)
mq 2m 2t2 2t

7. Identify the statement in the following.


Coulomb’s law correctly describes the electric force
that
a) binds the electron of an atom to its nucleus
b) binds the proton and neutrons in the nucleus of

m
an atom
c) binds atoms together to form molecules
d) binds atoms and molecules to form solids

8. An electron having charge e and mass m is moving in


a uniform electric field E. Its acceleration will be

a)
e2
m
b)
eE
m
c)
eE 2
m
d)
mE
e
ra a) a

a)
b) b

Transfer of electrons
c) c d) d

13. The cause of quantization of electric charge is


G
b) Transfer of protons
9. The electric flux for Gaussian surface A that encloses c) Transfer of integral number of electrons
the charge particles in free space is d) none of the above
given q1 = 14 nC, q2 = 78.85 nC, q3 = 56 nC)
14. What is the flux through a cube of side a if a point
u

charge of q is at one of its corner?


2q q q 2q 2
a) b) c) d) 6a
Ed

ϵ0 8ϵ0 ϵ0 ϵ0
15. A charge q is placed at the centre of the line joining
two exactly equal positive charges Q. The system of
three charges will be in equilibrium, if q is equal to
Q Q Q
a) 103 N m2 C −1 b) 103 CN −1 m−2 a) b) − c) − d) −Q
4 4 2
c) 6.32 × 103 N m−2 C −1 d) 6.32 × 103 CN −1 m−2
16. A charge Q is placed at the centre of cube of side a
10. Which of the following statements is false for a perfect then flux linked with one of its given faces will be
conductor? Q Q Q Q
a) b) c) d)
a) The surface of the conductor is an equipotential ϵ0 6ϵ0 ϵ0 a2 4πϵ0 a2
surface. 17. Dielectric constant for a metal is
b) The electric field just outside the surface of a a) zero b) infinite c) 1 d) 10
conductor is perpendicular to the surface.
c) The charge carried by a conductor is always 18. When air is replaced by a dielectric medium of con-
uniformly distributed over the surface of the stant K, the maximum force of attraction between two
conductor. charges separated by a distance
d) None of these. a) increases K times b) remain unchange
c) decreases K times d) increases K−1 times
11. The temperature T dependence of resistivity ρ of a
semiconductor is represented by 19. A body has negative charge of 1.6 × 10−17 C. It has

2 - Nikhil Kamboj — 8193989452 #edugramddun


Electric Field and Charge

a) an excess of 100 electrons a) Decreases


b) a deficiency of 100 electrons b) Increases
c) an excess of 10 electrons c) Remains unchanged
d) a deficiency of 10 electrons d) Nothing can be predicted as information is
insufficient
20. A metallic solid sphere is placed in uniform electric
27. An electric dipole consists of two opposite charges of
field as shown in figure. Which path is followed by
magnitude 1µC separated by a distance of 2cm. The
electric field lines?
dipole is placed in an electric filed 10−5 V/m. The
maximum torque experienced by the dipole is
a) 10−3 Nm b) 2 × 10−13 Nm
c) 3 × 10−3 Nm d) 4 × 10−3 Nm

28. A charge Q is uniformly distributed over a large square


plate of copper. The electric field at a point very close
a) a b) b c) c d) d to the centre of the plate is 10 V/m. If the copper plate
is replaced by a plastic plate of the same geometrical
21. Two equally charged spheres attract each other. Now dimensions and carrying the same charge Q uniformly
if they touch each other then they will distributed, then the electric field at the point P will
be
a) Repel each other
b) Attract each other with the same amount of force a) 5 V/m b) zero c) 10 V/m d) 20 V/m

m
c) Attract each other with less amount of force
d) Don’t attract or repel 29. The inward and outward electric flux for a closed sur-
face in units of N-m2 /coulomb are respectively 8 × 103
and 4 × 103 . Then, the total charge inside the surface
22. Pick the true statements about electric field lines.
a) It provides information about the direction of
the electric field
b) Electric field lines provide information about the
ra is
a) 4 × 103 C
c) −(4 × 103 )/ϵ0 C
b) −4 × 103 C
d) −(4 × 103 )ϵ0 C
G
type of charge 30. What is the electric flux linked with closed surface?
c) Electric field lines provide information about the
field strength
d) All of the above
u

23. An electric dipole formed by charge +6 µC and -6 µC,


separated by a distance of 2 mm.
Ed

The dipole moment is .... C m, directed from ....... to a) 1011 N m2 /C b) 1012 N m2 /C


....... c) 1010 N m2 /C d) 8.86 × 1013 N m2 /C
a) 12 × 10−9 , negative, positive
31. A hollow cylinder has a charge q coulomb within it. If
b) 24 × 10−9 , negative, positive
ϕ is the electric flux in units of voltmeter associated
c) 24 × 10−9 , positive, negative with the curved surface B, the flux linked with the
d) 12 × 10−9 , positive, negative plane surface A in unit of V-m will be

24. A force of 4N is acting between two charges in air. If


the space between them is completely filled with glass
(relative permittivity = 8), then the new force will be
a) 2N b) 5N c) 0.2N d) 0.5N
1
 
q ϕ q q
a) b) c) −ϕ d) −ϕ
2ϵ0 3 ϵ0 2 ϵ0
25. Two point charges Q and −3Q are placed some dis-
tance apart. If the electric field at the location of Q is 32. A proton has a mass of 1.67 × 10−27 kg and charge
E, the field at the location of −3Q is 1.6 × 10−19 coulomb. If the proton is to be accelerated
a) E b) -E c) E/3 d) -E/3 through a potential difference of one million volt, then
the KE is
26. A soap bubble is given a negative charge, then its ra- a) 1.6 × 10−15 J b) 1.6 × 10−13 J
dius c) 1.6 × 10−25 J d) 3.2 × 10−13 J

3 - Nikhil Kamboj — 8193989452 #edugramddun


Electric Field and Charge

Directions: a) (ϕ1 + ϕ2 )ϵ0 b) (ϕ1 − ϕ2 )ϵ0


(a) Both Assertion and Reason are correct and the c) (ϕ1 + ϕ2 )/ϵ0 d) (ϕ1 − ϕ2 )/ϵ0
Reason is a correct explanation of the Assertion.
(b) Both Assertion and Reason are correct but Reason 40. Charge is distributed uniformly on the surface of a
is not a correct explanation of the Assertion. spherical balloon (an insulator) with a point charge q
(c) Assertion is correct, Reason is incorrect inside. The electric force on q is greatest when:
(d) Both Assertion and Reason are false . a) It is near the inside surface of the balloon
33. Assertion : A metallic shield in form of a hollow shell b) It is at the centre of balloon
may be built to block an electric field. c) It is anywhere inside (the force is same every-
Reason : In a hollow spherical shield, the electric field where and is non-zero)
inside it is zero at every point. d) It is anywhere inside (the force is zero
everywhere)
a) a b) b c) c d) d
41. The total electric flux leaving spherical surface of ra-
34. Assertion : The tyres of aircraft are slightly conduct- dius 1 cm and surroundings an electric dipole is:
ing.
Reason : If a conductor is connected to ground, the a) q/ϵ0 b) zero c) 2q/ϵ0 d) 8π 2 q/ϵ0
extra charge induced on conductor will flow to ground.
42. Two charges Q1 and −Q2 are separated by a distance
a) a b) b c) c d) d r. The charges attract each other with a force F .
What is the new force between the charges if the dis-
35. Assertion: Electron move away from a region of lower tance is tripled?

m
potential to a region of higher potential. a) 16 F b) 9 F c) F/9 d) F/3
Reason: An electron has a negative charge.
a) a b) b c) c d) d 43. If two like charges of magnitude 1×10−9 C and 9×10−9
C are separated by a distance of 1 metre, then the
36. A non-conducting solid sphere of radius R is uniformly
charged. The magnitude of the electric field due to the
sphere at a distance r from its centre
a) Increases as r increases for r < R
ra point on the line joining the charges, where the force
experienced by a charge placed at that point is zero,
is
a) 0.25 m from the charge 1 × 10−9 coulomb
G
b) Decreases as r increases for 0 < r < infinite b) 0.75 m from the charge 9 × 10−9 coulomb
c) Decreases as r increases for R < r < infinite c) both 1 and 2
d) Both a and c d) at all points on the line joining the charges
u

37. If the electric field is given by 5î + 4ĵ + 9k̂, the electric 44. An electron moves through a small distance in a uni-
flux through a surface of area 20 unit lying in the Y-Z form electric field. The magnitude of electric field
plane will be: is 2 × 104 NC−1 . Now, if the direction of field is
Ed

reversed keeping the magnitude same and a proton


a) 100 unit b) 80 unit moves through the same distance, then which of the
c) 180 unit d) 20 unit following options is correct?

38. A square surface of side L meter in the plane of the a) The time of fall will be more in case of electron
paper is placed in a uniform electric field E (volt/m) b) The time of fall will be more in case of proton
acting along the same plane at an angle θ with horizon- c) The time of fall will be same in both cases
tal side of the square as shown in figure. The electric d) The time of fall will be independent of charge
flux linked to the surface, in units of volt-m is:
45. Point charges +4q, –q and +4q are kept on the x-axis
at points x = 0, x = a and x = 2a respectively
a) only –q is in stable equilibrium
b) none of the charges are in equilibrium
c) all the charges are in unstable equilibrium
d) all the charges are in stable equilibrium

46. The region surrounding an oscillating electric dipole


a) EL 2
b) EL cosθ c) EL sinθ d) zero
2 2 has
a) electric field only
39. The electric flux entering and leaving an enclosed sur- b) magnetic field only
face is ϕ1 and ϕ2 respectively, the electric charge inside c) electric as well as magnetic field
the enclosed surface will be: d) nothing can be predicted

4 - Nikhil Kamboj — 8193989452 #edugramddun


Electric Field and Charge

47. An electric line of force in x-y plane is given by a) 5 cm b) 10 cm c) 2.5 cm d) 20 cm


x2 + y 2 = 1. A particle with unit positive charge,
initially at rest at the point x = 1, y = 0 in the x-y 54. All free charges are integral multiple of a basic unit
plane charge e. Then quantization rule of electric charge
implies
a) will not move at all
b) will move along the straight line a) Q = e b) Q = ne c) Q = d) Q = e2
c) will move along the circular line of force n/e
d) information is insufficient to draw any conclusion
55. Two point mass each having mass m are placed in air
48. Gauss’ law in electrostatics is true, when the charges at distance r attract by applying force F . If medium
enclosed in the Gaussian surface are between these two particles is filled by material having
specific gravity 3, then what will be the new value of
a) stationary only b) moving only force?
c) moving or stationary d) none of these
a) F/3 b) 3F c) 9F d) F
49. It is not convenient to use a spherical Gaussian surface
56. Which of the following figures represent the electric
to find the electric field due to an electric dipole using
field lines due to a single negative charge?
Gauss’ theorem because
a) Gauss’ law fails in this case
b) this problem does not have spherical symmetry
c) Coulomb’s law is more fundamental than Gauss’

m
law
d) spherical Gaussian surface will alter the dipole
moment

50. An electric charge q exerts a force F on a similar elec-ra


tric charge q separated by a distance r. A third charge
57. In case of infinite long wire electric field is proportional
q/4 is placed mid-way between the two charges now,
to
the force F will.... 1 1 1
a) become F/4 b) Remain F a) b) 2 c) 3 d) r0
G
r r r
c) Become F/16 d) Become F/2
58. Electric field outside a long wire carrying charge q is
51. Two charges q and −3q are placed fixed on x axis sepa- proportional to
rated by distance d. Where should a third charge +2q 1 1 1 1
u
a) b) 2 c) 3/5 d) 3/2
be placed such that it will not experience any force? r r r r
d √ Directions: These questions consist of two
a) (1 + 3) to the left of q
Ed

2 statements, each printed as Assertion and Rea-


3d
b) to the left of q son. (a) Both Assertion and Reason are correct
2 and the Reason is a correct explanation of the
d √
c) (1 + 3) to the right of −3q Assertion. (b) Both Assertion and Reason are
2
d √ correct but Reason is not a correct explanation
d) (1 − 3) to the left of q of the Assertion. (c) Assertion is correct, Rea-
2
son is incorrect (d) Both Assertion and Reason
52. Two positive ions, each carrying a charge q, are sep- are False.
arated by a distance d. If F is the force of repulsion 59. Assertion : On bringing a positively charged rod near
between the ions, then the number of electrons miss- the uncharged conductor, the conductor gets attracted
ing from each ion will be (e being the charge on an towards the rod.
electron) Reason : The electric field lines of the charged rod are
perpendicular to the surface of conductor.
s r
4πϵ0 F d2 4πϵ0 F d2
a) b)
q2 e2 a) a b) b c) c d) d
4πϵ0 F d 2
4πϵ0 F d2
c) 2
d) 60. Assertion : On going away from a point charge or
q e2
a small electric dipole, electric field decreases at the
53. Two point charges exert force of F Newton when sep- same rate in both the cases.
arated by a distance of 10 cm in air. The distance Reason : Electric field is inversely proportional to
for which the force between them will be same if the square of distance from the charge or an electric dipole.
medium them has ϵr = 4, is : a) a b) b c) c d) d

5 - Nikhil Kamboj — 8193989452 #edugramddun


Electric Field and Charge

61. Assertion : No two electric lines of force can intersect Reason : Electron is lighter than proton.
each other.
a) a b) b c) c d) d
Reason : Tangent at any point of electric line of force
gives the direction of electric field.
63. Assertion : The surface densities of two spherical con-
a) a b) b c) c d) d ductors of different radii are equal. Then the electric
field intensities near their surface are also equal.
62. Assertion : Electric force acting on a proton and an
Reason : Surface density is equal to charge per unit
electron, moving in a uniform electric field is same,
area.
where as acceleration of electron is 1836 times that of
a proton. a) a b) b c) c d) d

Practice Questions and Numerical

QUANTISATION OF CHARGE 14. Calculate the Coulomb force between two alpha par-
ticles separated by a distance of 3.2 × 10−15 m
1. Differentiate Mass and Charge. [Ans. 90 N, repulsive]
2. What do you understand by quantisation of charge? 15. What equal charges would have to be placed on earth
Explain. [Ans. q = n e] and moon to neutralize their gravitational attraction?
Given, mass of earth = 1025 kg, mass of moon = 1023
3. Can 8.6 × 10−19 of charge be given to a conductor?
kg. [Ans. 8.6 x 1013 C]
Explain. [Ans. No]

m
4. Define one coulomb. Find the number of electrons con- 16. The electrostatic force of repulsion between two posi-
tained in 1 Coulomb of charge. tively charged ions carrying equal charges is3.7 x 10−9
[Ans. 1 C = 6.25 × 1018 e− ] N, when they are separated by a distance of 5 Å. How
many electrons are missing from each ion? [Ans. 2]
5. A body has negative charge of 1.6 × 10−17 C. What
does this mean? [Ans. excess of 100 e− ]
6. A person combs his hair on a dry day. The comb causes
ra
1022 electrons to leave the person’s hair don and stick
to the comb. Calculate the charge the comb carries.
17. Two small spheres each of mass ′ m′ kg and charge
9 coulomb are suspended from a point by insulating
threads each of l metre length, but of be negligible
mass. If is the angle which each string makes with the
G
vertical when equilibrium at has been reached, show
[Ans. -1.6 x 103 C]
that q 2 = (4mgl2 sin2 θ tan θ)4πϵ0 .
7. Estimate the number of free electrons in 36
18. Two particles, each having a mass of 5 g and charge
g of water and the negative charge possessed
107 C stay in limiting equilibrium on a horizontal table
by them. Given Avogadro’s number = 6.023
u
with a separation of 10 cm Gey between them. The
x 1023 and molecular weight of water = 18.
coefficient of friction between each particle and the
[Ans. 12.046 × 1024 , 1.93 x 106 C]
table is the same. Find the value of this coefficient.
Ed

8. What is the total charge on 75.0 kg of electrons ? [Ans. 0.18]


[Ans. 1.33 x 1013 C]
19. Two small charged spheres contain charges + q1 and
9. How many mega coulombs of positive (or negative) + q2 respectively. A charge dq is removed from sphere
charge are in 1.00 mole of neutral molecular hydrogen carrying charge q1 and is transferred to the other. Find
gas (H2)? [Ans. 0.1927 MC] charge on each sphere for maximum electric force be-
10. Calculate the total positive or negative charge on a 1
tween them. [Ans. (q1 + q2 )]
3.11 g copper penny. Given, Avogadro’s number = 2
6.023 x 1023 per gram mole; for copper, atomic charge 20. Two charges Q1 and −Q2 are separated by a distance
29 and atomic mass = 63.5. [Ans. ± 1.37 x 105 C] r attract each other with a force F. What is the new
COULOMB’S LAW force between the charges if the distance is tripled?
[Ans. F/9]
11. Write Coulomb’s law in vector form. 21. When air is replaced by a dielectric medium of con-
stant K, What will happens to the force of attraction
12. A charge of magnitude Q is divided into two parts q
between two charges. [Ans. decreases K times]
and (Q − q) such that the two parts exert maximum
force on each other. Calculate the ratio Q/q. [Ans. 2] 22. A force of 4N is acting between two charges in air.
13. Two identical metal spheres A and B have equal and If the space between them is completely filled with
similar charges. They repel each other with a force glass (relative permittivity = 8), then what will be
103 N, when they are placed 10 cm apart in a medium new force. [Ans. 0.5N]
of dielectric constant 7. Determine the charge on each 23. Two charges q and −3q are placed fixed on x axis sepa-
sphere. [Ans. 28.3 x 10−6 C] rated by distance d. Where should a third charge +2q

6 - Nikhil Kamboj — 8193989452 #edugramddun


Electric Field and Charge

be placed such that it will not experience any force? 35. Three point charges of +3 µC, 5 µC and to ear -5
d √ µC are kept on the vertices of an equilateral triangle
[Ans. (1 + 3) to the left of q]
2 of side 20 cm as shown in Fig. What should be the
24. Two like charges of magnitude 1×10−9 C and 9×10−9 sign and magnitude of charge to be placed at the mid
C are separated by a distance of 1 metre. Find the point (M) of side BC of triangle ABC so that charge
point on the line joining the charges, where the force A remains in equilibrium ? [Ans. 6.49 µC, Positive]
experienced by a charge placed at that point is zero.
[Ans. 0.25 m from the charge 1 × 10−9 coulomb]
25. Two point mass each having mass m are placed in air
at distance r attract by applying force F. If medium
between these two particles is filled by material having
specific gravity 3, then what will be the new value of
force? [Ans. same]
26. Two small identical spheres having charges +10 µC 36. Four equal point charges each 16 C are placed on four
and -90 µC attract each other with a force of F new- corners of a square of side 0.4 m. Calculate force on
ton. If they are kept in contact and then separated any one of the changes. [Ans. 27.56 N]
by the same distance, than what will be the new force 37. Two opposite corners of a square carry Q charge each
between them? [Ans. 16F/9] and the other two opposite corners of the same square
27. Two equal and opposite charges are placed at a certain carry q charge each. If the resultant toys force √
on q is
distance. The force between them is F . If 25% of one zero, how are Q and q related? [Ans. q = −2 2Q ]

m
charge is transferred to other, then what will be the 38. Equal charges each of 20 µ C are placed at x = 0, 2,
new force between them? [Ans. 15F/16] 4, 8, 16 cm on X-axis. Find the force experienced by
28. (i) Dielectric constant (or Relative permittivity) for the charge at x = 2 cm. [Ans. 1.2 x 103 N]
metals is - ............ ra
(ii) Dielectric constant for a insulators is - ............
(iii) Dielectric constant for a vacuum is - ............
[Ans. (i) infinite (ii) 1 to 1000 (iii) 1 ]
39. Charges q1 = 1.5 mC, q2 = 0.2 mC and q3 = 0.5 mC,
are placed at points A, B, C respectively as shown in
Fig. If r1 = 1.2 m and r2 = 0.6 m, calculate magnitude
of resultant force on q2 . [Ans. 3.1 x 103 N]
G
29. Calculate the electrostatic force between two
α-particles separated by 3.2 × 10−15 m.
[Ans. 90 N, repulsive]
30. Calculate the force between a proton and an electron
u
separated by 0.8 × 10−15 m. [Ans. 360 N, Attractive]
31. Two point charges +9e and +e are placed at a distance ELECTRIC FIELD AND ELECTRIC FIELD
Ed

of 16 cm from each other. At what point between these LINES


charges should a third charge q be placed so that it re-
mains in equilibrium. [Ans. 12 cm from +9e] 40. Differentiate Electric Field, Electric Field Lines and
Electric Field Intensity.
32. Three point charge q1 , q2 , q3 are in line at equal dis-
tance. q2 and q3 are opposite in sign. Find the mag- 41. Write properties of Electric Field lines. Why Elec-
nitude and sign of q1 , if the net force on q3 is zero. tric field lines do not intersect each other? What does
[Ans. q1 = 4 q2 and sign as of q3 ] Electric field lines signifies?
33. If a proton, a deutron and an α paerti- 42. Why are electric field lines perpendicular at a point on
cle are kept in the same electric field (E) an equipotential surface of a conductor?
than find force and acceleration on each. 43. An electron having charge e and mass m is moving in a
eE eE uniform electric field E. What will be its acceleration.
[Ans. proton: eE, ; deutron eE, ; α: 2eE,
m 2m [Ans.
eE
]
2eE m
]
4m 44. Explain Principle of superposition. Two point charges
34. A charged particle of mass m and charge q is released of +1 µC and +4 µC are kept 30 cm apart. How
from rest in an electric field of constant magnitude E. far from the + 1 µC charge on the line joining
Find the kinetic energy of the particle after time t. the two charge, will the net electric field be zero ?
E 2 q 2 t2 [Ans. 10 cm from +1 µC]
[Ans. ]
2m 45. Two point charges + q and -2q are placed at the ver-
PRINCIPLE OF SUPERPOSITION tices of an equilateral triangle ABC. Obtain the ex-
pression for (i) the magnitude and (ii) the direction of

7 - Nikhil Kamboj — 8193989452 #edugramddun


Electric Field and Charge

the resultant electric field at the third vertex due to 57. Is the electric field due to a charge configuration with
these two charges. total charge zero, necessarily zero? Justify. [Ans. No]
46. A small ball of paper has mass 9 x 10−5 kg and car- 58. Derive expression for Electric field due to Electric
ries a charge of 5 µ C. When it is held over is another dipole at a point on
charged ball of paper at a distance of 2 cm above it, (i) axial line (ii) equatorial line
the two balls stay in equilibrium. What is the charge
59. Two equal point charges separated by 1 m distance
on the second ball? [Ans. 7.84 x 10−12 C]
experience force of 8 N. What will be the force experi-
47. A particle of mass m and charge q is thrown at a speed enced by them, if they are held in water, at the same
u against a uniform electric field E. How much dis- distance? (Given: Kwater = 80) [Ans. 0.1 N]
tance will it travel before coming to momentary rest?
[Ans. mu2 /2qE] 60. Which orientation of an electric dipole in a uniform
electric field would correspond to stable equilibrium ?
48. A particle of mass m and charge q is released from [Ans. θ = 0◦ ]
rest in uniform electric field of intensity E. Calculate
the kinetic energy it attains after moving a distance x 61. An electric dipole formed by charge +6 µC
between the plates. [Ans. qEx] and -6 µC, separated by a distance of 2 mm.
Find the dipole moment and its direction.
49. Two electric charges +q and +4q are placed at a dis-
[Ans. 12 × 10−9 , negative to positive]
tance 6a apart on a horizontal plane. Find the position
of the point on the line joining the two charges where 62. An electric dipole consists of two opposite charges of
the electric field is zero. [Ans. 2 a from + q] magnitude 1µC separated by a distance of 2cm. The
dipole is placed in an electric filed 10−5 V/m. What
50. In Fig., the electric field lines on the left have twice

m
will be the maximum torque experienced by the dipole.
the separation of those on the right. (a) If the magni-
[Ans. 2 × 10−13 Nm ]
tude of the field at A is 40 N/C, what force acts on a
proton at A ? (b) What is the magnitude of the field 63. Calculate the magnitude of the force, due to an elec-
at B ? [Ans. (a) 6.4 x 10−18 N (b) 20 N/C] tric dipole of dipole moment 3.6 × 10−29 C-m, on
ra an electron 25 nm from the centre of the dipole,
along the dipole axis. Assume that this distance
is large relative to the dipole’s charge separation.
[Ans. 6.6 x 10−15 N]
G
64. Charge q1 = + 6.0 nC is on y-axis at y = + 3 cm
and charge q2 = -6.0 n C is on y-axis at y = -3 cm.
51. Eight identical point charges of q coulomb each are Calculate force on a test charge q0 = 2 nC placed on
X-axis at x = 4 cm. [Ans. - 51.8 ĵ µN]
u
placed at the corners of a cube of each side 0.1 m.
Calculate electric field at the centre of the cube. Cal- 65. Two charges +q and −q are located at points A (0,0,-
culate the field at the centre when one of the corner 2) and B(0, 0, 2) respectively. How much work will be
Ed

charges is removed. [Ans. Zero; (1.2 x 1012 g) NC−1 ] done in moving a test charge from point P (4, 0, 0) to
52. A charge of 4 × 10−9 C is distributed uniformly over Q (-5,0,0)? [Ans. Zero]
the circumference of a conducting ring of is radius 0.3 66. Two point charges each of 5 µC but opposite in sign
m. Calculate the field intensity at a point on the axis are placed 4 cm apart. Calculate the electric field in-
of the ring at 0.4 m from its centre, and also at the tensity at a point distant 4 cm from the mid point on
centre. [Ans. 115.2 N/C ; zero] the axial line of dipole. [Ans. 108 NC−1 ]
ELECTRIC DIPOLE 67. An electric dipole of dipole moment 4 x 10−5 C-m is
placed in a uniform electric field of 103 NC−1 making
53. Define electric Dipole moment and write its direction an angle of 30° with the direction of field. Determine
and SI unit. Which orientation of an electric dipole the torque exerted by the electric field on the dipole.
in a uniform electric field would correspond to stable [Ans. 2 x 10−8 Nm]
equilibrium ?
68. An electric dipole of length 10 cm having charges
54. Draw electric lines of force due to (i) single electron ± 6 x 10−3 C, placed at 30° with respect to a
(ii) two equal positive point-charges (iii) an electric uniform electric field experiences a torque of mag-

dipole. nitude 6 3 N − m . Calculate (i) magnitude
55. Derive an expression for the torque experienced by an of electric √field (ii) the potential energy of dipole.
electric dipole kept in a uniform electric field. [Ans. (i) 2 3 × 104 NC−1 (ii) -18 J]
56. An electric dipole is held at an angle θ in a uniform 69. An electric dipole of length 4 cm, when placed with
external electric field E. Explain what happens to the its axis making an angle of 60° √
with a uniform electric
dipole on being released. field, experiences a torque of 4 3 Nm. Calculate the

8 - Nikhil Kamboj — 8193989452 #edugramddun


Electric Field and Charge

potential energy of the dipole, if it has a charge ± 8 83. A uniform electric field E⃗ = Ex îN/C for x ¿ 0 and
nC. [Ans. - 4 J] E = −Ex îN/C for x ¡ 0 exists. A right circular cylin-

GAUSS’ THEOREM AND ELECTRIC FLUX der of length l cm and radius r cm has its centre at the
origin and its axis along X-axis. Find out the net out-
ward flux. What is the net charge within the cylinder
70. (a) Define electric flux. Write its S.I. units. ? [Ans. 2 πr2 Ex (10−4 N m2 /C ; 2πr2 Ex ϵ0 C ]
(b) Consider a uniform electric field E ⃗ = 3×103 î N/C.
Calculate the flux of this field through a square surface 84. If the electric field is given by E
⃗ = 8î + 4ĵ + 3k̂ N/C,
of area 10 cm2 when calculate the electric flux through a surface of area 100
(i) its plane is parallel to the y-z plane, and m2 lying in X-Y plane. [Ans. 300 N m2 /C]
(ii) the normal to its plane makes a 60° angle with the 85. A spherical Gaussian surface encloses a charge of
x-axis. 8.85 x 10−8 C (i) Calculate the electric flux pass-
71. State Gauss’s theorem in electrostatics. ing through the surface (ii) If the radius of Gaus-
72. Derive an expression for electric field intensity at a sian surface is doubled, how would the flux change?
point due to a uniformly charged thin conducting [Ans. 104 Nm2 /C, No change]
spherical shell at points inside, outside and on the sur- 86. A rectangular surface of sides 10 cm and 15 cm
face of shell. is placed inside a uniform electric field of 25 V/m,
73. Derive expression for Electric field due to uniformly such that normal to the surface makes an angle of
charged non-conducting solid Sphere at a point (a) 60° with the direction of electric field. Find the
inside the sphere, (b) on the surface of sphere (c) out- flux of electric field through the rectangular surface.

m
side the sphere. Draw the graph for the same. [Ans. 1.8 x 10−1 Nm2 /C]
74. Derive expression for Electric field due to uniformly 87. If the electric field is given by (6î + 4ĵ + 4k̂). calcu-
charged uniformly charged infinite wire (line of charge) late the electric flux through a surface of area 20 units
using Gauss’s theorem. lying in Y-Z plane. [Ans. 120 units]
75. Derive expression for Electric field due to uniformly
charged infinite thin sheet.
76. Derive expression for Electric field due to two infinite
parallel uniformly charged thin sheets.
ra 88. The electric field in a certain region of space is
(5î + 4ĵ − 4k̂) × 105 N/C. Calculate electric flux
due to this field over an area of (2î − ĵ) × 10−2 m2 .
[Ans. 6 × 103 N/Cm2 ]
G
77. An infinite line charge produces an electric field of 89. A charge of 2 x 10−9 C is placed on a corner of a cube
9.0 × 104 N/C at a distance of 2 cm. What is the of side 1 m. Find the electric flux passing through this
linear charge density? [Ans. 0.1 micro C/m] cube. [Ans. 28.25 N/C m2 ]
u
78. A charge Q is uniformly distributed over a large square 90. Five thousand lines of force enter a certain volume of
plate of copper. The electric field at point P very close space and three thousand lines emerge from it. What
to the centre of the plate is 10 V/m. If the copper plate is the total charge in coulomb within this volume?
Ed

is replaced by a plastic plate of the same geometrical [Ans. - 1.77 x 10−8 C]


dimensions and carrying the same charge Q uniformly
91. A positive charge of 17.7 µC is placed at the cen-
distributed, then what will be the electric field at the
tre of a hollow sphere of radius 0.5 m. Calculate
point P . [Ans. 10 V/m]
the flux density through the surface of the sphere
79. The inward and outward electric flux for a closed sur- [Ans. 6.4 x 105 N/C]
face in units of N-m2 /C are respectively 8 × 103 and
4 × 103 . Then what will be the total charge inside the 92. An infinite line charge produces a field of 9 x 104 N/C
surface. [Ans. −(4 × 103 )ϵ0 C] at a distance of 4 cm. Calculate the linear charge
density.[Ans. 2 x 10−7 C/m]
80. If the electric field is given by 5î + 4ĵ + 9k̂. What will
be electric flux through a surface of area 20 unit lying 93. An electron is revolving around a long line charge hav-
in the Y-Z plane. [Ans. 100 units] ing charge density 2 x 10−8 C/m. Find the kinetic en-
ergy of the electron, assuming that it is independent
81. What will be the total electric flux leaving spherical of the radius of electron’s orbit. [Ans. 2.88 x 10−17 J]
surface of radius 1 cm if an electric dipole is placed at
centre of sphere. [Ans. zero] 94. A large plane sheet of charge having surface charge
density 5 x 10−6 Cm−2 lies in XY plane. Find the
82. Given a uniform electric field E = 5 x 10 îN C . 3 −1
electric flux through a circular area of radius 0.1 m, if
Find the flux of this field through a square of
the normal to the circular area makes an angle of 60°
10 cm on a side, whose plane is parallel to Y-Z
with the Z-axis. [Ans. 4.44 x 103 Nm2 /C]
plane. What would be the flux through the same
square if plane makes an angle of 30° with X-axis ? CASE STUDY
[Ans. 50 Nm2 /C; 25 Nm2 /C]

9 - Nikhil Kamboj — 8193989452 #edugramddun


Electric Field and Charge

95. Case Study: In 1909, Robert Millikan was the first to electrostatics. It also discusses the principle of super-
find the charge of an electron in his now-famous oil- position, which states that the net electrostatic force
drop experiment. In that experiment, tiny oil drops experienced by a charge due to multiple charges is
were sprayed into a uniform electric field between a simply the vector sum of the individual forces exerted
horizontal pair of oppositely charged plates. The drops by each charge independently. Further, the chapter
were observed with a magnifying eyepiece, and the introduces the concept of an electric field, defined as
electric field was adjusted so that the upward force the electric force experienced by a unit positive charge
on some negatively charged oil drops was just suffi- at a point in space, and illustrates the way these fields
cient to balance the downward force of gravity. That are represented using field lines.
is, when suspended, upward force qE just equaled Mg.
Millikan accurately measured the charges on many oil (i) What is the basic property of electric charges?
drops and found the values to be whole number mul-
tiples of 1.6 × 10−19 C the charge of the electron. For a) Like charges repel, and unlike charges attract
this, he won the Nobel prize. b) Like charges attract, and unlike charges repel
c) All charges repel
d) All charges attract

(ii) What does the principle of superposition state in


terms of electrostatic forces?
a) The net electrostatic force is equal to the sum of
the individual forces

m
b) The net electrostatic force is equal to the differ-
ence of the individual forces
(i) If a drop of mass 1.08×10−14 kg remains stationary c) The net electrostatic force is independent of the
in an electric field of 1.68 × 105 N/C, then the charge individual forces
of this drop is
a) 6.40 × 10−19 C
c) 1.60 × 10−19 C
b) 3.20 × 10−19 C
d) 4.80 × 10−19 C
ra d) The net electrostatic force is inversely propor-
tional to the individual forces

()iii) What is an electric field?


G
(ii) Extra electrons on this particular oil drop (given a) The region around a charge where magnetic force
the presently known charge of the electron) are is experienced
a) 4 b) 3 c) 5 d) 8 b) The region around a charge where gravitational
force is experienced
u
(iii) A negatively charged oil drop is prevented from c) The region around a charge where electrostatic
falling under gravity by applying a vertical electric force is experienced
field 100 V/m.If the mass of the drop is 1.6 × 10−3
Ed

d) The region around a charge where nuclear force


g, the number of electrons carried by the drop is (g=
is experienced
10 m s−2 )
a) 1018 b) 1015 c) 1012 d) 109 (iv) How is the electric field defined?

(iv) The important conclusion given by Millikan’s ex- a) As the electric force experienced by a unit neg-
periment about the charge is ative charge
b) As the electric force experienced by a unit posi-
a) charge is never quantized tive charge
b) charge has no definite value
c) As the gravitational force experienced by a unit
c) charge is quantized
positive charge
d) charge on oil drop always increases
d) As the magnetic force experienced by a unit pos-
96. Case Study: Electric Charges and Fields, focuses on itive charge
the basic properties of electric charges, the concept of
electric fields, and the forces acting within these fields. (v) How are electric fields represented?
The chapter introduces the idea that like charges re- a) By field lines b) By electric charges
pel, and unlike charges attract, a fundamental law of c) By vectors d) By matrices

10 - Nikhil Kamboj — 8193989452 #edugramddun


Electric Potential and Capacitance
Important Formulae and Notes

W (j) If several slabs of dielectric constants,


▶ Electric Potential, V =
q0 K1 , K2 , K3 , ....., and respective thicknesses
where q0 = test charge t1 , t2 , t3 , ...., be placed between the two plates
▶ Electric potential due to charge q of a capacitor, then the capacitance will be
ϵ0 A
1 q C=
V =
h i
4πKϵ0 r d − (t1 + t2 + t3 ...) + Kt11 + Kt22 + Kt33 + ...
(k) If the parallel plate capacitor consists of two cir-
1 q1 q2 cular plates of radius r separated by a distance
▶ Electric Potential energy, U =
4πϵ0 r d, in air or vaccum, then capacity,
Charge Q ϵ0 πr2
▶ Capacity of conductor, C = = C=
Potential V d
1 coulomb (l) If dielectric medium of constants K1 and K2 are
▶ 1 Farad= filled in between the plates of the capacitor, then
1 volt
▶ Capacity of a spherical conductor (in air or vacuum) effective capacitance of the system will be given
C = 4πϵ0 r by
where r = radius of spherical conductor ϵ0 A
C= (K1 + K2 )

m
Q 2d
▶ Capacity or capacitance C = (m) When a dielectric of constant K is introduced into
V a parallel plate capacitor as shown in the figure,
where C is in farad, and charge Q in coulomb
then effective capacitance will be
▶ Parallel Plate capacitor
(a) Surface charge density, σ =
where A = area of plate
q = charge on plate
A
q ra C′ =
C(K + 1)

Where C = initial capacity =


2
ϵ0 A
d
(b) Intensity of the electric field,
G
σ q (n) If the parallel plate capacitor is made with three
E= = different dielectric slabs as shown in the figure,
ϵ Aϵ0
(c) Potential difference between the plates, V = then effective capacity of the capacitor will be
Ed
u
Ed =
 
ϵ0 A K1 K2 K3
Aϵ0 C= +
q ϵ0 A d 2 2 + K2 K3
(d) Capacity of the capacitor, C = =
Ed

V d (o) If the parallel plate capacitor is made with three


(e) If the gap between the plates is filled with di-
different slabs as shown in figure, then effective
electric medium of dielectric constant K, then
Kϵ0 A capacity
C=
d 2K1 (K2 + k3
 
ϵ0 A
(f) Force between the plates, C=
d 2K1 + K2 + K3
1 q2 1
F = = CE 2 d
2 ϵ0 A 2 ▶ Parallel combination of Capactitors
(g) When there is vaccum in between two plates of equivalent capacity in Parallel
parallel plate capacitor, then capcacity of paral-
lel plate capacitor Cp = (C1 + C2 + C3 + ....Cn )
ϵ0 A
C= equivalent capacity in Series
d
(h) If dielectric slab of thickness t is introdued in a 1 1 1 1 1
parallel plate air capacitor, whose plates are sep- = + + + .... +
Cs C1 C2 C3 Cn
arated by a distane d, then capacity of the capac-
itor increases and will be given by ▶ Energy of a charged capacitor
ϵ0 A
C= (a)
d−t+ K r
1 Q2 1 C 1
(i) If a metal sheet of thickness t is introduced then E= = = QV
2 C 2V 2 2
capacity is increases and is given by
ϵ0 A where Q = amount of charge stored
C+ C = capacity of the capacitor
d−t
Electric Potential and Capacitance

▶ if n identical drops each of charge q, capacity C and C ′ = C(n)1/3


potential V are combined to form a big drop, then (c) Potential of the big drop,
(a) Charge on the big drop, V ′ = V (n)2/3
q ′ = nq (d) potential energy of the big drop,
(b) Capacity of the big drop, = n5/3 × energy of each drop

Multiple Choice Questions

1. The P.E. of an electric dipole is maximum when it a) 4.5 × 10−3 N b) 5.4 × 10−3 N
makes an angle θ with electric field. The value of θ is c) 4.5 × 10−2 N d) 5.4 × 10−2 N
a) π/2 b) π c) 0 d) 3π/2
9. Four charges 2C, -3C, -4C and 5C respectively are
2. Equipotentials at a great distance from a collection of placed at all corners of a square. Which of the fol-
charges whose total sum is not zero are approximately lowing statement is true for the point of intersection
of diagonals?
a) Spheres b) Planes
c) Paraboloids d) Ellipsoids a) Electric field is zero but electric potential is non-
zero
3. A test charge is moved from lower potential point to b) Electric field is non-zero but electric potential is
a higher potential point. The potential energy of test zero
charge will c) Both electric field and electric potential are zero

m
a) remain the same b) Increase d) Neither electric field nor electric potential is zero
c) Decrease d) become zero
10. The work done required to put the four charge to-
4. Which of the following statement is true? gether at the corners of a square of side a as shown in
a) Electrostatic force is a conservative force.
b) Potential at a point is the work done per unit
charge in bringing a charge from any point to
infinity.
ra the fig. is
G
c) Electrostatic force is non-conservative 1 q2 2.6 q 2
d) Potential is the product of charge and work. a) b) −
4πϵ0 a 4πϵ0 a
2.6 q 2
5. If a unit positive charge is taken from one point to c) + d) none of these
u
4πϵ0 a
another over an equipotential surface, then
a) work is done on the charge. 11. A solid conducting sphere of radius R is
Ed

b) work is done by the charge. charged uniformly. The electrostatic potential


c) work done is constant. V is plotted as a function of displacement r
d) no work is done. from the centre of the sphere. Which of the
given ahead represents the best resulting curve?
6. A hollow metal sphere of radius 5 cm is charged so
that the potential on its surface is 10 V. The potential
at the centre of the sphere is
a) 0V
b) 10 V
c) Same as at point 5 cm away from the surface
d) Same as at point 25 cm away from the surface

7. Three charges Q, +q and +q are placed at the vertices


of an equilateral triangle of side l. If the net electro-
static energy of the system is zero, then Q is equal
to
a) -q b) +q c) Zero d) –q/2

8. The force between two small charged spheres having 12. A positive charge is moved from a low potential point
charges of 1 × 10−7 C and 2 × 10−7 C placed 20 cm (A) to a high potential point (B). Then the electric
apart in air is potential energy

12 - Nikhil Kamboj — 8193989452 #edugramddun


Electric Potential and Capacitance

a) increases a) giving some protons to it


b) decreases b) removing some electrons from it
c) will remain the same c) giving excess of electrons to it
d) nothing definite can be predicted d) removing some neutrons from it

13. The variation of potential with distance R from a fixed 20. A regular hexagon of side 10 cm has a charge 5 µC
point is as shown. The electric field at R = 5 m is at each of its vertices. Calculate the potential at the
centre of the hexagon
a) 2.7 × 10−6 V b) 5.4 × 10−6 V
c) 2.7 × 106 V d) 2.7 × 10−12 V

21. The work done in taking a unit positive charge from


P to A is WA and from P to B is WB . Then

a) 2.5 V/m b) −2.5 V/m


c) 2/5 V/m d) −(2/5) V/m

14. The electric potential at a point (x, y) in the xy-plane


is given by V = – Kxy. The electric field intensity at a) WA > WB b) WA < WB
a distance r from the origin varies as c) WA = WB d) WA + WB = 0
a) r2 b) r c) 1/r d) 2r−2 22. One ampere current is passed for one second is called

m
as one:
15. For the isolated charged conductor shown in the fig-
a) b) Unit c) Coulomb d) Ohm
ure, the potentials at points A, B, C and D are VA ,
Ampere
VB , VC and VD respectively. Then

a) VA = VB > VC > VD b) VD > VC > VB = VA


ra 23. In case of infinite long wire electric field is proportional
to
a)
1
r
b) 2
r
1
c) 3
1
r
d) r0
G
c) VD > VC > VB > VA d) VD = VC = VB = VA 24. For the isolated charged conductor of above figure,
the electric fields at points A, B, C and D are
16. Two insulated charged spheres of radii R1 and R2 hav- EA , EB , EC andED respectively. Then
ing charges Q1 and Q2 respectively are connected to
u
each other; then there is
a) no change in the energy of the system
b) an increase in the energy of the system
Ed

a) EA = EB > EC > ED
c) always a decrease in the energy of the system
b) ED > EC > EB = EA
d) a decrease in energy of the system unless
c) ED > EA > EC , EB = 0
Q1 R2 = Q2 R1
d) EB = 0, EA = EC = ED
17. Equipotential surfaces associated with an electric field 25. 1000 small water drops each of radius r and charge
which is increasing in magnitude along the X-direction q coalesce together to form one spherical drop. The
are potential of the bigger drop is larger than that of the
a) planes parallel to YZ-plane smaller one by a factor
b) planes parallel to XY-plane a) 1000 b) 100 c) 10 d) 1
c) planes parallel to XZ-plane
d) coaxial cylinders of increasing radii around the 26. Two parallel metal plates carry charges +Q and –Q
x-axis respectively. A test charge q0 placed between them
experiences a force F. Now the separation between the
18. If a unit positive charge is taken from one point to plates is doubled, then the force on the test charge will
another over an equipotential surface, then be
a) work is done on the charge a) F b) 2F c) F/2 d) F/4
b) work is done by the charge
c) work done is constant 27. The work done in increasing the voltage across the
d) no work is done plates of a capacitor from 5 V to 10 V is W. The work
done in increasing the voltage from 10 V to 15 V will
19. A body can be negatively charged by be

13 - Nikhil Kamboj — 8193989452 #edugramddun


Electric Potential and Capacitance

a) W b) 4W/3 c) 5W/3 d) 2W 35. A parallel plate capacitor with air as medium between
the plates has a capacitance of 10 µF. The area of the
28. A capacitor consists of capacitor is divided into two equal halves and filled
with two media having dielectric constants K1 = 2
a) Two insulators separated by a conductor
and K2 = 4. The capacitance of the system will now
b) Two conductors separated by an insulator
be
c) Two insulators only
d) Two conductors only a) 10 µF b) 20 µF c) 30 µF d) 40 µF
36. Capacitance of capacitor shown in figure is
29. Force acting upon a charged particle kept between the
plates of a charged capacitor is F. If one of the plates
of the capacitor is removed, force acting on the same
particle will becomes
a) 0 b) F/2 c) F d) 2F ϵ0 A ϵ0 A K1 + K2
 
a) (K1 + K2 ) b)
d d  2
30. There is an air-filled 1 pF parallel plate capacitor. ϵ0 A ϵ0 A K 1 − K 2

When the plate separation is doubled and the space c) 2(K1 + K2 ) d)
d d 2
is filled with wax, the capacitance increases to 2 pF.
The dielectric constant of wax is 37. A parallel plate capacitor of area A, plate separation d
and capacitance C is filled with three different dielec-
a) 2 b) 4 c) 6 d) 8
tric materials having dielectric constants K1, K2 and

m
K3 as shown. If a single dielectric material is to be
31. A capacitor is connected to a cell of emf E having some used to have the same capacitance C in this capacitor,
internal resistance r. The potential difference across then its dielectric constant K is given by: (A = Area
the
a) cell is < E
c) capacitor is > E
b) cell is E
d) capacitor is < E
ra
32. The plates of a parallel plate capacitor are not exactly of plates)
G
parallel. The surface charge density
1 1 1 1 1
a) is lower at the closer end a) = + + +
K K1 K1 K2 2K3
b) will not be uniform 1 1 1
b) = +
c) each plate will have the same potential at every K K1 + K2 2K3
u
point K1 K2
c) K = + 2K3
d) both (b) and (c) K1 + K2
d) K = K1 + K2 + 2K3
Ed

33. A parallel plate capacitor is charged. If the plates are 38. Capacitance of a capacitor becomes 7/6 times its orig-
pulled apart inal value if a dielectric slab of thickness t = (2/3) d
a) the capacitance increases is introduced between plates. d is the separation be-
b) the potential difference increases tween plates. The dielectric constant of dielectric slab
c) the total charge increases is
d) the charge and potential difference remain the a) 14/11 b) 11/14 c) 7/11 d) 11/7
same
39. An air capacitor C connected to a battery of emf V
34. A parallel plate condenser with plate area A acquires a charge q and energy E. The capacitor C
shown in the figure. The capacitance will be is disconnected from the battery and a dielectric slab
is placed between the plates. Which of the following
statements is correct?
a) V and q decreases but C and E increases.
b) V remains unchanged, but q, E and C increases.
c) q remains unchanged, C increases, V and E
decreases
d) q and C increases but C and E decreases.
ϵ0 A K1 + K2
 
ϵ0 A
a) (K1 + K2 ) b)
d  d  K1 K2  40. Seven capacitors each of capacitance 2 µF are to
2ϵ0 A 2ϵ0 A K1 + K2 be connected to obtain a capacitance of (10/11) µF.

K1 K2
c) d) Which of the following combinations is possible?
d K1 + K2 d K1 K2

14 - Nikhil Kamboj — 8193989452 #edugramddun


Electric Potential and Capacitance

a) 5 in parallel, 2 in b) 4 in parallel, 3 in a) All in series


series series b) All in parallel
c) 3 in parallel, 4 in d) 2 in parallel, 5 in c) Two in parallel and third in the series with this
series series combination
d) Two in series and third in parallel across this
41. The force between the plates of a parallel plate ca- combination
pacitor of capacitance C and distance of separation of
the plates d with a potential difference V between the 49. Three capacitors each of capacity 4 µF are to be con-
plates is nected in such a way that the effective capacitance is
CV 2 C 2V 2 C 2V 2 dV 2 6 µF. This can be done by
a) b) c) d) a) connecting all of them in series
2d 2d2 d2 C
b) connecting them in parallel
42. A parallel plate capacitor is made by stacking n equally c) connecting two in series and one in parallel
spaced plates connected alternately. If the capacitance d) connecting two in parallel and one in series
between any two plates is C, then the resulting capac-
itance is 50. A network of four capacitors of capacity equal to
a) C b) nC c) (n-1)C d) (n+1)C C1 = C, C2 = 2C, C3 = 3C and C4 = 4C are con-
nected to a battery as shown in the figure. The ratio
43. The capacitance C of a capacitor is of the charges on C2 and C4 is
a) independent of the charge and potential of the
capacitor

m
b) dependent on the charge and independent of
potential
c) independent of the geometrical configuration of
the capacitor
d) independent of the dielectric medium between
the two conducting surfaces of the capacitor

44. A spherical drop of capacitance 1 µF is broken into


ra a) 4/7 b) 3/22 c) 7/4 d) 22/3

51. A capacitor of 4 µF is connected as shown in the cir-


cuit. The internal resistance of the battery is 0.5 Ω.
G
eight drops of equal radius. The radius of each small The amount of charge on the capacitor plates will be
drop is
a) 1/2 µF b) 1/4 µF c) 1/8 µF d) 1/16
µF
u

45. Three capacitors 50 µf, 25 µf and 100 µf are connected


in series across 240 v supply. Find the total capaci-
Ed

tance value. a) 0 b) 4 µC c) 16 µC d) 8 µC
a) 175 µf b) 150 µf
c) 14.28 µf d) 12.25 µf 52. The capacitor of a parallel plate condenser does not
depend upon
46. The capacitance between two plates increases with a) area of the plates
a) Shorter plate area and higher applied voltage b) medium between the plates
b) Shorter play area and shorter distance between c) distance between the plates
them d) metal of the plates
c) Larger plate area, longer distance between plates
and higher applied voltage 53. The energy required to charge a parallel plate con-
denser of plate separation d and plate area of cross-
d) Larger plate area and shorter distance between
section A such that the uniform electric field between
plates
the plates E, is
47. The capacitance is a circuit component that oppose 1 ϵ0 E 2 ϵ0 E 2 1 ϵ0 E
a) b) c) ϵ0 E 2 Ad d)
the change in circuit 2 Ad Ad 2 Ad
a) Current b) Voltage 54. In a parallel plate capacitor, the capacity increases if
c) Impedance d) None of the above
a) area of the plate is decreased.
b) distance between the plates increases.
48. Three capacitors each of the capacity C are given. The
2C c) area of the plate is increased.
resultant capacity can be obtained by using them d) dielectric constantly decreases
3

15 - Nikhil Kamboj — 8193989452 #edugramddun


Electric Potential and Capacitance

55. A capacitor has some dielectric between its plates, and 58. In which of the following forms is the energy stored in
the capacitor is connected to a dc source. The battery a capacitor?
is now disconnected and then the dielectric is removed, a) Charge b) Potential
then c) Capacitance d) Electric field
a) capacitance will increase.
b) energy stored will decrease. 59. The capacitance of earth, viewed as a spherical con-
c) electric field will increase. ductor of radius 6408 km is
d) voltage will decrease. a) 1420 b) 712 µF c) 680 µF d) 540 µF
µF
56. Two metal plates form a parallel plate capacitor. The
distance between the plates is d. A metal sheet of 60. When a metal plate is introduced between the two
thickness d/2 and of the same area is introduced be- plates of a charged capacitor and insulated from them,
tween the plates. What is the ratio of the capacitance then which of following statement(s) is/are correct ?
in the two cases? I. The metal plate divides the capacitor into two ca-
pacitors connected in parallel to each other
a) 2 : 1 b) 3 : 1 c) 4 : 1 d) 5 : 1
II. The metal plate divides the capacitors into two ca-
pacitors connected in series with each other
57. Two capacitors of capacitance 6 µF and 4 µF are put III. The metal plate is equivalent to a dielectric of zero
in series across a 120 V battery. What is the potential dielectric constant
difference across the 4 µF capacitor ?
a) I only b) II only

m
a) 72 V b) 60 V c) 48 V d) zero c) I and II only d) I, II and III

Practice Questions and Numerical

POTENTIAL DIFFERENCE AND


WORKDONE

1. TWo charges −q and +q are located at points


A(0, 0, −a) and B(0, 0, +a) respectively. How much
ra of sphere (ii) at a distance of 0.1 cm from the centre
of sphere. [Ans. 1.08 x 106 V, 1.08 x 106 V]
8. Two point charges, one of + 100 µC and another
of - 400 µC, are kept 30 cm apart. Find the point
G
work is done in moving a test charge from point of zero potential on the line joining the two charges.
P (7, 0, 0) to Q(−3, 0, 0) ? [Ans. Zero] [Ans. 6 cm from 100 µC charge]
2. If 100 J of work must be done to move electric charge 9. Two point charges 4 µC and -2 µC are separated by
equal to 4 C from a place where potential is -10 V to a distance of 1 m in air. Calculate at what point on
u

another place where potential is V volt, find the value the line joining the two charges is the electric potential
of V. [Ans. 15 V] zero ? [Ans. 2/3 m from 4 µC]
Ed

3. If 10 J of work is to be done in moving a charge of 10. The electric field at a point due to a point charge
-200 C from A to B, which of the two points is at is 30 N/C, and the electric potential at that point
higher potential ? What is the potential difference ? is 15 J/C. Calculate the distance of the point
[Ans. A ; - 0.05 V] from the charge and the magnitude of the charge.
[Ans. 0.5 m ; 0.83 x 104 C]
ELECTRIC POTENTIAL
11. A point charge of 10−8 C is situated at the origin of
4. A hollow metal sphere of radius 5 cm is charged so co-ordinates. Find the potential difference between
that the potential on its surface is 10 V. What will be the points A (4, 4, 2) and B (1, 2, 2). [Ans. -15 V]
the potential at the centre of the sphere? [Ans. 10 V] 12. Twenty seven charged water droplets each with a di-
5. The electric field at a point due to a point charge ameter of 2 mm and a charge of 10−12 C coalesce to
is 20 N/C and electric potential at that point form a single drop. Calculate the potential of the big-
is 10 J/C. Calculate the distance of the point ger drop. [Ans. 81 V]
from the charge and the magnitude of the charge. 13. A charge of 20 g C produces an electric field. Two
[Ans. 0.5 m ; 0.55 x 10−9 C] points are 10 cm and 5 cm from this charge. Find the
6. Two point charges q and −2q are kept ’d’ distance values of potentials at these points and calculate work
apart. Find the location of the point relative to charge done to take an electron from one point to the other.
q at which potential due to the system of charges is [Ans. 1.8 x 106 V, 3.6 x 106 V, 2.88 x 10−13 J]
zero. [Ans. d/3 from charge] 14. Calculate the voltage required to balance an oil drop
7. A charge of 24 µC is given to a hollow metallic sphere carrying 10 electrons, when located between plates of
of radius 0.2 m. Find the potential (i) at the surface a capacitor, which are 5 mm apart. Given mass of

16 - Nikhil Kamboj — 8193989452 #edugramddun


Electric Potential and Capacitance

drop = 3 x 10−16 kg, charge on electron = 1.6 x 10−19 25. Two identical particles, each having a charge of 2 x
C and g = 9.8 m/s2 . [Ans. 9.2 V] 10−4 C and mass of 10 g, are kept at a separation
15. A 500 µC charge is at the centre of a square of side 10 of 10 cm and then released. What would be the
cm. Find the work done in moving a charge of 10 µC speeds of particles when the separation becomes large
between two diagonally opposite points on the square. ? [Ans. 600 m/s]
[Ans. zero] 26. Two point charges A and B of value + 15 µC and +
16. A proton and an α particle are accelerated through 9 µC are kept 18 cm apart in air. Calculate the work
the same potential. Find the ratio of ve- done when charge B is moved by 3 cm towards A.
locities and linear momenta acquired by them. [Ans. 1.35 J]
1 √ 27. Set up an arrangement of three point charges: +q, +2q
[Ans. Vα /Vp = √ ; Pα /Pp = 2 2]
2 and x q separated by equal finite distances so that elec-
POTENTIAL GRADIENT AND ELECTRIC tric potential energy of the system is zero. What is x
−2q
FIELD ? [Ans. Charges +q, +2q and at vertices of an
3
equilateral triangle ; x = — 2/3]
17. An infinite plane sheet of charge density 10−8 Cm−2
is held in air. In this situation how far apart 28. Three charges −q, Q and −q are placed at equal dis-
are two equipotential surfaces, whose p.d is 5 V ? tances on a straight line. If the potential energy of the
[Ans. 8.85 mm] system of three charges is zero, then what is the ratio
of Q : q ? [Ans. 1 : 4]
18. An electric field of 20 N/C exists along the X-axis in
space. Calculate the potential difference (VB − VA ) 29. N drops of mercury of equal radii and possessing

m
where the co-ordinates of A and B are given by (i) A equal charges combine to form a big drop. Compare
(0, 0) ; B (4 m, 2 m) (ii) A (4 m, 2 m) ; B (6 m, 5 m) the charge, capacitance and potential of bigger drop
[Ans. - 80 V ; - 40 V] with the corresponding quantities of individual drops.
[Ans. N ; N 1/3 ; N 2/3 ]
19. What is potential gradient at a distance of 10−12 m
from the centre of the platinum nucleus ? What
is the potential gradient at the surface of the
nucleus ? Atomic number of platinum is 78
and radius of platinum nucleus is 5 x 10−15 m.
ra 30. When electrons equal to Avogadro number are trans-
ferred from one conductor to another, a potential
difference of 106 V appears between them. Calcu-
late the capacity of the system of two conductors.
[Ans. 9.637 x 10−2 F]
G
[Ans. 1.123 x 1017 V/m ; 4.5 x 1021 V/m]
20. If the potential in the region of space around 31. A charged spherical conductor has a surface density of
the point (- lm, 2m, 3m) is given by 0.07 C cm−2 , When the charge is increased by 4.4 C,
V = (10x2 + 5y 2 − 3z 2 ), calculate the three the surface density changes to 0.084 C cm−2 . Find the
u
components of electric field at this point. initial charge and capacitance of the spherical conduc-
[Ans. Ex = 20V /m; Ey = −20V /m; Ez = 18V /m] tor. [Ans. 22 C ; 5.56 x 1012 F]
CAPACITANCE & PARALLEL PLATE
Ed

ELECTROSTATIC POTENTIAL ENERGY


CAPACITORS

21. Three charges Q, +q and +q are placed at the ver- 32. What is meant by electrical capacitance of a conduc-
tices of an equilateral triangle of side l. If the net tor? Derive the formula for the capacitance of an iso-
electrostatic energy of the system is zero, then find Q. lated metallic sphere. [Ans. C = 4πϵ0 r]
[Ans. −q/2] 33. Explain the principle of capacitor. Draw diagram and
22. Two point charges 20 × 10−6 C and −4 × 10−6 C are explain its working.
separated by a distance of 50 cm in air. 34. Derive expression for capacitance of parallel plate ca-
(i) Find the point on the line joining the charges, ϵ0 A
where the electric potential is zero. pacitor. [Ans. C = ]
d
(ii) Also find the electrostatic potential energy of the 35. Derive expression for capacitance of parallel plate ca-
system. [Ans. (i) 5/14 m from 20 × 10−6 C on the line joiningpacitor with dielectric slab of constant ’K’ and thick-
the charges (ii) 1.45 J]ness ’t’ is inserted between the plates of capacitor.
23. A test charge is moved from lower potential point to a ϵ0 A
[Ans. C = ]
higher potential point. How will the potential energy d − t + t/K
of test charge will change. [Ans. Increases] 36. A parallel plate air capacitor consists of two circular
24. Two charges of magnitude 5 nC and — 2 nC are placed plates of diameter 8 cm. At what distance should the
at points (2 cm, 0, 0) and (x cm, 0, 0) in a region of plates be held so as to have the same capacitance as
space. Where there is no other external field. If the that of a sphere of diameter 20 cm ? [Ans. 4 mm]
electrostatic potential energy of the system is — 0.5 37. Calculate the area of paper required to construct a
µJ. What is the value of x ? [Ans. x = 4 cm] parallel plate capacitor of 0.004 µF , if the dielectric

17 - Nikhil Kamboj — 8193989452 #edugramddun


Electric Potential and Capacitance

constant of paper be 2.5 and its thickness 0.025 mm.


[Ans. 4.52 x 10−3 m2 ]

38. What distance should the two plates each of area .


0.2 x 0.1 m2 of an air capacitor be placed in order to
have the same capacitance as a spherical conductor of
radius 0.5 m ? [Ans. 3.18 mm]

39. The plates of a parallel plate capacitor are 5 mm


apart and 2 m2 in area. The plates are in vac-
uum. A potential difference of 1000 volt is ap- 47. Two capacitors have a capacitance of 5 µF when con-
plied across the capacitor. Calculate (i) the capac- nected in parallel and 1.2 µF when connected in series.
itance (ii) the charge on each plate and (iii) elec- Calculate their capacitances. [Ans. 2 µF , 3 µF ]
tric intensity in the space between the two plates. 48. Connect three capacitors of 3 µF , 3 µF and 6
[Ans. 3.54 x 10−9 F ; 3.54 x 10−6 C ; 2 x 103 N/C] µF such that their equivalent capacitance is 5 µF .
[Ans. Series combination of 3 µF and 6 µF in
40. A sphere of radius 0.03 m is suspended within a hol- parallel with 3 µF ]
low sphere of radius 0.05 m. If the inner sphere is 49. Find the equivalent capacitance between the points
charged to a potential of 1500 volt and outer sphere is P and Q as shown in Fig. Given C = 18 µF and
earthed. Find the capacitance and the charge of the C1 = 12µF . [Ans. 11 µF ]
inner sphere. [Ans. 8.33 pF, 1.25 x 10−8 C]

m
41. A co-axial cable used in transmission line has inner ra-
dius of 0.1 mm and outer radius of 0.6 mm. Calculate
capacitance per metre of the cable. [Ans. 71.5 pF/m]

42. A spherical capacitor has an outer sphere of radius


0.15 m and the inner sphere of diameter 0.2 m. The
outer sphere is earthed and the inner sphere is given
ra 50. There are infinite number of capacitors each of capac-
G
a charge of 6 µC. The space between the concentric itance 1 µF. They are connected in rows, such that
spheres is filled with a material of dielectric constant 6. number of capacitors in first, second, third rows are
Calculate capacitance And potential of inner sphere. respectively 1, 2, 4, 8 ...... . The rows of these capaci-
[Ans. 0.2 x 10−9 F ; 18 x 104 V] tors are connected between A and B, as shown in Fig.
u
Determine the equivalent capacitance of the network
43. A cable consisting of a wire 3 mm in diameter is in- between points A and B. [Ans. 2 µF ]
sulated with 3 mm thick dielectric of relative permit-
Ed

tivity 10. Calculate the capacitance of 1 km length of


the cable. [Ans. 0.506 µF ]

GROUPING OF CAPACAITORS

44. Two capacitors of capacitances 3 µF and 6 µF are


charged to potentials of 2 V and 5 V respectively.
These two charged capacitors are connected in series.
Find the potential across each of the capacitors now.
51. Find the equivalent capacitance of the ladder between
[Ans. 12 V ; 6 V]
points A and B. [Ans. 2 µF]
45. If C1 = 20µF , C2 = 30µF and C3 = 15µF and the
insulated plate of C1 be at a potential of 90 V, one
plate of C3 being earthed : What is the p.d between
the plates of C2 , three capacitors being connected in
series. [Ans. 20 V]

46. For the network shown in Fig, find the potential differ-
ence between points A and B, and that between B and 52. Find out the potential difference across the plates goal
C in the steady state. [Ans. VAB = 25V, VBC = 75V ] of 1 µ F capacitor in Fig. [Ans. 3.82 V]

18 - Nikhil Kamboj — 8193989452 #edugramddun


Electric Potential and Capacitance

59. A parallel plate capacitor is filled with dielectrics


as shown in Fig. What is its capacitance?
2ϵ0 AK1 K2
[Ans. ]
53. Find the p.d. between points A and B of arrangement d(K1 + K2 ))
shown in Fig. [Ans. - 8 V]

60. Calculate the charges which will flow in sections


1 and 2 in Fig. when key K is pressed.
54. The equivalent capacitance of the combination be- EC1 C2
tween A and B in the given Fig. is 4 µ F [Ans. EC1 ; ]
(C1 + C2 )

m
(i) Calculate capacitance of capacitor C. (ii) Cal-
culate charge on each capacitor if a 12 V bat-
tery is connected across A and B. (iii) What
will be the potential drop across each capacitor ?
ra
G
[Ans. 5 µ F; 48 µ C; 2.4 V, 9.6 V]
55. The capacities of three capacitors are in the ratio
61. In the circuit shown in Fig. the emf of each bat-
of 1:2:3. Their equivalent capacity in parallel is
tery is E = 12 volt and the capacitances are C1 =
greater than their equivalent capacity in series by
2.0 µ F and C2 = 3.0 µ F. Find the charges which
u
60/11 pF. Calculate their individual capacitances.
flow along the paths 1, 2, 3 when key K is pressed.
[Ans. 1 pF, 2 pF, 3 pF]
[Ans. 24 µ C; -36 µ C; 12 µ C]
Ed

56. A combination of four identical capacitors is shown


in Fig. If resultant capacitance of the combination be-
tween the points P and Q -1 µ F, calculate capacitance
of each capacitor. [Ans. 4 µ F]

57. What is the capacitance of arrangement of 4 plates 62. Calculate the equivalent capacitance between the
each of area A at a distance d in air in Fig. points A and B in the combination shown in Fig.
[Ans. 2 ϵ0 A/d] [Ans. 13.44 µF]

58. What is the capacitance of arrangement of 4 plates of


area A at distance d in air in Fig. [Ans. 3 ϵ0 A/d]

19 - Nikhil Kamboj — 8193989452 #edugramddun


Electric Potential and Capacitance

63. Calculate the equivalent capacitance between the


points A and B in the combination shown in Fig.
[Ans. 15 µF ]

68. Two capacitors of capacitance C1 and C2 are con-


nected (i) in series, (ii) in parallel. Derive expressions
64. The outer cylinders of two cylindrical capacitors of
for the equivalent capacitance for each of these com-
capacitance 2.2 µ F each, are kept in contact and the 1 1 1
inner cylinders are connected through a wire. A bat- bination. [Ans. )i) = + ; (ii)Cp = C1 + C2 ]
Cs C1 C2
tery of e.m.f. 10 V is connected as shown in Fig. Find
the total charge supplied by the battery to the inner 69. How will you arrange Seven capacitors each of ca-
cylinders. [Ans. 44 µ C] pacitance 2 µF obtain a capacitance of (10/11) µF.
[Ans. 5 in parallel, 2 in series]
70. A parallel plate capacitor is made by stacking n
equally spaced plates connected alternately. If the ca-
pacitance between any two plates is C, then find the
resulting capacitance. [Ans. (n-1)C]

m
ENERGY STORED IN CAPACITORS

71. A 800 pF capacitor is charged by a 100 V battery.


65. If C1 = 3 pF and C2 = 2 pF, calculate the equiva-
lent capacitance of the network shown in Fig. between
points A and B. [Ans. 1 pF]
ra After sometime, the battery is disconnected. The ca-
pacitor is then connected to another 800 pF 10 ca-
pacitor. What is the electrostatic energy stored ?
[Ans. 2 x 10−6 J]
G
72. Net capacitance of three identical capacitors in series
is 1 µF. What will be their net capacitance in par-
allel? Find the ratio of energy stored in two con-
figurations if they are connected to the same source.
u

[Ans. 9 µF; 1:9]


73. Fig. shows a network of five capacitors connected to a
Ed

100 V supply. Calculate the total charge and energy


stored in the network. [Ans. 4 x 10−4 C ; 0.02 J]

66. Find the equivalent capacitance of the combination of


capacitors between the points A and B as shown in
Fig. Also calculate the total charge that flows in the
circuit, when a 100 V battery is connected between
the points A and B. [Ans. 20 µF; 2 x 10 C]

74. In Fig. the energy stored in C4 is 27 J. Calculate the


total energy stored in the system. [Ans. 594.0 J]

67. Two capacitors C1 and C2 are connected to a battery


of 6 V as shown in Fig. Find the charge on each ca-
pacitor. [Ans. 24 µ C; 36 µ C]

20 - Nikhil Kamboj — 8193989452 #edugramddun


Electric Potential and Capacitance

75. Find the total energy stored in capacitors in the net- the capacitor. (i) before (ii) after the introduction
work shown In Fig. [Ans. 3.6 x 10−5 J] of plates (iii) what would be the p.d. if a dieletric
76. Net capacitance of three identical capacitors in series slab (K = 2) were introduced in place of metal plate.
is 1 µF. What will be their net capacitance if con- [Ans. 1500 V, 1200 V, 1350 V]
nected in parallel ? Find the ratio of energy stored in 85. The two circular plates of a parallel plate capacitor are
the two configurations. if they arc both connected to 8 cm in diameter each and 15 mm apart. An ebonite
the same source. [Ans. 9 µF; 1 :9] plate 0.5 cm thick is introduced between the plates.
77. Three identical capacitors C1 , C2 and C3 of capac- Calculate its capacity. If the plate were of copper,
itance 6 µ F each are connected to a 12 V bat- what would be the new capacity ? Take K = 2.5.
tery as shown in Fig. Find (i) charge on each ca- [Ans. 3.71 x 10−12 F ; 4.45 x 10−12 ]
pacitor (ii) equivalent capacitance of the network 86. When a slab of insulating material 4 mm thick is intro-
(iii) energy stored in the network of capacitors: duced between the plates of a parallel plate capacitor,
[Ans. q1 = q2 = 36µC, q3 = 72µC; 9µF ; E1 = E2 = it is found that the distance between the plates has
108µJ, E3 = 432µJ] to be increased by 3.2 mm to restore the capacity to
78. A 20 µ F capacitor is charged by a 30 V d.c. sup- its original value. Calculate dielectric constant of the
ply and then connected across an uncharged 50 µ material. [Ans. 5]
F capacitor. Calculate (i) the final potential diff 87. The area of parallel plates of an air capacitor is
across the combination (ii) initial and final energies. 0.2 m2 and the distance between them is 0.01 m.
How will you account for the difference in energies ? The potential difference between the plates is 3000
[Ans. 8.57 V ; 9 x 10−3 J, 2.57 x 10−3 J] V. When a 0.01 m thick sheet of an insulating

m
79. Two parallel plate capacitors X and Y have the same material is placed between the plates, the poten-
area of plates and same separation between them. X tial difference decreases to 1000 volt. Determine
has air between the plates and Y contains a dielectric (i) capacitance of capacitor before placing the sheet
medium of ϵr = 4. Calculate (i) capacitance of X and (ii) charge on each plate (iii) dielectric constant
Y if equivalent capacitance of combination is 4 µ F.
(ii) Pot. diff. between the plates of X and Y (iii)
What is the ratio of electrostatic energy stored in X
and Y ? [Ans. 5 µF, 20 µF, 9.6 V, 2.4 V; 4 : 1]
ra of material (iv) capacitance after placing the in-
sulator (v) absolute permittivity of the dielectric.
[Ans. 1.77 x 10−10 F; 5.31 x 10−7 C ; 3; 5.31 x 10−10
F ; 2.66 x 10−11 F.m]
G
88. A parallel plate capacitor is to be designed with volt-
80. In the circuit shown in Fig. the energy stored in both
age rating of 1 kV using a material of dielectric con-
capacitors is U1 . If switch S is opened and a dielectric
stant 3 and dielectric strength of 107 V/m. For safety,
slab of constant 5 is put in free spaces of the capac-
we like the field never to exceed 10% of dielectric
itors, the energy stored is found to be U2 . Calculate
u
strength. What minimum area of plate is required
U1 /U2 [Ans. 5/13]
to have a capacitance of 50 pF. [Ans. 19 cm2 ]
81. Derive an expression for energy stored in a capacitor
89. There is an air-filled 1 pF parallel plate capacitor.
Ed

of capacitor of capacitance C, charged to a potential


1 When the plate separation is doubled and the space
V. [Ans. CV 2 ] is filled with wax, the capacitance increases to 2 pF.
2
Find the dielectric constant of wax. [Ans. 4]
82. 2 Capacitors C1 and C2 are connected (i) in series,
(ii) in parallel. Show that the energy stored is same in 90. A parallel plate capacitor is charged. What will hap-
both the case. pens to charge and potential difference if the plates
EFFECT OF INTRODUCING SLAB IN are pulled apart. [Ans. both remain same]
CAPACITORS 91. A parallel plate capacitor with air as medium between
the plates has a capacitance of 10 µF. The area of the
83. The two plates of a parallel plate capacitor are 4 mm capacitor is divided into two equal halves and filled
apart. A slab of dielectric constant 3 and thickness with two media having dielectric constants K1 = 2
3 mm is introduced between the plates with the faces and K2 = 4. What will be the capacitance of the sys-
parallel to them. The distance between the plates is so tem now. [Ans. 30 µF ]
adjusted that the capacitance of the capacitor becomes
2 92. Capacitance of a capacitor becomes 7/6 times its orig-
rd of its original value. What is the new distance be- inal value if a dielectric slab of thickness t = (2/3)d
3
tween the plates ? [Ans. 8 mm] is introduced between plates. d is the separation be-
84. An electric field E0 = 3 × 104 V /m is established be- tween plates. Find the dielectric constant of dielectric
tween the plates 0.05 m apart, of a parallel plate ca- slab. [Ans. 7/11]
pacitor. After removing the charging battery, an un- 93. An air capacitor C connected to a battery of emf
charged metal plate of thickness r = 0.01 m is in- V acquires a charge q and energy E. The capac-
serted between capacitor plates. Find the pd across itor C is disconnected from the battery and a di-

21 - Nikhil Kamboj — 8193989452 #edugramddun


Electric Potential and Capacitance

electric slab is placed between the plates. How will system of two charge q1 and q2 placed ’r’ distance a
the following quantities will change: (i) Charge (ii) part.
capacitance (iii) Electric potential (iv) electric field.
102. Derive expression for workdone in rotating an electric
[Ans. q remains unchanged, C increases, V and E
dipole in an electric field. OR
decreases]
Derive Expression for potential energy of the dipole in
DESCRIPTIVE QUESTIONS electric field. [Ans. W = U = −⃗ ⃗
p.E]
103. Derive expression for potential due to spherically
94. Differentiate Electric Potential, Electric Potential Dif- symmetric distribution of charges at (i) at ex-
ference, and Electric Potential Energy. ternal point at distance r from centre, (ii) at
95. What is Electric Potential. Derive an expression for surface of sphere (radius R) (iii) at point in-
Electric potential due to point charge ′ q ′ at distance side the sphere. Draw the graph for the same.
1 q 1 q
kq [Ans. (i) ; (ii) ; (iii)zero]
r from it. [Ans. V =
′ ′
] 4πϵ0 r 2 4πϵ0 R2
r
96. Derive Expression for Potential due to Electric Dipole 104. How much work is done in taking a unit positive charge
at a point on (i) axial line (ii) equatorial line. from one point to another over an equipotential sur-
2kp face. [Ans. Zero]
[Ans. (i) V = 2 , (ii) Zero]
r 105. Electric charge is distributed uniformly on the surface
97. Derive Expression for Potential due to Electric Dipole of a spherical rubber balloon. Show how the value of
1 p cos θ electric intensity and potential vary (i) on the surface
at any point. [Ans. V = . ]
4πϵ0 r2 (ii), inside and (iii) outside ?

m
98. What are equipotential surfaces. Establish relation 106. A capacitor has some dielectric between its plates,
dV
between electric field and potential. [Ans. E = ] and the capacitor is connected to a dc source. The
dr
battery is now disconnected and then the dielectric
99. Why two equipotential surfaces cannot intersect?

surface?
101. Derive expression for Electric Potential Energy of a
ra
100. Why electric field is perpendicular to the equipotential
is removed, then how will Electric field will change.
[Ans. increases]
107. What will be the capacitance of earth, viewed as a
spherical conductor of radius 6408 km. [Ans. 712 µF]
u G
Ed

22 - Nikhil Kamboj — 8193989452 #edugramddun


Current Electricity
Important Formulae and Notes

I r
▶ Drift velocity, vd = (b) Total internal resistance of cells =
neA n
r
N n0 d (c) Total resistance in circuit = R +
▶ Free electron density, n = n
A E
I (d) Current in the circuit, i =
▶ Current density, J = R + nr
A ▶ Temperature coefficient of resistance
▶ EMF of cell E = V + Ir ρ1 − ρ0 R1 − R0
α= =
r = internal resistance, V = terminal potential differ- ρ0 t R0 t
ence where ρ0 and R0 are respectively the specific resistance
and resistance at 00 C.
▶ Potential difference across resistance R V = IR =
ER ▶ Combination of resistances
R+r (a) In series : Rs = R1 + R2 + R3 + ..... + Rn
▶ Combination of cells 1 1 1 1 1
(b) In Parallel : = + + + ..... +
▶ Cells in Series Rp R1 R2 R3 Rn
RA
(a) Total emf = nE ▶ Specific resistance (resistivity), ρ =

m
l
(b) Total internal resistance of cells 1
= nr ▶ Conductivity, σ =
ρ
(c) Total resistance in circuit = R + nr
▶ Wheatstone network
(d) Current in the circuit, i =

▶ Cells in Parallel
(a) Total emf = E
nE
R + nr
ra (a) Balancing Condition,
R1
R2
=
R3
R4
(b) Sensitivity of Wheatstone’s network is maximum,
if R1 = R2 = R3 = R4
G
Multiple Choice Questions

1. In the series combination of two or more than two re-


u
sistances
Ed

a) the current through each resistance is same.


b) the voltage through each resistance is same
c) neither current nor voltage through each resis-
tance is same
d) both current and voltage through each resistance
are same

4. In parallel combination of n cells, we obtain


a) more voltage b) more current
2. Maximum resistance that can be obtained by arrang- c) less voltage d) less current
ing 3 resistors 5 Ω, 4.5 Ω and 3 Ω is
5. If n cells each of emf ’e’ and internal resistance ’r’ are
a) 12.5 Ω b) 13.5 Ω c) 14.5 Ω d) 16.5 Ω connected in parallel, then the total emf and internal
resistance will be
r n r
a) ϵ, b) ϵ, c) nϵ, d) nϵ, nr
n r n
3. A cell having an emf E and internal resistance r is con- 6. The drift velocity of electron in a conductor is nearly:
nected across a variable external resistance R. As the
resistance R is increased, the plot of potential differ- a) 10−8 m/s b) 10−4 m/s
ence V across R is given by c) 103 m/s d) 108 m/s
Current Electricity

7. Given three equal resistors, how many different com- a) resistance will be doubled and specific resistance
binations(taken all together) of these three resistances will be halved
can be made? b) resistance will be halved and specific resistance
a) 4 b) 6 c) 8 d) 9 will remain uncharged
c) resistance will be halved and the specific resis-
8. If a certain piece of copper is to be shaped into a con- tance will be doubled
ductor of minimum resistance, its length (L) and cross- d) resistance and specific resistance will both re-
sectional area (a) shall respectively be main uncharged
a) L, 2A b) L/2, c) 2L, 2A d) 2l, A/2 17. n resistances, each of r Ω, are connected in parallel
2A gives an equivalent resistance of R Ω. If these re-
sistances were , connected in series, the combination
9. Specific resistance of all metals is mostly affected by would have a resistance in Ω is equal to
a) temperature b) pressure a) n2 R b) R/n2 c) R/n d) nR
c) magnetic field d) volume
18. Ohm’s law is valid when the temperature of conductor
10. The specific resistance of a rod of copper as compared is :
to that of thin wire of copper is a) very low b) very high
a) less b) more c) varying d) constant
c) same d) can’t say
19. Copper wire is used as connecting wire because :

m
11. A wire of resistance 3 Ω is cut into three equal pieces, a) copper has high electrical resistivity
which are then joined to form a triangle. The equiva- b) copper has low electrical resistivity
lent resistance between any corners of the triangle is c) copper has low electrical conductivity
d) copper has high value of elasticity
a) 2/3 Ω b) 3/2 Ω c) 1/2 Ω d) 1/3 Ω

12. Which of the following I-V graph represents ohmic


conductors
ra 20. The heat sensitive device whose resistivity changes
very rapidly with change of temperature is called a
:
G
a) resistor b) super-conductor
c) thermocouple d) thermistor

21. The specific resistance of a conductor increases with :


u

a) increase in temperature
b) increase in cross-sectional area
c) decrease in length
Ed

d) decrease in cross-sectional area

22. Two conductors of equal length and radii the ratio of


2 : 3 are; connected in parallel to the source of elec-
tricity. The ratio of the velocity of electrons in the
13. The length of a conductor is halved by cutting. Its conductor be :
resistance will be a) 2 : 3 b) 4 : 9 c) 1 : 1 d) 3 : 2
a) halved b) doubled
c) unchanged d) quadrupled 23. The charge flowing in a conductor varies with time as
: q = αt + 12βt2 + 16γt3
Where α, β, γ are positive constants. Then the initial
14. Siemen is the unit of:
current (I) is given by the condition :
a) resistance b) conductance
a) I = α b) I = α2
c) specific conductance d) None of these
c) I = α − 1 d) None of these
15. A 5 A fuse wire can with stand a maximum power of 24. The smallest resistance that can be obtained by the
1 W in circuit. The resistance of the fuse wire is: combination of n resistors each resistance r is :
a) 0.2 Ω b) 5 Ω c) 0.4 Ω d) 0.04 Ω a) r/n b) n/r c) nr d) n2r

16. The length and radius of an electric resistance of a 25. Which of the following has a negative temperature co-
certain wire are doubled simultaneously, then the: efficient of resistance ?

24 - Nikhil Kamboj — 8193989452 #edugramddun


Current Electricity

a) Tungsten b) Carbon c) increases, thermal velocity of the electron


c) Nichrome d) Platinum decreases
d) decreases, thermal velocity of the electron
26. A wire P has half the diameter and half the length decreases
as compared to other wire Q of similar material. The
ratio of resistances of P to that of Q is: 34. The electric field intensity E, current density J and
specific resistance k are related to each other through
a) 4 : 1 b) 8 : 1 c) 2 : 1 d) 1 : 2 the relation
a) E = J/k b) E = J k
27. In a current carrying conductor the net charge is
c) E = k/J d) k = J E
a) 1.6 x 10−19 C b) 6.25 x 10−18 C
c) zero d) infinite 35. The relaxation time in conductors
a) increases with the increases of temperature
28. The current which is assumed to be flowing in a circuit b) decreases with the increases of temperature
from positive terminal to negative, is called c) it does not depends on temperature
d) all of sudden changes at 400 K
a) direct current
b) pulsating current 36. In conductor when electrons move between two colli-
c) conventional current sions, their paths are .......A.... when external fields are
d) alternating current absent and .......B...... when external filed is present.
Here, A and B refer to
29. When no current is passed through a conductor,

m
a) straight lines, straight lines
a) the free electrons do not move b) straight lines, curved lines
b) the average speed of a free electron over a large c) curved lines, straight lines
period of time is not zero d) curved lines, curved lines

period of time is zero


d) the average of the velocities of all the free elec-
trons at an instant is non zero
ra
c) the average velocity of a free electron over a large
37. The effective resistance between C and D in given cir-
cuit is
G
30. Drift velocity of electrons is due to
a) motion of conduction electrons due to random
collisions.
u
b) motion of conduction electrons due to electric
field a) R b) 3R c) 2R/3 d) R/3
c) repulsion to the conduction electrons due to in-
Ed

ner electrons of ions. 38. Two or more resistors are said to be in .......A..... if
d) collision of conduction electrons with each other. one end of all resistors is joined together and similarly
the other ends joined together, Here, A refers to
31. In the absence of an electric field, the mean velocity of a) series b) parallel
free electrons in a conductor at absolute temperature c) both a & b d) None
(T); 0 Kelvin is
39. At temperature 0 Kelvin, the germanium behaves as
a) zero b) inversely prop. to T a
c) proportional to T d) proportional to T2 a) conductor b) insulator
c) super-conductor d) ferromagnetic
32. The current density (number of free electrons per m3 )
in metallic conductor is of the order of 40. A strip of copper and another of germanium are cooled
from room temperature to 80 K. The resistance of
a) 10 −22
b) 10 4
c) 10 9
d) 10 28
a) each of these increases
33. A metal wire is subjected to a constant potential dif- b) each of these decreases
ference. When the temperature of the metal wire in- c) copper strip increases and that of germanium
creases, the drift velocity of the electron in it decreases
d) copper strip decreases and that of germanium
a) increases, thermal velocity of the electron increases
increases
b) decreases, thermal velocity of the electron 41. The electric resistance of a certain wire of iron is R. If
increases its length and radius are both doubled, then

25 - Nikhil Kamboj — 8193989452 #edugramddun


Current Electricity

a) the resistance and the specific resistance, will


both remain unchanged
b) the resistance will be doubled and the specific
resistance will be halved
c) the resistance will be halved and the specific re-
sistance will remain unchanged
d) the resistance will be halved and the specific re-
sistance will be doubled

42. Nichrome or Manganin is widely used in wire bound


standard resistors because of their
48. The resistance of the coil of an ammeter is R. The
a) temperature independent resistivity shunt required to increase its range n-fold should have
b) very weak temperature dependent resistivity. a resistance
c) strong dependence of resistivity with
temperature. a) R/n b) R/(n- c) R/(n+1) d) nR
d) mechanical strength. 1)
X
43. With increase in temperature the conductivity of 49. Kirchhoff’s first law, i.e., i = 0 at a junction, deals
with the conservation of
a) metals increases and of semiconductor decreases.

m
b) semiconductors increases and metals decreases. a) charge b) energy
c) in both metals and semiconductors increases c) momentum d) angular momentum
d) in both metal and semiconductor decreases. X X
50. The Kirchhoff’s second law iR = E, where the
44. The resistance of a metal increases with increasing
temperature because
a) the collisions of the conducting electrons with
the electrons increase
ra symbols have their usual meanings, is based on
a)
b)
c)
conservation
conservation
conservation
of
of
of
momentum
charge
potential
G
b) the collisions of the conducting electrons with d) conservation of energy
the lattice consisting of the ions of the metal
increase 51. In potentiometer a balance point is obtained, when
c) the number of conduction electrons decreases a) the e.m.f. of the battery becomes equal to the
u

d) the number of conduction electrons increases e.m.f of the experimental cell


b) the p.d. of the wire between the +ve end of bat-
Ed

45. To minimise the power loss in the transmission cables tery to jockey becomes equal to the e.m.f. of the
connecting the power stations to homes and factories, experimental cell
the transmission cables carry current c) the p.d. of the wire between +ve point of cell
and jockey becomes equal to the e.m.f. of the
a) at a very low voltage. battery
b) at a very high voltage
d) the p.d. across the potentiometer wire becomes
c) at 220 volt
equal to the e.m.f. of the battery
d) neither at a very high voltage nor at a very low
voltage.
52. For measuring voltage of any circuit, potentiometer is
preferred to voltmeter because
46. To draw a maximum current from a combination of
a) the potentiometer is cheap and easy to handle.
cells, how should the cells be grouped?
b) calibration in the voltmeter is sometimes wrong
a) Parallel .
b) Series c) the potentiometer almost draws no current dur-
c) Mixed grouping ing measurement.
d) Depends upon the relative values of internal and d) range of the voltmeter is not as wide as that of
external resistances. the potentiometer.

47. Find equivalent resistance of the following between A 53. Two cells of emf’s approximately 5 V and 10 V are
and B to be accurately compared using a potentiometer of
length 400 cm.

26 - Nikhil Kamboj — 8193989452 #edugramddun


Current Electricity

a) The battery that runs the potentiometer should a) a high resistance in parallel
have voltage of 8 V. b) a high resistance in series
b) The battery of potentiometer can have a volt- c) a low resistance in parallel
age of 15 V and R adjusted so that the potential d) a low resistance in series
drop across the wire slightly exceeds 10 V.
c) The first portion of 50 cm of wire itself should 60. How much electric energy is consumed by a 100 W
have a potential drop of 10 V. lamp used for 6 hours everyday for 30 days?
d) Potentiometer is usually used for comparing re- a) 18 kJ b) 18 kWh c) 1.8 J d) None
sistances and not voltages.
61. Two wires of copper are of the same length but have
54. An electric heater is connected to the voltage supply. different diameters. When they are connected in se-
After few seconds, current gets its steady value then ries across a battery, the heat generated is H1 When
its initial current will be connected in parallel across the same battery, the heat
generated during the same time is H2. Then :
a) equal to its steady current
b) slightly higher than its steady current a) H1 = H2 b) H1 < H2
c) slightly less than its steady current c) H1 > H2 d) H1 > H2
d) zero
62. Kirchhoffs first and second laws for electrical circuits
are consequences of:
55. In a Wheatstone bridge if the battery and galvanome-
ter are interchanged then the deflection in galvanome- a) conservation of energy
ter will b) conservation of electrical charge and energy

m
respectively
a) change in previous direction
b) not change c) conservation of electric charge
c) change in opposite direction d) neither conservation of energy nor electric charge
d) none of these.

56. When a metal conductor connected to left gap of a


meter bridge is heated, the balancing point
ra 63. When three identical bulbs of 60 W, 200 V rating are
connected in series to a 200 V supply, the power drawn
by them will be:
a) 20 W b) 60 W c) 180 W d) 10 W
a) shifts towards right b) shifts towards left
G
c) remains unchanged d) remains at zero 64. Why is the Wheatstone bridge more accurate than
other methods of measuring resistances: ‘
57. AB is a wire of potentiometer with the increase in the
a) It is a null method
value of resistance R, the shift in the balance point J
u
b) It is based on Kirchhoffs laws
will be
c) It has four resistances
d) It does not involve ohm’s law
Ed

65. In an experiment with potentiometer, null point with


a cell is found at 240 cm. When the cell is shunted
with a resistance 2 Ω, the null point becomes 120 cm
a) towards B internal resistance of cell is :
b) towards A a) 4 Ω b) 2 Ω c) 1 Ω d) 1/2 Ω
c) remains constant
d) first towards B then back towards A 66. An electric heating element consumes 500 W, when
connected to a 100 V line. If the line voltage becomes
58. In a potentiometer of 10 wires, the balance point is 150 V, the power consumed Will be:
obtained on the 7th wire. To shift the balance point a) 500 W b) 750 W c) 1000 W d) 1125 W
to 9th wire, we should
67. In following figure shows currents in a part of electri-
a) decrease resistance in the main circuit
cal circuit, then the value of I(in ampere) is given by
b) increase resistance in the main circuit.
:
c) decrease resistance in series with the cell whose
emf is to be measured.
d) increase resistance in series with the cell whose
emf is to be determined.

59. A galvanometer acting as a volt meter will have with


its coil.

27 - Nikhil Kamboj — 8193989452 #edugramddun


Current Electricity

a) 0.3 A b) 0.5 A a) 25 W
c) 1.3 A d) None of these b) 100 W
c) both bulbs have equal resistance
68. A uniform wire connected across a supply produces d) resistance of bulbs can not be compared
heat H per second. If wire is cut into three equal parts
and all the parts are connected in parallel across the
same supply, the heat produced per second will be : 72. Potentiometer measures the potential difference more
accurately than a voltmeter because:
a) H/9 b) 9 H c) 3 H d) H/3
a) It has a wire of high resistance
69. In India electricity is supplied for domestic use at 220 b) It has a wire of low resistance
V. It is supplied at 110 V in U.S.A. If the resistance c) It does not draw current from external circuit
of a 60 W bulb use in India is R. the resistance of a d) It draws heavy current from external circuit
60 W bulb for use in USA will be:
a) 2R b) R c) R/2 d) R/4 73. In a Wheatstone bridge, the resistance each arm is 10
70. In the following figure represents a balanced Wheat- Ω. If the resistance galvanometers is also 10 Ω, then
stone bridge circuit. What is the value of X ? effective resistance of the bridge will be :
a) 40 Ω b) 20 Ω c) 10 Ω d) 5 Ω

74. When the wires are connected in parallel, the heat pro-
duced in the thinner wire is H1 and that in the thicker

m
wire is H2 Then:
a) H1 = H2 b) H1 < H2
c) H1 > H2 d) H1 >= H2

a) 15 Ω b) 20 Ω c) 25 Ω d) 30 Ω

71. Two bulbs 25 W, 220 V and 100 W, 220 V are given.


ra 75. Two equal resistors are connected in series across a
battery and consume a power of P. If these are con-
nected in parallel, then the power consumed will be:
G
Which has higher resistance? a) 2p b) 4p c) p/4 d) p

Practice Questions and Numerical


u
ELECTRIC CURRENT [Ans. (a) 5.76 x 104 C (b) 19.2 A]
6. The charge flowing through a conductor varies with
1. What do you understand by electric cur- time as q = 8t − 3t2 + 5t3
Ed

rent? Define SI unit and mention the direc- Find (i) the initial current (ii) time after which
tion of electric current in the circuit. Also ex- the current reaches a maximum value (iii)
plain if current is a scalar or vector quantity. the maximum or minimum value of current.
[Ans. Flow of charge; Ampere; High V to Low V; [Ans. (i) 8 A (ii) 0.2 s (iii) 7.4 A]
Vector] 7. Fig. shows a plot of current I (in ampere) through
2. In Bohr model of hydrogen atom, the electron revolves the cross-section of the wire over a time interval
around the nucleus in a circular orbit of radius 5.1 x of 14 s. Find the amount of charge that passes
10−11 m at a frequency of 6.8 x 1015 revolutions per through the wire (i) during 10 s (ii) during 14 s.
second. Find the equivalent current at any point on [Ans. (i) 40 C (ii) 50 C]
the orbit of the electron. [Ans. 1.088 x 10−3 A]
3. A solution of sodium chloride discharges 6.0 x 1016
Na+ ions and 4.5 x 1016 Cl−1 ions in 2 seconds.
What is the current passing through the solution ?
[Ans. 8.4 mA]
4. In hydrogen atom, the electron moves in an orbit of
radius 5.0 x 10−11 m with a speed of 2.2 x 106 m/s.
Find the equivalent current. [Ans. 1.12 mA]
5. If 0.6 mole of eletrons flow through a wire in 50 min-
utes. What is (a) the total charge that passes through 8. An electric current of 20 µA appeals in a discharge
the wire, and (b) the magnitude of the current ? tube. If the discharge current is due to flow of equal

28 - Nikhil Kamboj — 8193989452 #edugramddun


Current Electricity

number of electrons and protons, then how many elec- 20. State Ohm’s law and hence draw V-I graph form ohmic
trons flow across a cross-section of the tube in 2 min- and non-ohmic device. Also give two examples each of
utes. [Ans. 7.5 x 1015 ] ohmic and non-ohmic device. [Ans. V ∝ I]
DRIFT VELOCITY AND ELECTRON 21. What do you understand by Resistance of conductor?
MOBILITY What is its cause? Explain the factors on which resis-
tance of conductor depends.
9. Define the term ‘drift velocity’ of charge carriers in a
conductor and write its relationship with the current 22. Define Specific resistance and express it in terms of
flowing through it. [Ans. I = neAvd ] mass, charge, number density and relaxation time.
ρl
10. Establish relation between current and drift velocity. [Ans. R = ]
A
[Ans. I = neavd ]
23. A wire is stretched to increase its length by 5%. Find
11. What is average relaxation time. How does the aver- the percentage change in its resistance. [Ans. 10.25%]
age relaxation time in conductors varies with temper-
ature. 24. A wire of 15 ohms resistance is gradually stretched
to double its original length. It is then cut into two
12. Fill in the blanks equal parts. These parts are then connected in parallel
In conductor when electrons move between two colli- across a 3 Volt battery. Find the current drawn from
sions, their paths are ...................... when external the battery. [Ans. 0.2A]
fields are absent and ...................... when external
filed is present. [Ans. straight lines, curved lines] 25. The length and radius of an wire are dou-
bled simultaneously. How will the Resis-
13. Two wires X and Y have the same resistivity but their
tance and resistivity of the wire will change.

m
cross sectional areas are in the ratio 2:3 and length in
[Ans. R =¿ halved and ρ =¿ uncharged]
ratio 1:2. They are first connected in series and then
in parallel to a dc source. Find out the ratio of drift 26. A wire P has half the diameter and half the length as
speeds of the electrons in the two wires for the two compared to other wire Q of similar material. What
cases. [Ans. 3:2, 2:1]
14. A metal wire is subjected to a constant potential dif-
ference. When the temperature of the metal wire in-
creases, how will the drift velocity and the thermal
ra is the ratio of resistances of P to that of Q. [Ans. 2:1]
27. A wire connected to a bulb does not glow, whereas the
filament of the bulb glows when same current flows
through them. Why? [Ans. ans]
G
velocity of the electron in it will change. Explain. 28. An arc lamp operates at 80 V, 10 A. Suggest a method
15. A conductor of length L is connected to a d.c. source to use it with a 240 V d.c. source. Calculate the value
of emf e. The drift velocity of electron is vd . If the of the electric component required for this purpose.
length of conductor is tripled by stretched it, keeping [Ans. Resistor of value 16 Ω]
u
e constant, find its drift velocity. [Ans. vd /3]
29. A wire 50 cm long and 0.12 mm diameter has a resis-
16. The number of free electrons per 5 cm of ordinary tance of 4.0 Ω. Find the resistance of another wire of
Ed

copper wire is 2 x 1021 . The average drift speed of the same material whose length is 1.5 m and diameter
electrons is 0.25 mm/s. What is the current flowing ? is 0.15 mm. [Ans. 1.92 Ω]
[Ans. 1.6 A]
30. A uniform wire of length I and radius r has resistance
17. What is the drift velocity of electrons in a silver wire of 100 Ω. It is recast into a thin wire of (i) length 21
length 1 m, having cross-sectional area 3.14 x 10−6 m2 (ii) radius r/2. Calculate the resistance of new wire
and carrying a current of 10 A. Given atomic weight in each case. [Ans. (i) 400 Ω (ii) 1600 Ω]
of silver = 108, density of silver 10.5 x 103 kg/m3 .
[Ans. 3.4 x 10−4 m/s] 31. There are two wires of copper and iron of the same
length but different radii. When equal potential dif-
18. Protons in cosmic-rays strike the earth’s upper atmo- ference is applied between the ends of each wire, the
sphere at a rate, average over the earth’s surface of same current flows in them. What the ratio of their
10 protons m−2 s−1 . What does the total current the radii. Specific resistances of copper and iron are 1.6 x
earth receive from beyond its atmosphere in the form 10−8 Ωm and 1.0 x 10−7 Ωm respectively. [Ans. 2/5]
of incident cosmic ray protons ? The earth’s radius is
6.4 x 106 m. [Ans. 0.82 mA] 32. A wire of mass 10 g, radius 1 mm is compressed to its
length by 10%. Calculate the percentage change in its
19. A uniform copper wire of length 1 m and cross sec-
resistance. [Ans. 19%]
tional area 5 x 10−7 m2 carries a current of 1 A. As-
suming that there are 8 x 1028 free electrons per m3 33. A wire is stretched to increase its length by 5%. Calcu-
in copper, how long will an electron take to drift from late percentage change in its resistance. [Ans. 10.25%]
one end of the wire to the other. [Ans. 6.4 x 103 s] 34. A hollow cylinder of length l and of radii a and b
OHM’S LAW, RESISTANCE, RESISTIVITY is filled with a material of resistivity ρ and is con-
nected to a battery of emf ϵ through an ammeter,

29 - Nikhil Kamboj — 8193989452 #edugramddun


Current Electricity

as shown in Fig. Find the current through ammeter.


E2πl
[Ans. ]
ρloge (b/a)

43. A wire carries a current of 2.0 A, when a potential


difference of 3.0 V is applied across it. What is its
conductance ? If the wire is of length 3 m and area of
cross-section 5.4 x 10−6 m2 , calculate its conductivity.
[Ans. 0.67 S ; 3.7 x 105 S/m]
44. Calculate the radius of the wire of conductance 10 Ω−1
and length 10 cm whose electrical conductivity is 105
S/m [Ans. 1.78 mm]

35. Calculate the mass of copper required to draw a wire VARIATION OF RESISTANCE WITH
TEMPERATURE
5 km long having resistance of 5 Ω. The density of
copper is 8.9 x 103 kg m−3 and resistivity of copper is 45. Define Temperature coefficient of resistivity. Write its
1.7 x 10−8 [Ans. 756.5 kg] SI unit. Draw a graph showing the variation of resis-
36. Two wires A and B of the same material have their tivity with temperature for (i) conductor (ii) semicon-
lengths in the ratio 5 : 3 and diameter in the ratio ductor (iii) insulator.
2 : 3. If the resistance of wire A is 15 Ω, find the 46. The resistance of a tungsten filament at 160°C is 132

m
resistance of wire B. [Ans. 4 Ω] Ω. What will be its resistance at 400°C ? The tem-
37. A wire of 15 Ω resistance is gradually stretched to perature coefficient of resistance of tungsten is 4.5 x
double its original length. It is then cut into two 10−3 . [Ans. 215 Ω]
equal parts. These parts are then connected in paral- 47. The temperature coefficient of a resistance wire is
lel across a 3.0 volt battery. Find the current drawn
from the battery. [Ans. 0.2 A]
38. Find the time of relaxation between collision and mean
free path of electrons in copper at room temperature.
ra 0.00125°C−1 . At 300 K its resistance is 1 Ω. At
what temperature the resistance of wire will be 2 Ω?
[Ans. 1127 K]
48. A metal wire of diameter 2 mm and length 50 cm
G
Given resistivity of copper = 1.7 x 10−8 Ωm; number has a resistance 0.31 Ω at 25°C and 0.51 Ω at 125°C.
density of electron in copper = 8.5 x 1028 m−3 . Charge Find (i) the temperature coefficient of resistance (ii)
on electron = 1.6 x 10−19 C, mass of electron = 9.1 x resistance at 0°C and (iii) resistivity at 0°C and 25°C.
10−31 kg. Drift velocity of free electron= 1.6 x 10−4 [Ans. (i) 6.45 x 10−3 °C−1 (ii) 0.267 Ω, (iii) 1.68 x
u
m/s. [Ans. 2.5 x 10−14 s ; 4.0 x 10−18 m] 10−6 Ωm, 1.948 x 10−6 m]
39. The length of a conductor is halved by cut- 49. (a) At what temperature would the resistance of a cop-
Ed

ting. How will Resistance of conductor be changed. per conductor be double of its value at 0°C ?
[Ans. become half] (b) Does this same temperature hold for all copper con-
40. A 5 A fuse wire can with stand a maximum power of ductors, regardless of the size and shape ? Tempera-
1 W in circuit. What should be the resistance of the ture coefficient of resistance of copper is 4.0 x 10−3 /°C.
fuse wire ? [Ans. 0.04Ω] [Ans. (a) 250°C (b) yes]
CURRENT DENSITY, CONDUCTANCE AND 50. A strip of copper and another of germanium are
CONDUCTIVITY cooled from room temperature to 80 K. How
will the resistance of both will change. Explain.
41. A rheostat has 1000 turns of a wire of radius 0.4 [Ans. copper strip decreases and that of germanium
mm, having resistivity 49 x 10−8 m. The diameter increases]
of each turn is 4 cm. What are the maximum val- 51. How does the resistivity and conductivity of metals
ues of conductance and conductivity of rheostat wire. and semiconductors changes with increase in temper-
[Ans. 8.2 x 10−3 S; 2.04 x 106 S/m] ature.
42. Current flows through a constriction conductor as COMBINATION OF RESISTANCES IN
shown in Fig. The diameter and current density to SERIES AND PARALLEL
the left of constriction are 2.0 mm and 1.3 x 106
Am−2 . (i) How much current flows through the con- 52. Given three equal resistors, how many different com-
striction ? (ii) If the current density is tripled as it binations(taken all together) of these three resistances
emerges from the right side of the constriction, what can be made?
is the diameter of the right hand side of constriction ? 53. A wire of resistance 3Ω is cut into three equal pieces,
[Ans. (i) 4.09 A ; (ii) 1.15 mm] which are then joined to form a triangle. Find the

30 - Nikhil Kamboj — 8193989452 #edugramddun


Current Electricity

equivalent resistance between any corners of the trian- K2 is open (ii) I/2 when both keys K1 , and K2 are
gle. [Ans. 2/3 Ω] closed. Find the expression for the resistance of X in
RS
54. A wire of resistance 8R is bent in the form of a circle. terms of the resistances of R and S. [Ans. ]
What is the effective resistance between the ends of a R−S
diameter? [Ans. 2R]
55. What is the maximum resistance that can be obtained
by arranging 3 resistors 5 Q, 4.5 Q and 3 Q. [Ans. 12.5]
56. n resistances, each of rΩ, are connected in parallel
gives an equivalent resistance of RΩ. If these resis-
tances were , connected in series, then what will be
the resistance. [Ans. n2 R]
57. What is the smallest resistance that can be obtained
by the combination of n resistors each resistance r. 64. Find the equivalent resistance of the circuit then
[Ans. r/n] shown in Fig., between the points A and B.
58. Two equal resistors are connected in series across a [Ans. 4/3 J]
battery and consume a power of P . If these are con-
nected in parallel, then what will be the power con-
sumed. [Ans. 4P]
59. A wire of uniform cross-section and length has a re-

m
sistance of 16 Ω. It is cut into four equal parts. Each
part is stretched uniformly to length l and all the four
stretched parts are connected in parallel, calculate the
total resistance of the combination so formed. Assume
that stretching of wire does not cause any change in
the density of its material. [Ans. 16 Ω]
60. Two wires X, Y have the same resistivity, but their
cross-sectional areas are in the ratio 2:3 and lengths
ra EMF, TERMINAL P.D. INTERNAL
RESISTANCE
in the ratio 1:2. They are first connected in series and
G
then in parallel to a d.c. source. Find out the ratio of 65. Differentiate EMF and potential difference.
drift speeds of the electrons in the two wires for the
two cases. [Ans. 3:2; 2:1] 66. If n cells each of emf ′ ϵ′ and internal resistance ′ r′ are
connected in parallel, then what will be the total emf
u
61. What is the equivalent resistance between A and H of r
and internal resistance. [Ans. ϵ, ]
resistances as shown in fig. [Ans. 10 Ω] n
67. To draw a maximum current from a combination of
Ed

cells, how should the cells be grouped?


68. The reading on a high resistance voltmeter, when a
cell is connected across it, is 2.2 V. When the cell ter-
minals are connected to a resistance of 5 Ω as shown
in Fig., the voltmeter reading drops to 1.8 V. Find the
10
internal resistance of the cell. [Ans. Ω]
62. Calculate the value of the resistance R in the circuit 9
shown in the Fig, so that the current in the circuit is
0.2 A. What would be the potential difference between
points A and B ? [Ans. 5 Ω, 1 volt]

69. A battery of emf 12 V and internal resistance 2Ω is


connected to a 4 Ω resistor as shown in Fig. Show
that a voltmeter when placed across the cell and
63. The reading of an ideal ammeter, in the circuit shown across the resistor, in turn gives the same reading.
in Fig. equals (i) I when key K1 , is closed and key [Ans. 8 V, yes]

31 - Nikhil Kamboj — 8193989452 #edugramddun


Current Electricity

70. A cell of emf ϵ and internal resistance r is connected


across a variable load resistance R. It is found that
when R = 4Ω, the current is 1 A and when R increases
to 9 Ω, the current reduces to 0.5 A. Find the values 75. The galvanometer G in the circuit reads zero. Find the
of the emf ϵ and internal resistance r. [Ans. 5 V, 1 Ω] value of R. Batteries are resistance less. [Ans. 100 Ω]

71. A voltmeter of resistance 990 Ω is connected across a


cell of emf 3 V and internal resistance 3 Ω. Find the
terminal potential difference of the cell and percentage
error in the reading of voltmeter. [Ans. 2.99 V, 0.33%]

m
72. A battery of emf ϵ, and internal resistance r, gives a
current of 0.5 A with an external resistor of 12 Ω and
a current of 0.25 A with an external resistor of 25 Ω.
ra
Calculate (i) internal resistance of the cell and (ii) emf
of the cell. [Ans. 1 Ω, 6.5 V]

73. 4 cells each of identical emf E, internal resistance r, are


76. A voltmeter with resistance 500 Ω is used to measure
the emf of a cell of internal resistance 4 Ω. What will
G
connected in series to a variable resistor. Fig. shows be the percentage error in the reading of the voltmeter.
the variation of terminal voltage of the combination [Ans. 0.8 %]
with the current output:
(a) What is the emf of each cell used? 77. In the circuit shown in fig, when switch S1 is closed
u
(b) For what current from the cells, does maximum and S2 is open, the ideal voltmeter shows a reading
power dissipation occur in the circuit. 18 V. When switch S2 is closed and S1 is open, the
(c) Calculate the internal resistance of each cell. reading of the voltmeter is 24 V. What will be the
Ed

[Ans. (a) 1.4 V (b) 1 A (c) 0.7 Ω] reading of voltmeter when S1 and S2 both are closed?
[Ans. 14.4 V]

GROUPING OF CELLS
74. The following graph shows the variation of terminal
potential difference V , across a combination of three
cells in series to a resistor, versus the current I. (a)
Calculate the emf of each cell (b) For what current I, 78. In the two electric circuit shown in fig. determine the
will the power dissipation of the circuit be maximum. readings of ideal ammeter (A) and ideal voltemeter
[Ans. (a) 2V (b) 1 A] (V ). [Ans. (a) 7.5 V, 1.5 A (b) -1.5 V, 7.5 A]

32 - Nikhil Kamboj — 8193989452 #edugramddun


Current Electricity

85. Two bulbs 25 W, 220 V and 100 W, 220 V are given.


Which has higher resistance? [Ans. 25 W]
86. A generator is supplying power to a factory by cables
of resistance 20 Ω. If the generator is generating 50
kW power at 5000 V. what is the power received by
factory ? [Ans. 48 kW]
87. Calculate the amount of heat produced per second (in
calories) when a bulb of 100 W, 220 V glows, assuming
that only 20% of electric energy is convened into light.
J = 4.2 J/cal [Ans. 19.05 cal]
88. An electric motor operates on a 50 V supply and draws
a current of 15 A. If the motor yields a mechanical
power of 150 W, estimate the power dissipated across
its windings. Also find the efficiency of the motor ?
[Ans. 600 W, 20%]
89. Find the resistance of 240 V-200 watt electric
bulb when glowing. If this resistance is 10 times
the resistance at 0°C and the temperature of the

m
glowing filament is 2000°C, then find the tem-
perature coefficient of resistance of the filament.
79. A cell of emf 1.1 V and internal resistance 0.5 Ω is
[Ans. 288 Ω; 4.5 x 10−3 °C−1 ]
connected to a wire of Resistance 0.5 Ω. Another cell
of the same emf is connected in series but the cur- 90. Three equal resistances connected in series across a
rent in the wire remains the same. Find the internal
resistance of second cell. [Ans. 1 Ω]
ra
80. A set of 4 cells each of emf 2 V and internal resistance
1.5 Ω are connected across an external load of of 10Ω
source of e.m.f. consume 20 watt. If the same
resistors are connected in parallel across the same
source of e.m.f., what would be the power dissipated
? [Ans. 180 watt]
G
with 2 rows, two cells in each row. Calculate the cur- 91. In a house having 220 V line, the following appliances
rent in each row and potential difference across 10 Ω. are working (i) a 60 W bulb (ii) a 1000 W heater
[Ans. 0.175 A, 0.175 A, 3.5 V] (iii) a 40 W radio. Calculate (a) the current drawn
by heater and (b) the current passing through the fuse
u
81. Find the minimum number of cells required to produce
an electric current of 1.5 A through a resistance of 30 line. [Ans. (a) (50/11) A (b)5 Ω]
Ω. Given that the emf and internal resistance of each 92. A series battery of 10 lead accumulators each of emf
Ed

cell are 1.5 V and 1.0 Ω respectively. [Ans. 120] 2 V and internal resistor 0.25 ohm is charged by
82. n identical cells are joined in series with two cells A a 220 V d.c. mains. To limit the charging cur-
and B with reverse polarities. EMF of each cell is E rent, a resistance of 47.5 Ω is used in series in the
and internal resiatnce r. What is the potential differ- charging circuit. What is (a) the power supplied by
2 the mains and (b) power dissipated as heat ? (c)
ence across the cell A or B. [Ans. 2E 1 − ] Account for the difference of power in (a) and (b).
n
83. A set of 6 cells, each of emf 2 V and internal resis- [Ans. (a) 880 W (b) 800 W (c) 80 W stored in
tance 1.5 Ω, are connected across an external load of battery as chemical energy]
10 with 2 rows, 3 cells in each branch. Calculate cur- 93. In India electricity is supplied for domestic use at 220
rent in each branch and potential difference across 10 V. It is supplied at 110 V in U.S.A. If the resistance
Ω. [Ans. 0.245 A, 0.245 A, 4.9 V] of a 60 W bulb use in India is R. What will be the
84. A student connects a cell, of emf ϵ2 and internal re- resistance of a 60 W bulb for use in USA? [Ans. R/4]
sistance r2 , with a cell of emf ϵ2 and internal resis- 94. When three identical bulbs of 60 W, 200 V rating are
tance r1 , such that their combination has a net in- connected in series to a 200 V supply, then what will
ternal resistance less than r1 . This combination is be the power drawn by them. [Ans. 20 W]
then connected across a resistance R. Draw a circuit 95. How much electric energy is consumed by a 100
of the ’set up’ and obtain an expression for the cur- W lamp used for 6 hours everyday for 30 days?
rent flowing through the resistance R. (CBSE 2016) [Ans. 18 kWh]
ϵ1 r2 × ϵ2 r1
[Ans. ]
(r1 × r2 ) + R()r1 + r2 ELECTRIC ENERGY AND HEATING
EFFECT OF CURRENT
ELECTRIC POWER

33 - Nikhil Kamboj — 8193989452 #edugramddun


Current Electricity

96. A dry cell of emf 1.6 V and internal resistance of 0.10


Ω is connected to a resistor of resistance R Ω. If the
current drawn from the cell is 2 A, then (i) What is the
voltage drop across R ? (ii) What is the rate of energy
dissipation in the resistor ? [Ans. (i) 1.4 V (ii) 2.8 W]
97. Two wires A and B of same material and mass, have
their lengths in the ratio 1 : 3. On connecting them,
one at a time to the same source of emf, the rate of
heat dissipated in B is 10 W. What is the rate of heat
dissipated in A ? [Ans. 90 W] 105. A house wiring, supplied with a 220 V supply line is
98. An electric bulb is marked to 100 W, 230 V. If the sup- protected by a 9 ampere fuse. Find the maximum
ply voltage drops to 115 V, what is the heat and light number of 60 W bulbs in parallel that can be turned
energy produced by the bulb in 20 minutes. Calculate on. [Ans. 33]
the current flowing through it. [Ans. 30 kJ, (5/23) A]
106. (a) Two bulbs rated 25 W - 220 V and 100 W -
99. An electric kettle used to prepare tea, takes 2 minutes 220V are connected in series to a 440 V supply.
to boil 4 cups of water (1 cup contains 200 c.c. of Show with necessary calculations which bulb if any
water), and the room temperature is 25°C. (a) If the will fuse ? (b) What should happen if the two
cost of electricity is Rs. 3.00 per unit of electricity, bulbs were connected in parallel to the same supply ?
calculate the cost of boiling 4 cups of water. (b) What [Ans. (a) 25 W bulb will fuse (b) Both the bulbs will
will be the corresponding cost if the room temperature fuse]

m
drops to 5°C ? [Ans. (a) 21 paise (b) 27 paise]
100. Find the heat developed per minute in each of 107. A cell sends a current through a resistance R1 for time
the three resistors R1 , R2 and R3 shown in Fig. t, next the same cell sends current through another re-
[Ans. 640 J; 1280 J; 960 J] sistance R2 for the same time t. If the same amount
ra of heat is developed in both the resistances,
p then find
the internal resistance of the cell. [Ans. R1 R2 ]

108. Two wires of copper are of the same length but


have different diameters. When they are con-
G
nected in series across a battery, the heat gener-
ated is H1 When connected in parallel across the
same battery, the heat generated during the same
time is H2. In which case heat produced is more.
u
V2
[Ans. H1 ¡ H2; H = I 2 RT = H is inversely
101. A motor operating at 110 V draws a current of 2A. If R
proportional to R.]
Ed

the heat is developed in the motor at the rate of 10


cal/s, what is its efficiency ? [Ans. 80.9%] 109. An electric heating element consumes 500 W, when
102. A room is lighted by 200 W, 124 V incandescent lamps connected to a 100 V line. If the line voltage be-
fed by a generator whose output voltage is 130 V. The comes 150 V, than what will be the power consumed.
conducting wires from the generator to the user are [Ans. 750 W]
made of aluminium wire of total length 150 m and
cross-sectional area 15 mm2 . How many such lamps 110. A uniform wire connected across a supply produces
can be installed ? What is the total power consumed heat H per second. If wire is cut into three equal
by the user ? sp. resistance of aluminium is 2.9 x parts and all the parts are connected in parallel across
10 Ωm [Ans. 12, 2.4 kW]
−8 the same supply, what will be the heat produced per
second now? [Ans. H/3]
103. Twenty one electric bulbs are connected in series with
the mains of a 220 V supply. After one bulb is fused, KIRCHOFF’S LAWS
the remaining 20 bulbs are again connected in series
across the same mains. By what percentage will the
illumination of (i) a bulb change (ii) all the bulbs 111. State kirchhoff’s law in electricity and deduce it using
change ? [Ans. (i) 10.25% (ii) 5%] kirchoff’s rules. Also state the fundamental concepts
104. Three 60 W, 120 V light bulbs are connected on which two kirchhoff’s rules are based.
across a 120 V power line as shown. Fig.
112. Find the potential difference across each cell
Find (a) the voltage across each bulb (b)
and the rate of energy dissipated in R, fig.
the total power developed across three bulbs. 48 48
[Ans. (a) VA = 80 V ; VB = Vc = 40V ; (b) 40 W] [Ans. V, V, 11.75 W]
7 7

34 - Nikhil Kamboj — 8193989452 #edugramddun


Current Electricity

117. Eleven identical wires each of resistance 5 Ω are joined


to form the edges of an incomplete cube. Find the
total resistance from one end of vacant edge to the
113. Using Kirchoff’s rule, calculate the potential differ- other end of the same cube. If a battery of emf 2 V
ence between B and D in the circuit as shown in fig. is connected across the vacant edge of cube. Find the
[Ans. 2/13 V] current supplied by the battery. [Ans. 7 Ω, 1/7 A]

118. Using Kirchoff’s rules, write the expression for the cur-
rent I1 , I2 and I3 in the circuit diagram shown in fig..
[Ans. I1 = 2/13AA; I2 = 7/13A; I3 = 9/13A]

m
114. Apply Kirchoff’s rule to the loops P RSP and P RQP

in the circuit shown in fig. [Ans. A; A;


11
ra
to write the expressions for the currents I1 , I2 and I3
39 4
A]
G
860 215 172

119. Twelve identical wires, each of resistance 6 Ω, are ar-


ranged to form a skeleton cube. A current of 40 mA
u
is led into the cube at one corner and out at the diag-
onally opposite corner. Calculate the potential differ-
ence developed across these corners and the effective
Ed

resistance of the network. [Ans. 0.2 V; 5 Ω]

120. In the circuit fig. E, F, G, and H are cells of emf 1,


2, 2, and 4 volt and their internal resistances are 1, 2,
2 and 1 Ω respectively. Calculate (i) the potential dif-
ference between D and B,(ii) the potential difference
115. In the circuit shown in fig, the galvanometer G shows across the terminals and each of the cells G and H.
zero deflections. If the batteries A and B have negli- [Ans. (i) 0.29 V (ii) 3.14 V; 3.43 V]
gible internal resistance find the value of the resistor
R. [Ans. 100 Ω]

116. Find the currents I1 , I2 and I3 through the 121. In the given circuit fig. assuming point A to be at zero
three resistors of the circuit as shown in fig.. potential, use kirchoff’s rules to determine the poten-
[Ans. Zero in each case] tial at point B. [Ans. 2 V]

35 - Nikhil Kamboj — 8193989452 #edugramddun


Current Electricity

the 40 Ω and 20 Ω resistors in the following circuit.


[Ans. 0 A, 4 A]

122. Find the potential difference across R3 in the circuit


(E1 R2 + E2 R1 )R3
shown in fig. [Ans. ]
R1 R2 + R2 R3 + R1 R3

128. Find the value of the unknown resistance X, in the


following circuit, if no current flows through the sec-
tion AO, fig.. Also, calculate the current drawn by
the circuit from the battery of emf 6V and negligible
internal resistance. [Ans. 4 Ω; 1 A]

m
WHEATSTONE BRIDGE PRINCIPLE AND
SLIDE WIRE BRIDGE

123. In a Wheatstone bridge, the resistance each arm is


10Ω. If the resistance galvanometers is also 10Ω, then
what will be the effective resistance of the bridge.
[Ans. 10 Ω]
ra
124. (i) In a meter bridge, the balance point is found to be
at 30 cm from the end A when the Resistance R in
G
left gap of bridge is of 12 Ω. Find resistance S in the
right gap of bridge.
(ii) If the cell and the galvanometer are interchanged 129. The resistance in the two arms of the meter bridge are
at the balance point, would it effect the flow of current R = 5Ω and S respectively. When the resistance S
u
through the galvanometer. is shunted with an equal resistance, the new balance
(iii) Calculate the balance point of the bridge if R as S length found to be 1.5 l1 , where l1 is the initial bal-
are interchange. [Ans. (i) 28 Ω (ii) no effect (iii) 70 cm] ancing length. calculate the value of S. [Ans. 10 Ω]
Ed

125. P , Q, R and S are four resistance wires of resistances


3, 3, 3 and 4 ohms respectively. They are connected
to form the four arms of Wheatstone bridge circuit.
Find out the resistance with which S must be shunted
in order that bridge may be balanced. [Ans. 12 Ω]
126. In the circuit fig. a meter bridge is shown in its bal-
anced state. The meter bridge wire has a resistance of
1 Ω/cm. Calculate the unknown resistance X and the
current drawn from the battery of negligible internal
resistance. [Ans. 4 Ω, 0.66 A]

130. A resistance R = 2Ω is connected to one of the gaps


in a meter bridge, which uses a wire of length 1 m. An
unknown resistance X > 2Ω is connected to the other
gap as shown in fig. The balance point is noticed at
l cm from the positive end of the battery. On inter-
changing R and X, it is found that the balance point
further shifts by 20 cm (away from end A). Neglect-
ing the end correction, calculate the value of unknown
127. Using Kirchhoff’s rules, calculate the current through resistance X used. [Ans. 3 Ω]

36 - Nikhil Kamboj — 8193989452 #edugramddun


Current Electricity

the position of the null point when the 30 Ω resis-


tance is connected in series with resistance Y instead
of X. Determine the values of resistances X and Y .
[Ans. 33.3 cm; 60 Ω; 90 Ω]

131. In the simple Wheatstone bridge circuit, where the


length AB of bridge wire is 1 m the resistors X and
Y have values 5 Ω and 2 Ω respectively, fig.. When X
is shunted by a length of a wire, the balance point is
found to be 0.625 m from A. What is the resistance of MISC PROBLEM
the shunt? If the shunt wire is 0.75 m long and 0.25
mm in diameter, what is the resistivity of the material
of the wire? [Ans. 10 Ω; 6.54 ×10−7 Ω m] 134. 1 kg piece of copper is drawn into a wire 1 mm thick,
and another identical piece into a wire 2 mm thick.
Compare the resistance of these wires. [Ans. 16:1]
135. An infinite ladder net work of resistance is constructed
with 1 Ω and 2 Ω resistances, as shown in Fig. The 6 V

m
battery between A and B has negligible internal resis-
tance. (i) Show that the effective resistance between
A and B is 2 Ω. (ii) What is the current that passes
through the 2 Ω resistance nearest to the battery ?

null point D is obtained at a distance of 40 cm from


end A of the the meter bridge wire. If a resistance
ra
132. In the meter bridge experiment setup shown in fig. the

of 10 Ω is connected in series with R1 , null point is


[Ans. 1.5 A]
G
obtained at AD = 60cm. Calculate the values of R1
and R2 . [Ans. 8 Ω and 12 Ω]
u

136. A cell of emf 2 V and internal resistance 0.1Ω sup-


plies a current through a coil of resistance 11.9Ω. The
Ed

current is being measured by an ammeter whose resis-


tance is 6Ω. What reading does it give ? What is the
percentage difference from the actual current, when
the meter is not used ? [Ans. 0.11 A ; 33.3%]
137. You are given several identical resistances each of value
R = 10Ω and each capable of carrying a maximum cur-
133. Fig. shows experimental set up of a meter bridge. rent of one ampere. It is required to make a suitable
When the two unknown resistances X and Y are in- combination of these resistances so as to have a resis-
serted, the null point D is obtained 40 cm from the tance of 5 capable of carrying a current of 4 ampere.
end A. When a resitance of 30 Ω is connected in Find the minimum number of resistances of the type
series with X, the null point shifts by 10 cm. Find R that will be required for the job. [Ans. 8]

37 - Nikhil Kamboj — 8193989452 #edugramddun


Moving Charge and Magnetism
Important Formulae and Notes

µ0 Idl sin θ ▶ Torque on a rectangular coil, τ = N IBA cos θ


▶ Biot-Savart’s Law, dB = ×
4π r2 where θ = angle rotated by the plane of the coil from
µ0 I the direction of the magnetic field
▶ Ampere’s law, B = ×
2π r ▶ Magnetic dipole moment, M = N IA
▶ Work done by magnetic force on magnetic pole (of where A = area of current carrying coil
strength m) for n rotations
▶ Current passing through moving coil galvanometer,
W =µmnI
▶ Magnetic field due to a current carrying conductor
 
K
I= θ
N BA
µ0 I
B= (sin ϕ1 + sin ϕ2 ) where Kθ = restoring couple
4π r
N IBA = deflection couple
where ϕ1 and ϕ2 are angles between the perpendicu-
N AB
lar from the observation point to the conductor and ▶ Current sensitivity, Si =
K
the line joining the two ends of the conductor to the
N AB Si
observation point. ▶ Voltage sensitivity, Sv = =
KG G
▶ Magnetic induction in various cases where G = resistance of galvanometer

m
(a) If a current of I amperes flows through a cir- Ig × G
▶ Shunt of an ammeter, S =
cular coil of radius a, containing N turns, then I − Ig
magnetic induction at the centre of the coil B = where I = range of ammeter
µ0 N I Ig = maximum current that can pass through the gal-
2a
(b) Magnetic induction produced on the axial line of
a circular coil of radius a, containing N turns,
B=
µ0 N a2 I
ra vanometer
▶ Resistance connected in series to voltmeter, R =
V
Ig
−G
G
2(a2 + x2 )3/2 where V = range of voltmeter
▶ Magnetic force on a charged particle ▶ Motion of charged particle inside magnetic field
F = Bqvsinθ mv
# # (a) Radius of the circular path, r =
▶ Lorentz force, F = q( E + #
u
v × B) Bq
where m = mass of the charge
▶ Force on a current carrying conductor placed inside a 2πmv cos θ
magnetic field, (b) Pitch =
Ed

Bq
# # ▶ Motion of charged particle in electric and magnetic
F = | I l × B| = BIl sin θ
fields
▶ Magnetic force acting on the particle provides it nec- Bq
(a) Cyclotron frequency ν =
essary centripetal force for the circular motion, i.e. 2πm
(b) Maximum energy attained by charged (positive)
mv 2 1 B 2 q 2 R2
qvb = particle Emax =
r 2 m
where R = radius of dees of cyclotron.
▶ Force between two parallel conductors carrying cur- ▶ Motion of charged particles in an electric field
rents I1 and I2 (a) parallel to the electric field
µ0 I1 I2 l
F = (A) Force acting on the charged particle
2π r
▶ Magnetic field due to a solenoid F = qE
(a) At a point well inside a solenoid where q = charge on the particle
(B) Electric field
B = µ0 N I V
E=
d
(b) At a point on one end of the solenoid where V = potential difference applied be-
1 tween the positive and negative plates
B= µ0 N I d = distance between them
2
Moving Charge and Magnetism

(C) Acceleration produced on the charged parti- (a) Perpendicular to the magnetic field
cle (A) Force acting on the charged particle
F qE
a= = # #
m m F = q #
v ×B
where m = mass of the charged particle
(D) Kinetic energy acquired by the particle where q = charge on particle,
v = velocity of charged particle
1 B = magnetic field of induction
K= mv 2 = qV
2 (B) if θ is the angle between v and B then force
acting on the charged particle
(b) Perpendicular to the electric field
(A) Force acting on the charged particle, F = qvBsinθ
F = qE (C) The magnetic force F supplies the necessary
centripetal force and the charged particle un-
(B) Acceleration produced on the charged parti- dergoes uniform circular motion given by
cle,
F qE mv 2
a= = F = Bqv =
m m r
(C) If l is the length of the electric field tra-
where m = mass of the particle
versed by the charged particle in a time t,
l r = radius of the circle or

m
mv p
then t = r= =
v Bq Bq
(D) Displacement of the particle from its original where p = momentum of the particle
direction at the end of the electric field, (D) KE of the particle
1
1 2 ra K = mv 2 = qV or
s= at r2
2
r
2K 2qV
v= =
▶ Motion of charged particle in a magnetic field m m
G
Multiple Choice Questions

1. The magnetic field of a given length of a wire for sin- a) the radius of the coil
gle turn coil at its centre is B. Then its value for two b) the external magnetic field
u
turns of coil(Length of wire remains same) will be c) the number of turns of the coil
a) B/4 b) B/2 c) 4B d) 2B d) all the above
Ed

2. When charged particle enters a uniform magnetic field, 5. The permeability of a paramagnetic substance is:
its K.E.: a) very large
a) remains constant b) increases b) small but more than unity
c) decreases d) becomes zero c) less than unity
d) negative
3. A cubical region of space is filled with some uniform
electric and magnetic fields. An electron enters the 6. If an electron is moving with velocity n produces a
cube across one of its faces with velocity v and a magnetic field B , then
positron enters via opposite face with velocity -v. At a) the direction of field B will be same as the direc-
this instant which of the given state is not correct tion of velocity n
a) the electric forces on both the particles cause b) the direction of field B will be opposite to the
identical accelerations. direction of velocity n
b) the magnetic forces on both the particles cause c) the direction of field B will be perpendicular to
equal accelerations. the direction of velocity n.
c) Only electron gains or looses energy. d) the direction of field B does not depend upon the
d) the motion of the centre of mass (CM) is deter- direction of velocity n .
mined by E alone.
.
4. The sensitivity of a tangent galvanometer can be in- 7. Current flows through uniform, square frames as
creased by increasing: shown in the figure. In which case is the magnetic

39 - Nikhil Kamboj — 8193989452 #edugramddun


Moving Charge and Magnetism

field at the centre of the frame not zero? a) (I) is correct


b) (II) is correct
c) both (I) and (II) are correct
d) neither (I) nor (II) is correct

14. A strong magnetic field is applied on a stationary elec-


tron. Then the electron
a) moves in the direction of the field.
b) remained stationary.
8. A short bar magnet has a magnetic moment of 0. 65 J c) moves perpendicular to the direction of the field.
T-1, then the magnitude and direction of the magnetic d) moves opposite to the direction of the field.
field produced by the magnet at a distance 8 cm from
15. Two α−particles have the ratio of their velocities as
the centre of magnet on the axis is
3 : 2 on entering the field. If they move in different
a) 2.5 × 10-4 T, along NS direction circular paths, then the ratio of the radii of their paths
b) 2.5 × 10-4 T along SN direction is
c) 4.5 × 10-4 T, along NS direction a) 2 : 3 b) 3 : 2 c) 9 : 4 d) 4 : 9
d) 4.5 × 10-4 T, along SN direction
16. A current carrying power line carries current from west
9. A current carrying loop is placed in a uniform mag- to east. Then the direction of the magnetic field 2 m
netic field. The torque acting on it does depend upon above it is :

m
a) area of loop b) value of current a) west to east b) south to north
c) magnetic field d) all the above c) north to south d) None of these

17. The force between two parallel current carrying con-


10. In a moving coil galvanometer the deflection (F) on ra ductors is F. If the current in each conductor is dou-
the scale by a pointer attached to the spring is
    bled, then the force between them becomes :
NA N a) 4F b) 2F c) F d) F/4
a) I b) I
 kB   kAB 
G
N AB N AB 18. A charge + q is sent through a magnetic field. The
c) I d)
k kI force acting on it is maximum when the angle between
the direction of motion of the charged particle and the
11. A moving coil galvanometer can be converted into an magnetic field :
u
ammeter by a) 0° b) 45° c) 90° d) 180°
a) introducing a shunt resistance of large value in
19. Biot-Savart law indicates that the moving electrons
series.
Ed

(velocity v) produce a magnetic field B such that


b) introducing a shunt resistance of small value in
parallel. a) B perpendicular to v.
c) introducing a resistance of small value in series. b) B parallel to v.
d) introducing a resistance of large value in parallel. c) it obeys inverse cube law.
d) it is along the line joining the electron and point
of observation.
12. The conversion of a moving coil galvanometer into a
voltmeter is done by 20. A circular coil of radius 4 cm and of 20 turns carries a
a) introducing a resistance of large value in series. current of 3 amperes. It is placed in a magnetic field
b) introducing a resistance of small value in of intensity of 0.5 weber/m2. The magnetic dipole mo-
parallel. ment of the coil is
c) introducing a resistance of large value in parallel. a) 0.15 ampere-m2 b) 0.3 ampere-m2
d) introducing a resistance of small value in series. c) 0.45 ampere-m2 d) 0.6 ampere-m2

13. When a magnetic compass needle is carried nearby to 21. The conversion of a moving coil galvanometer into a
a straight wire carrying current, then voltmeter is done by
(I) the straight wire cause a noticeable deflection in a) introducing a resistance of large value in series.
the compass needle. b) introducing a resistance of small value in
(II) the alignment of the needle is tangential to an parallel.
imaginary circle with straight wire as its centre and c) introducing a resistance of large value in parallel.
has a plane perpendicular to the wire d) introducing a resistance of small value in series.

40 - Nikhil Kamboj — 8193989452 #edugramddun


Moving Charge and Magnetism

22. An electron is projected with uniform velocity along c) if the wire moves under the influence of B, no
the axis of a current carrying long solenoid. Which of work is done by the force.
the following is true? [NCERT Exemplar] d) If the wire moves under the influence of B, no
a) The electron will be accelerated along the axis. work is done by the electric force on the ions,
b) The electron path will be circular about the axis. assumed fixed within the wire.
c) The electron will experience a force at 45° to the
29. The maximum current that can be measured by a gal-
axis and hence execute a helical path.
vanometer of resistance 40 W is 10 mA. It is converted
d) The electron will continue to move with uniform into voltmeter that can read upto 50 V. The resistance
velocity along the axis of the solenoid. to be connected in the series with the galvanometer is
23. Which of the following shows that the earth behaves a) 2010 W b) 4050 W c) 5040 W d) 4960 W
as a magnet? 30. The strength of magnetic field at the centre of circular
a) Repulsion between like poles . coil is
b) Attraction between unlike poles
c) Null points in the magnetic field of a bar magnet
d) No existence of isolated magnetic poles

24. A charged particle of mass m and charge q travels on a


circular path of radius r i.e., perpendicular to the mag-
netic field B. The time taken by particle to complete

m
on revolution is
2πqB 2πm 2πqm 2πq 2 B
a) b) c) d)
m qB B m 31. A current carrying closed loop of an irregular shape
lying in more than one plane when placed in uniform

The magnetic field at the centre of the loop is


a) 1.57 × 10-3Wb/m2
c) 2.0 × 10-3 Wb/m2
b) 8.0 × 10-5 Wb/m2
d) 3.l4 × 10-1 Wb/m2
ra
25. Circular loop of radius 0.0157 m carries a current 2 A. magnetic field, the force acting on it
a) will be more in the plane where its larger posi-
tion is covered
b) is zero
G
c) is infinite
26. The correct plot of the magnitude of magnetic field B d) may or may not be zero
vs distance r from centre of the wire is, if the radius
of wire is R 32. A current loop placed in a non-uniform magnetic field
experiences
u

a) a force of repulsion b) a force of attraction


c) a torque but not d) a force and a torque
Ed

force

33. If the beams of electrons and protons move parallel to


each other in the same direction, then they
a) attract each other
27. The nature of parallel and anti-parallel currents are b) repel each other
a) parallel currents repel and antiparallel cur¬rents c) no relation
attract. d) neither attract nor repel
b) parallel currents attract and antiparallel cur-
34. According to Gauss’s theorem in magnetism, sur-
rents repel.
face integral of magnetic field intensity over a surface
c) both currents attract. (closed or open) is always
d) both currents repel.
a) -1 b) 1 c) 0 d) infinity
28. Consider a wire carrying a steady current I placed in a 35. Tesla is a unit of
uniform magnetic field B perpendicular to its length.
Consider the charges inside the wire. It is known that a) electric flux b) magnetic flux
magnetic forces do not work. This implies that c) magnetic field d) electric field

a) motion of charges inside the conductor is unaf- 36. A conducting circular loop of radius r carries a con-
fected by B, since they do not absorb energy. stant current i. It is placed in a uniform magnetic
b) Some charges inside the wire move to the surface field B, such that B is perpendicular to the plane of
as a result of B. the loop. The magnetic force acting on the loop is

41 - Nikhil Kamboj — 8193989452 #edugramddun


Moving Charge and Magnetism

a) irB b) 2priB c) zero d) priB a) both momentum and energy of particle change.
b) momentum as well as energy are constant.
37. An electron is projected along the axis of a circular c) energy is constant but momentum changes.
conductor carrying the same current. Electron will d) momentum is constant but energy changes.
experience
a) a force along the axis 46. A circular current loop of magnetic moment M is in
b) a force perpendicular to the axis. an arbitrary orientation in an external magnetic field
c) a force at an angle of 4 degree with axis. B. The work done to rotate the loop by 30° about an
d) no force experienced. axis perpendicular to its plane is

a) MB b) 3M B 2 c) MB2 d) zero
38. Three long, straight parallel wires, carrying current
are arranged as shown in the figure. The force experi- 47. A rectangular loop carrying a current i is situated near
enced by a 25 cm length of wire C is a long straight wire such that the wire is parallel to
the one of the sides of the loop and is in the plane of
the loop. If a steady current I is established in wire as

shown in figure, the loop will


a) 10-3 N b) 2.5 × 10-3 N a) rotate about an axis parallel to the wire.

m
c) zero d) 1.5 × 3 N b) move away from the wire or towards right.
c) move towards the wire.
39. A positive charge enters in a magnetic field and travels
d) remain stationary.
parallel to but opposite the field. If experiences
a) an upward force.
c) an accelerated force.
b) a downward force.
d) no force.

40. The magnetic field around a long straight current car-


rying wire is
ra 48. A charged particle is moving in a cyclotron, what ef-
fect on the radius of path of this charged particle will
occur when the frequency of the ratio frequency field
is doubled?
G
a) It will also be doubled.
a) spherical symmetry b) cylindrical symmetry b) It will be halved.
c) cubical symmetry d) unsymmetrical c) It will be increased by four times.
d) It will remain unchanged.
u
41. The current sensitivity of a moving coil galvanometer
increases with decrease in: 49. Which of the following is not correct about cyclotron?
a) magnetic field b) area of a coil a) It is a machine to accelerate charged particles or
Ed

c) number of turns d) couple per unit twist ions to high energies.


b) Cyclotron uses both electric and magnetic fields
42. A current carring coil is placed in a uniform magnetic in combination to increase the energy of charged
field. If the coil turns through an angle θ, then the particles.
torque is directly proportional to c) The operation of the cyclotron is based on the
a) sin θ b) cos θ c) cot θ d) tan θ fact that the time for one revolution of an ion is
independent of its speed or radius of its orbit.
43. The magnetic moment of a current I carrying circular d) The charged particles and ions in cyclotron can
coil of radius r and number of turns N varies as move on any arbitrary path.
a) 1/r2 b) 1/r c) r d) r2
50. What happens to the magnetic field at the centre of
44. In a cyclotron, a charged particle a circular current carrying coil if we double the radius
of the coil keeping the current unchanged?
a) undergoes acceleration all the time.
b) speeds up between the dees because of the mag- a) halved b) doubled
netic field. c) quadrupled d) remains unchanged
c) speeds up in a dee.
51. When we double the radius of a coil keeping the cur-
d) slows down within a dee and speeds up between
rent through it unchanged, what happens to the mag-
dees.
netic field directed along its axis at far off points?
45. If a charged particle moves through a magnetic field a) halved b) doubled
perpendicular to it c) quadrupled d) remains unchanged

42 - Nikhil Kamboj — 8193989452 #edugramddun


Moving Charge and Magnetism

52. The strength of the magnetic field around an infinite a) 25 × 10−7 N towards wire.
charge current carrying conductor is b) 25 × 10−7 N away from wire.
a) same everywhere c) 35 × 10−7 N towards wire.
b) inversely proportional to the distance d) 35 × 10−7 N away from wire.
c) directly proportional to the distance
d) None of these
53. An electron having mass ‘m′ and Kinetic energy E en- 57. In a circular coil of radius r, the magnetic field at the
ters in uniform magnetic field B perpendicular, then centre is proportional to
its frequency will be
eE 2πm a) r2 b) r c) 1/r d) 1/r2
a) b)
qmB eB
eB 2m
c) d)
2πm eBE 58. The SI unit of magnetic dipole moment is
54. A wire of length 2 metre carries a current 1 ampere,
is bent to form a circle. The magnetic moment of the a) Ampere b) Ampere metre2
coil is c) Tesla d) None of these
a) 2p b) p/2 c) p/4 d) 1/p
55. In an inertial frame of reference, the magnetic force on 59. Which one of the following is correct statement about
a moving charged particle is F . Its value in another magnetic forces?

m
inertial frame of reference will be
a) remained same a) Magnetic forces always obey Newton’s third law.
b) changed due to change in the amount of charge b) Magnetic forces do not obey Newton’s third law.
c) changed due to change in velocity of charged c) For very high current, magnetic forces obey New-
particle
d) changed due to change in field direction
56. What is the net force on the rectangular coil?
ra ton’s third law.
d) Inside low magnetic field, magnetic forces obey
Newton’s third law.
G
60. A charged particle is moving on circular path with
velocity v in a uniform magnetic field B, if the veloc-
ity of the charged particle is doubled and strength of
u
magnetic field is halved, then radius becomes

a) 8 times b) 4 times
Ed

c) 2 times d) 16 times

Practice Questions and Numerical

BIOT-SAVART’S LAW AND MAGNETIC of 3 A. Calculate the magnetic field at a point mid
FIELD DUE STRAIGHT CONDUCTOR way between them when the currents in them are :
(i) in the same direction and (ii) in opposite direction.
1. A current of 10 A is flowing east to west in a long wire [Ans. (i) Zero (ii) 2 ×10−5 T, normally downwards]
kept in the east-west direction. Find magnetic field in
a horizontal plane at a distance of (i) 10 cm north (ii)
20 cm south from the wire, and in a vertical plane at a 4. A long straight wire carrying a current of 25 A is
distance of (iii) 40 cm downwards, (iv) 50 cm upwards. placed in an external uniform magnetic field 3 × 10−4
[Ans. (i) 2 × 10−5 T, vertically downwards (ii) 10−5 T parallel to the current. Find the magnitude of the
T, vertically upward (iii) 5 × 10−6 T, horizontally resultant magnetic field at a point 1.5 cm away from
pointing south (iv) 4 × 10−6 T, horizontally pointing the wire. [Ans. 4.48 ×10−4 T]
north.]
2. Calculate the magnetic field induction at the centre of
the coil in the form of a square of side 10 cm, carrying 5. A rectangular loop of conductor of length a, breadth b
a current of 10 A. [Ans. 1.13 × 10−4 T] carrying current I as shown in Fig. Find the mag-
netic field √
induction at the centre O of the loop.
3. Two long parallel wires are placed at a distance of 12 2µ0 I a2 + b2
cm from each other in air. Each wire has a current [Ans. ]
πab

43 - Nikhil Kamboj — 8193989452 #edugramddun


Moving Charge and Magnetism

13. A conducting circular loop of radius r carries a con-


stant current i. It is placed in a uniform magnetic
field B, such that B is perpendicular to the plane of
the loop. What is the magnetic force acting on the
loop? [Ans. Zero]

14. Circular loop of radius 0.0157 m carries a current 2 A.


6. Two insulating infinite long conductors carrying cur-
What is the magnetic field at the centre of the loop.
rents I1 (= 2A) and I2 (= 4 A) lie mutually
[Ans. 8.0 × 10−5 Wb/m2]
perpendicular to each other in the same plane,
as shown in Fig. Find the magnetic field at
the point P (a, b), where a = 4 cm; b = 3 cm. 15. A straight wire of length π/2 metre, is bent into a cir-
[Ans. 1.66 ×10−5 T; normally outwards] cular shape. If the wire were to carry a current of 5 A,
calculate the magnetic field, due to it, before bending,
at a point distance 0.0l times the radius of the circle
formed from it. Also calculate the magnetic field, at
the centre of the circular loop formed, for the same
value of current. [Ans. 4 x 10−4 T ; 1.256 x 10−5 T]

16. The electron of hydrogen atom moves along a circular


path of radius 0.5 × 10−10 m (i) with a speed of 4 × 106
m/s (ii) with a frequency 6.8 × 1015 Hz. Calculate the

m
magnetic field produced at the centre of the circular
7. Equal currents I = lA are flowing through the long path. [Ans. 25.6 T, 13.4 T]
wires parallel to y-axis located at x = +1 m, X =
+2m, x = +4m, x = +Bm and so on but in opposite 17. The magnetic field due to a current-carrying circular
directions as shown in Fig. What is the magnetic field
induction at the origin O? [Ans. 1.33 ×10−7 T k̂]
ra loop of radius 10 cm at its centre is 0.60 × 10−4 T.
Find the magnetic field due to this loop at a point
on the axis at a distance of 4 cm from the centre.
[Ans. 4 x 10−5 T]
G
18. An alpha particle moves along a circular path of radius
2 A with a uniform speed of 2 × 106 m/s. Calculate
the magnetic field set up at the centre of circular path.
u
[Ans. 1.6 T]

8. An electric current is flowing due south along a 19. A circular segment of radius 20 cm subtends an an-
Ed

power line. What is the direction of the mag- gle of 60° at its centre. Fig. A current of 10 A
netic field at a point (a)above it and (b) below it? is flowing through it. Find the magnitude and di-
[Ans. above:East to west; below :west to south] rection of the magnetic field produced at the centre.
[Ans. 5.2 × 10−6 T]
9. A current carrying power line carries current from west
to east. Then what is the direction of the magnetic
field 2 m above it. [Ans. west to east]
MAGNETIC FIELD DUE TO CURRENT
THROUGH CIRCULAR COIL

10. What happens to the magnetic field at the centre


of a circular current carrying coil if we double the
radius of the coil keeping the current unchanged?
[Ans. reduces to half]
11. When we double the radius of a coil keeping the cur-
rent through it unchanged, what happens to the mag-
netic field directed along its axis at far off points?
20. A current I (= 4A) flows along a thin wire PQRS
[Ans. becomes four times]
shaped as shown in Fig.The radius of the curved part
12. The magnetic field of a given length of a wire for single of the wire is 10 cm. The angle θ = 90°. Find the
turn coil at its centre is B. Then find its value for two magnitude of the total magnetic field at the point O.
turns of coil(Length of wire remains same). [Ans. 4B] [Ans. 2.68 × 10−5 T]

44 - Nikhil Kamboj — 8193989452 #edugramddun


Moving Charge and Magnetism

25. A conductor carrying current I is of the type as shown


in Fig. Find the magnetic field induction at the com-
4µ0 Iθ
21. Two identical loops P and Q each of radius 5 cm mon centre O of all the three arcs. [Ans. ]
24πr
are lying in perpendicular planes such that they
have a common centre as shown in the Fig. Find
the magnitude and direction of the net magnetic
field at the common centre of the two coils, if they
carry currents equal to 3 A and 4 A respectively.
[Ans. 6.28 ×10−5 T; Direction can not be determined
as direction of current is not given]

m
ra AMPERE’S CIRCUITAL LAW

26. A long straight solid metal wire of radius R Carries


a current i, uniformly distributed over its circular
G
cross-section. Find the magnetic field at a distance
22. Two identical circular
√ coils. P and Q. carrying cur- r from axis of wire (i) inside and (ii) outside the wire.
rents 1 A and 3 A respectively, are placed concen- µ0 µr ir µ0 2i
[Ans. (i) (ii) ]
trically and perpendicular to each Other lying in the 2πR2 4πr
u
XY and YZ planes. Find the magnitude and direc- 27. A long straight solid conductor of radius 6 cm carries
tion of the net magnetic field at the centre of the coils. a current of 8 A, which uniformly distributed over its
µ0
[Ans. T in X-Z plane] circular cross-section. Find the magnetic field (a) at
Ed

R a distance of 3 cm from the axis of the conductor (b)


23. In the network shown in Fig., find the mag- at a distance 10 cm from the axis of the conductor.
netic field at the centre O of the coil. [Ans. (a) 1.33 × 10−5 T (b) 1.6 × 10−5 T ]
[Ans. 2 × 10−5 normally out of the plane of paper] MAGNETIC FIELD DUE TO SOLENOID

28. How is the K.E. of of a charged particle is effected


when it enters a uniform magnetic field.
29. A beam of α particles projected along +x-axis, ex-
periences a force due to a magnetic field along the
+y-axis. What is the direction of the magnetic field?
[Ans. negative of z-axis]
30. A charged particle is moving on circular path with ve-
locity v in a uniform magnetic field B, if the velocity
of the charged particle is doubled and strength of mag-
netic field is halved, then what will be the new radius.
24. An electric current of I ampere is flowing in
[Ans. increases 4 times]
a long conductor CG as shown in the Fig.
Find the magnitude and direction of magnetic 31. A charge +q is sent through a magnetic field. For what
induction at the angle between the direction of motion of the charged
 centre O of circular part.
µ0 I 1 particle and the magnetic field the force acting on it
[Ans. + 1 acting ⊥ r to loop upwards] is maximum? [Ans. 90]
2r π

45 - Nikhil Kamboj — 8193989452 #edugramddun


Moving Charge and Magnetism

32. Two α−particles have the ratio of their velocities as 20 cm. Calculate the magnitude of the force experi-
3 : 2 on entering the field. If they move in different enced by the electron. Write the direction of this force.
circular paths, then what is the ratio of the radii of [Ans. 1.6 × 10−19 N, in the plane of paper away from
their paths. [Ans. 3:2] XY]
33. A Solenoid of length 0.20 m, having 120 turns carries
a current of 2.5 A. Find the magnetic field : (a) in the
interior of the solenoid, (b) at one end of the solenoid.
[Ans. (a) 1.885 × 10−3 T (b) 0.9425 × 10−3 T]
34. A long wire carries a current of 20 A along
the directed axis of a long solenoid. The field
due to a solenoid is 4 mT. Find the resultant
field at a point 3 mm from the solenoid axis. 42. A proton, a deutron and an alpha particle, after
[Ans. 4.2 mT ; 18.4° with the directed axis] being accelerated through the same potential differ-
35. A copper wire having a resistance of 0.02 Ω/m is used ence enter a region of uniform magnetic field ⃗(B),
to wind a 500 turns solenoid of radius 2 cm and length in a direction perpendicular to ⃗(B). Find the ra-
30 cm. What should be the emf of the battery which tio of their kinetic energies. If the radius of pro-
when connected across the solenoid would produce ton’s circular path is 7 cm, What Wlll be the
a magnetic field of 10−2 T, near the centre of the radii of the paths of√deutron and√alpha particle?
solenoid. [Ans. 6 V] [Ans. 1 : 1 : 2, rd = 7 2 cm, rα = 7 2 cm]

m
MOTION OF CHARGED PARTICLES IN EF 43. A beam of proton passes undeflected with a horizontal
AND MF velocity v, through a region of electric and magnetic
fields, mutually perpendicular to each other and per-
36. An electron enters electric field of 104 V/m perpendic- pendicular to the direction of the beam. If the magni-
ular to the field with a velocity of 107 m/s. Find the

Mass of electron = 9.1×10−31 kg. [Ans. 3.51 × 109 m]


37. A chamber is maintained at a uniform magnetic
ra
vertical displacement of electron after 2 milliseconds.
tudes of the electric and magnetic fields are 100 kV/m,
50 mT respectively, calculate the velocity of the beam
‘v ′ . [Ans. 2 × 106 m/s]

44. If the maximum value of accelerating potential pro-


G
field of 5 × 10−3 T. An electron with a speed
of 5 × 107 m/s enters the chamber in a direction vided by a radio frequency oscillator be 10 kV, calcu-
normal to the field. Calculate (i) radius of the late the number of revolutions made by an on-particle
path (ii) frequency of revolution of the electron. in a cyclotron to achieve one-tenth of the speed of light.
[Ans. 470 revolutions]
u
[Ans. (i) 5.7 cm (ii) 1.4 X 108 Hz]
38. A proton projected in a magnetic field of 0.02 T 45. A cyclotron has an oscillatory frequency of 10 M
travels along a helical path of radius 6 cm and Hz and a dee radius of 60 cm. Calculate the mag-
Ed

pitch 24 cm. Find the components of velocity of netic field required to accelerate deutrons of mass
the proton along and perpendicular to the mag- 3.3x10−27 kg and charge 1.6 × 10−19 C. Find the
netic field. mass of proton = 1.6 × 10−27 kg. energy of deutrons emerging from the cyclotron.
[Ans. 1.2 × 10 m/s, 7.6 × 104 m/s]
5 [Ans. 1.3 T ; 14.74 MeV]
39. An electron after being accelerated a potential dif- 46. In a cyclotron, a magnetic field of 2.4 T is used
ference of 100 V enters a magnetic field of 0.004 to accelerate protons. How rapidly should the
T, perpendicular to its direction of motion. Calcu- electric field between the dees be reversed? The
late the radius of the path described by the electron. [Ans. 1.37 × 10−8 s, 3.64 × 107 Hz]
[Ans. 8.4 mm]
FORCE ON CURRENT CARRYING
40. A beam of protons enters a uniform magnetic field of
CONDUCTOR IN MF
0.3 T with a velocity of 4X105 m/s at an angle of 60°
to the field. Find the radius of the helical path taken
by the beam. Also find the pitch of the helix (distance 47. A current of 4A enters at the comer ‘a′ of a square
travelled by a proton parallel to the magnetic field dur- frame of side 10 cm and leaves at opposite corner
ing one period of rotation). [Ans. 1.2 cm ; 4.35 cm] ‘c′ . A magnetic field of B = 0.20 T acts in a di-
rection perpendicular to the plane of paper directed
41. An infinite long straight conductor XY is carrying a outwards Fig. Find the magnitude and direction of
current of 10 A. An electron is moving with a speed of the magnetic forces on the four arms of the frame.
105 m/s parallel to the conductor in air from point [Ans. 0.04 N on each arm; Force on arms ad and bc is
A to B, as shown in Fig.. The perpendicular dis- towards right and on arms ab and dc is downwards in
tance between the electron and the conductor XY is plane of frame]

46 - Nikhil Kamboj — 8193989452 #edugramddun


Moving Charge and Magnetism

ammeter reads 12 A. Find the magnetic force per


unit length. (a) of wire AB (b) of wire CD.
[Ans. (a) 2.4 × 10−4 N/m directed upwards (b) Zero]

48. A horizontal wire 0.1 m long having mass 3 g carries 53. A short conductor of length 5 cm is placed parallel to
a current of 5 A. Find the magnitude of the magnetic a long conductor of length l.5 m near its centre. The
field which must act at 30° to the length of the wire conductors carry currents 4 A and 3 A respectively
in order to support its weight. [Ans. 0.1176 T] in the same direction. What is the total force expe-
49. On a smooth inclined plane at 30° with the horizontal rienced by the long conductor, when they are 3 cm
a thin current carrying metallic rod is placed parallel apart? [Ans. 4 × 10−6 N ; attractive]
to the horizontal ground. The plane is located in a 54. Two very long, straight parallel wires P and Q carry
uniform magnetic field of 0.15 tesla in the vertical di- currents 5 A and 10 A respectively and are at a dis-
rection. For what value of current can the rod remain tance of 10 cm apart, as shown in Fig. If a third wire
stationary ? The mass per unit length of rod is O.3O R (length 10 cm) having a current of 5 A is placed in
kg/m. [Ans. 11.3 A] middle between them, how much force will be acting

m
50. Find the magnitude of the magnetic force on the seg- on R ? The direction of current in all the wires is the
ment CD of length 30 cm placed in a magnetic field of same. [Ans. 10−5 N towards Q]
0.3 T and a current of 4 A flows through it as shown
in Fig.. [Ans. 0.31 N]
ra
G
55. Three long straight parallel wires are kept as shown
u

in Fig.. The wire (3) carries a current I. (i) The di-


51. Fig., shows an equilateral triangular loop CDE, car-
rection of flow of current I in wire (3), is such that
rying current I. Length of each side of trian-
Ed

the net force, on wire (1), due to other two wires,


gle is l. If a uniform magnetic field exists paral-
is zero (ii) By reversing the direction of I, the net
lel to side DE of loop, then find the forces act-
force, on the wire (2) due to the other two wires,
ing on the√three wires CD, DE and EC separately.
√ becomes zero. What will be the directions of cur-
[Ans. IlB 3/2. normally outwards, Zero, IlB 3/2
rent I, in the two cases ? Also obtain the rela-
normally inwards]
tion between the magnitudes of current I1 , I2 and I.
[Ans. (i) Downwards ; I = 212 (ii) upwards; I = I1 ]

FORCE BETWEEN TWO PARALLEL ST.


CONDUCTORS

52. Three long parallel wires AB, CD and EF of


equal resistance are connected as shown in Fig.
The separation between the neighbouring wires is 2 56. A rectangular loop of wire of size 4 cm × 10 cm carries
cm and wires AE and BF are of negligible resis- a steady current of 2 A. A straight long wire carrying
tance. The current is supplied to the wires and 5 A current is kept near the loop 1 cm from ’10 cm’

47 - Nikhil Kamboj — 8193989452 #edugramddun


Moving Charge and Magnetism

arm. If the loop and the wire are coplanar, find (i) the magnetic field to be radial (a) what is the max-
torque acting on the loop and (ii) the magnitude and imum current that can be measured by this gal-
direction of the force on the loop due to the current vanometer if scale can accommodate 45° deflection
carrying wire. [Ans. (i) zero ? (b) What is the smallest current that can be de-
(ii) 16 × 10−6 N] tected if minimum observed deflection is 0.1 degree ?
57. The force between two parallel current carrying con- [Ans. (a) 4.69 x 10−4 A (b) 1.56 X 10−6 A]
ductors is F . If the current in each conductor is dou- 63. The coil of a moving coil galvanometer has an effective
bled, then what will be the new force between them? area 6 x 10−2 m2 . It is suspended in a magnetic field of
[Ans. 4F] 0.03 Wb/m2 . If the torsional constant of the suspen-
TORQUE ON CURRENT LOOP sion fibre is 5 × 10−9 Nm deg−1 , Find its current sensi-
tivity in degree per micro-ampere. [Ans. 0.36 deg/µA]

58. A circular coil of 200 turns and radius 10 cm is 64. To increase the current sensitivity of a moving coil
placed in a uniform magnetic field of 0.5 T, nor- galvanometer by 50% resistance is increased so that
mal to the plane of the coil. If the current in the the new resistance becomes twice its initial resistance.
coil is 3 A, calculate the (a) total torque on the coil By what factor does its voltage sensitivity change?
(b) total force on the coil (c) average force on each [Ans. Decrease by 25%]
electron in the coil, due to the magnetic field. As- 65. A galvanometer needs 30 mV for a full Scale deflec-
sume that the area of cross-section of the wire to tion of 30 divisions. Determine its voltage sensitivity.
be 10−5 m2 and the free electron density is 1029 What must be its resistance if its current sensitivity is
[Ans. (a) zero (b) zero (c) 1.5 × 10−24 N] 2 div/µA? [Ans. 103 div/V ; 2000 Ω]

m
59. A circular coil of 200 turns, radius 5 cm carries a 66. A moving coil meter has the following particulars
current of 2.5 A. It is suspended vertically in a uni- : Number of turns, n = 24 ; Area of coil, A =
form horizontal magnetic field of 0.25 T, with the 2.0x10−3 m2 , magnetic field strength, B = 0.20 T,
plane of the coil making an angle Of 60° with the Resistance of the coil, R = 14 Ω. (a) Indicate
field lines. Calculate the magnitude of the torque
that must be applied on it to prevent it from turn-
ing. [Ans. 0.49 Nm]
ra a simple way to increase the current sensitivity of
the meter by 25%. (It is not easy to change A or
B). (b) If in so doing, the resistance of the coil in-
creases by 7 Ω, is the voltage sensitivity of the mod-
60. A uniform magnetic field of 3000 G is established
G
ified meter greater or lesser than the original meter ?
along the positive Z direction. A rectangular loop
[Ans. (a) n should be increased from 24 to 30 (b)
of sides 10 cm and 5 cm carries a current of 12 A.
Voltage sensitivity is lesser than original]
It is placed in the magnetic field. (a) Calculate the
u
maximum torque the loop can experience. (b) Find CONVERSION OF GALVANOMETER INTO A
the orientation of the loop for which the torque is AND V
zero. (c) In which orientation the loop would be
Ed

(i) in stable equilibrium (ii) unstable equilibrium ? 67. A galvanometer with a coil of resistance 12 Ω shows
[Ans. (a) 1.8 × 10−2 Nm (b) Angle between area full scale deflection for a current 2.5 mA. How will
vector A
⃗ and B ⃗ is 0° or 180° (c) Loop will be in you convert the meter into (i) an ammeter of range
stable equilibrium if area vector A⃗ is parallel to 0 to 7.5 A i a voltmeter of range 0 to 10 V.
magnetic field B⃗ and in unstable equilibrium when A ⃗ [Ans. (a) Shunt resistance 4 x 10−3 Ω (b) series
is antiparallel to B]
⃗ resistance 3988 Ω.]
68. An ammeter gives full scale deflection with a current
61. A coil in the shape of an equilateral triangle of side 0.02
of 1 ampere. It is converted into an ammeter of range
m is suspended from a vertex such that it is hanging
10 ampere. Find the ratio of the resistance of ammeter
in a vertical plane between the pole pieces of a perma-
to the shunt resistance used. [Ans. 9 : 10]
nent magnet producing a horizontal magnetic field of
5x10−2 T. Find the couple acting on the coil. When 69. A galvanometer of resistance 50 Ω gives full deflec-
a current of 0.1 ampere is passed through it and the tion for a current of 0.05 A. Calculate the length of
magnetic field is parallel to its plane. shunt wire required to convert the galvanometer into
an ammeter of range 0 to 5 A. The diameter of the
[Ans. 8.66 × 10−7 Nm]
shunt wire is 2mm and its resistivity is 5 × 10−7 Ω m.
MOVING COIL GALVANOMETER [Ans. 3.174 m in parallel]
70. A galvanometer can be converted into voltmeter of cer-
62. The coil of a galvanometer is 0.02 × 0.8 m2 . It tain range by connecting a resistance of 880 Ω in series
consists of 200 turns of the wire and is in a mag- with it. When the resistance of 420 Ω is connected in
netic field of 0.20 T. The restoring torque constant series, the range becomes half. Find the resistance of
of suspension fibre is l0−6 Nm/degree. Assuming galvanometer. [Ans. 30 Ω]

48 - Nikhil Kamboj — 8193989452 #edugramddun


Moving Charge and Magnetism

71. The resistance of a pivoted type galvanometer is 8 Ω uniformly distributed across the cross-section. Apply
and current for full scale deflection on it is 0.01 A. Ampere’s circuital law to calculate the magnetic field
This galvanometer is to be converted into an amme- at a point ′ r′ in the region for (i) r ¡ a, and (ii) r ¿ a.
ter of 5 A range. The only shunt available is 0.02 Ω. 81. Write the difference between Magnetic field and elec-
Find the value R to be connected in series with the tric field?
galvanometer coil. [Ans. 1.98 Ω]
82. Write the expression, in a vector form, for the
Lorentz magnetic force F⃗ due to a charge mov-
ing with velocity ⃗v in a magnetic field B. ⃗
What is the direction of the magnetic force?
[Ans. F⃗ = q(⃗v × B),
⃗ This force is normal to both the
directions of velocity ⃗v and magnetic field B]

72. The voltmeter V in the Fig. reads 117 V and the am- 83. A charged particle of mass m and charge q travels on
meter A reads O.13 A. The resistance of the voltmeter a circular path of radius ′ r′ , perpendicular to the ex-
is 9000 Ω and the resistance of ammeter is 0.O15 Ω. ternal magnetic field ′ B ′ . Derive the expression for
Compute (i) the resistance R, (ii) the power input to (i) the time taken by particle to complete one revolu-
R [Ans. (i) 1000 Ω (ii) 13.689 watt] tion, (ii) radius of circular path (iii) angular velocity.
2πm
[Ans. ; Equating magnetic force to centripetal
qB
force
mv 2
= qvBsin90◦

m
r
2πr
Time to complete one revolution, T = ]
v
84. Derive an expression for the maximum force experi-
73. In a galvanometer there is a deflection of 10 divisions
enced by a straight conductor of length ′ l′ , carrying
per mA. The internal resistance of the galvanometer
is 60 Ω. If a shunt of 2.5 Ω is connected to the gal-
ra
vanometer and there are 50 divisions in all on the scale
of galvanometer what maximum current can this gal-
vanometer read ? [Ans. 125 mA]
current ′ I ′ and kept in a uniform magnetic field ′ B ′ .
85. Two long straight parallel conductors carry steady cur-
rents I1 and I2 separated by a distance d. If the cur-
rents are flowing in the same direction, show how the
G
74. The maximum current that can be measured by a gal- magnetic field set up in one produces an attractive
vanometer of resistance 40 W is 10 mA. It is converted force on the other. Obtain the expression for this force.
into voltmeter that can read upto 50 V. What is the Hence define one ampere.
u
value of resistance to be connected in the series with 86. Derive an expression for the torque on a rectangular
the galvanometer. [Ans. 4960 W] coil of area A, carrying a current I and placed in a
DESCRIPTIVE QUES. magnetic field B. The angle between the direction of
Ed

B and vector perpendicular to the plane of the coil is


θ. Indicate the direction of the torque acting on the
75. State Bio-Savart’s law. Express it in vector form. loop.
µ0 Idl sin θ
[Ans. dB = ] 87. Define current sensitivity and voltage sensitivity of a
4π r2
76. Write any two important points of similarities and galvanometer. Increasing the current sensitivity may
differences each between Coulomb’s law for the elec- not necessarily increase the voltage sensitivity of a gal-
trostatic field and Biot-Savart’s law for the magnetic vanometer. Justify.
field. 88. Describe the working principle of a moving coil gal-
77. Derive expression for the magnitude of the magnitude vanometer. Why is it necessary to use (i)a radial
of magnetic field at the centre of a circular loop of ra- magnetic field and (ii)a cylindrical soft iron in a gal-
dius r carrying a steady current I. Draw the field lines vanometer?
due to the current loop. 89. Explain how a galvanometer can be converted into an
78. State Ampere’s Circuital law. and prove this law for ammeter of a given range. Derive an expression for
circular path around a long current carrying conduc- the shunt resistance and current for full scale deflec-
tor. tion. Find the effective resistance of ammeter.
79. Derive an expression for the magnetic field produced 90. Explain how will you convert a galvanometer into a
by an infinitely long straight wire carrying a current voltmeter to read a maximum potential difference of
I, at a short distance ’a’ from itself. V volts.
80. A long straight wire of a circular cross-section of ra- 91. Write the methods to increase the sensitivity of a tan-
dius ′ a′ carries a steady current ′ I ′ . The current is gent galvanometer.

49 - Nikhil Kamboj — 8193989452 #edugramddun


Moving Charge and Magnetism

CASE STUDY c) moves in the direction of the field


d) moves perpendicular to the direction of the field
92. Case Study: A charged particle moving in a mag-
netic field experiences a force that is proportional to (ii) A proton is projected with a uniform velocity v
the strength of the magnetic field, the component along the axis of a current carrying solenoid, then
of the velocity that is perpendicular to the magnetic a) the proton will be accelerated along the axis
field, and the charge of the particle. This force is given b) the proton path will be circular about the axis
by F⃗ = q(⃗v × B ⃗ where q is the electric charge of the c) the proton moves along a helical path
particle, ⃗v is the instantaneous velocity of the particle, d) the proton will continue to move with velocity v
and B ⃗ is the magnetic field (in tesla). The direction of along the axis
force is determined by the rules of the cross product
of two vectors Force is perpendicular to both velocity (iii) A charged particle experiences a magnetic force
and magnetic field. Its direction is the same as ⃗v × B ⃗ in the presence of the magnetic field. Which of the
if q is positive and opposite of ⃗v × B ⃗ if q is negative following statement is correct?
The force is always perpendicular to both the velocity a) The particle is stationary and the magnetic field
of the particle and the magnetic field that created it. is perpendicular
Because the magnetic force is always perpendicular to
b) The particle is moving and the magnetic field is
the motion, the magnetic field can do no work on an
perpendicular to the velocity
isolated charge. It can only do work indirectly, via the
c) The particle is stationary and the magnetic field
electric field generated by a changing magnetic field.
is parallel

m
d) The particle is moving and the magnetic field is
parallel to the velocity

(iv) A charge q moves with a velocity 2 m/s along the


ra x-axis in a uniform magnetic field B

a) in z-y plane
c) along +z axis
⃗ = î + 2ĵ + 33̂T
then a charge will experience a force
b) along -y axis
d) along -z axis
G
(i) When a magnetic field is applied to a stationary (v) Moving charge will produce
electron, it
a) electric field only b) magnetic field only
a) remains stationary c) both electric and d) none of these
u
b) spins about its own axis magnetic field
Ed

50 - Nikhil Kamboj — 8193989452 #edugramddun


Magnetism and Matter
Important Formulae and Notes

▶ Magnetic dipole moment (M ) = m × 2l ▶ Totalqinduction of earth’s magnetic field


where m = pole strength of each pole B = BH 2 + B2
v
2l= magnetic length of dipole
where vertical component, Bv = B sin θ
µ
▶ Relative permeability, µr = horizontal component, BH = B cos θ
µ0 Bv Vertical component
where, µ0 = absolute permeability of a free space tanθ = =
BH Horizontal component
▶ Torque acting on a bar magnet placed in a uniform
magnetic field τ = M B sin θ M
▶ Intensity of magnetisation, I =
where, M = magnetic moment V
M = Magnetic dipole moment
B = Magnetic field of induction
V = Volume of magnetic specimen.
▶ Magnetic induction at any point due to a short mag-
netic dipole I
▶ Magnetic susceptibility, χ =
H
µ0 M p where I = intensity of magnetisation
B= 3cos2 θ + 1
4π r3 H = magnetising force
where, M = magnetic dipole moment B
r = distance of point from centre of the dipole ▶ Magnetic permeability, µ =

m
H
θ = angle made by line joining the point from centre
of dipole with axis of dipole ▶ Magnetic induction, B = (µ(H + I)
µ0 = permeability of the free space ▶ Permeability, µ = µ0 (1 + χ)
▶ Magnetic field due to a bar magnet on its axial line and µr = 1 + χ

Baxial =
µ0 2M r
4π (r2 − l2 )2
where r = distance on axial line from the centre of the
ra where µr = relative permeability
χ = magnetic susceptibility
▶ Curie’s law,
G
bar magnet χm T = constant
l = length of the bar magnet
▶ Magnetic field due to a bar magnet on its equatorial where χm = magnetic susceptibility of para magnetic
µ0 M substance
line, Bequitorial =
u
4π (r2 + l2 )2/3 T = absolute temperature

Multiple Choice Questions


Ed

1. The earth behaves as a magnet with magnetic field b) The ferromagnetic property does not depend on
pointing approximately from the geographic temperature.
a) North to South b) South to North c) At high enough temperature ferromagnet be-
c) East to West d) West to East comes a diamagnet.
d) At low temperature ferromagnet becomes a
2. The strength of the earth’s magnetic field is paramagnet.
a) constant everywhere.
b) zero everywhere. 5. The primary origin of magnetism lies in
c) having very high value. a) atomic current and intrinsic spin of electrons.
d) vary from place to place on the earths surface b) polar and non polar nature of molecules.
c) pauli exclusion principle.
3. The vertical
√ component of earth’s magnetic field . at a d) electronegative nature of materials.
place is 3 times the horizontal component the value
of angle of dip at this place is 6. Magnetic moment for solenoid and corresponding bar
a) 30 ◦
b) 45 ◦
c) 60 ◦
d) 90◦ magnet is
a) equal for both b) more for solenoid
4. Which of the following property shows the property of c) more for bar magnet d) none of these
ferromagnetic substances?
a) The ferromagnetic property depends on 7. Which of the following is correct about magnetic
temperature. monopole?
Magnetism and Matter

a) Magnetic monopole exist. 14. According to Curie’s law, the magnetic susceptibility
b) Magnetic monopole does not exist. of a substance at an absolute temperature T is pro-
c) Magnetic monopole have constant value of portional to
monopole momentum. a) 1/T b) T c) 1/T2 d) T2
d) The monopole momentum increase due to in-
crease at its distance from the field. 15. A magnetic dipole moment is a vector quantity di-
rected from:
8. Two identical bar magnets are fixed with their centres
at a distance d apart. A stationary charge Q is placed a) S to N b) N to S
at P in between the gap of the two magnets at a dis- c) E to W d) W to E
tance D from the centre O as shown in the figure. The
16. current carrying power line carries current from west
force on the charge Q is
to east. What will be direction of magnetic field 1
meter above it?
a) N to S b) S to N
c) E to W d) W to E

a) zero 17. On quadrupling the moment of inertia of a magnet, its


b) directed along OP frequency of oscillation will become:
c) directed along PO
a) half b) double
d) directed perpendicular to the plane of paper
c) four times d) one-fourth

m
9. Which of the following is not showing the essential dif-
18. The neutral point in the magnetic field of a horizon-
ference between electrostatic shielding by a conducting
tally placed bar magnet is a point where the magnetic
shell and magneto-static shielding?
field due to that bar magnet is:
a) Electrostatic field lines can end on charges and
conductors have free charges.
b) Magnetic field lines can end but conductors can-
not end them.
c) Lines of magnetic field cannot end on any mate-
ra a)
b)
c)
d)
zero
more than that of earth
less than that of earth
equal to that of earth
G
rial and perfect shielding is not possible. 19. Which of the following has higher magnetic suscepti-
d) Shells of high permeability materials can be used bility?
to divert lines of magnetic field from the interior
a) diamagnetic b) paramagnetic
region.
u
c) ferromagnetic d) None of these
10. The net magnetic flux through any closed surface, kept
20. The magnetic field of earth is due to:
in a magnetic field is
Ed

µ0 a) induction effect of the sun


a) zero b) b) the presence of a large magnet at the centre of

4π the earth
c) d) 4πµ0
µ0 c) interaction of cosmic rays with the current of
earth
11. A magnetic bar of M magnetic moment is placed m
d) motion and distribution of some material in an
the field of magnetic strength B, the torque acting on
outside the earth
it is :
a) M ⃗ .B
⃗ b) −M ⃗ .B
⃗ 21. Which of the following is responsible for the earth’s
c) M ⃗ ×B ⃗ d) B⃗ ×M ⃗ magnetic field?
a) Convective currents in earth’s core
12. The magnetic lines of force inside a bar magnet: b) Diversive current in earth’s core.
a) do not exist c) Rotational motion of earth.
b) depends on area of cross-section of bar magnet d) Translational motion of earth
c) are from N-pole to S-pole of the magnet
d) are from S-pole to N-pole of the magnet. 22. Which of the following independent quantities is not
used to specify the earth’s magnetic field?
13. At which place on the earth, the earth’s magnetic field a) Magnetic declination (j).
becomes horizontal? b) Magnetic dip (d).
a) Magnetic pole b) Geographical pole c) Horizontal component of earth’s field (BH).
c) Magnetic meridian d) Magnetic equator d) Vertical component of earth’s field (BV).

52 - Nikhil Kamboj — 8193989452 #edugramddun


Magnetism and Matter

23. In a bar magnet, magnetic lines of force z : 31. The variation of magnetic susceptibility (q) with ab-
a) are produced only at north pole like rays of light solute temperature (T) for a diamagnetic substance is
from a bulb
b) starts from north pole and ends at the south pole
c) emerge in circular paths from the middle of the
bar
d) run continuously through the bar and outside

24. A wire of length I has a magnetic moment M. It is


then bent into a semi-circular arc. The new magnetic
moment is :
given by
2M M
a) M b) MI c) d) 32. A ferromagnetic substance is placed in the varying
π π
magnetising field H. The magnetic induction B is mea-
25. The radius of curvature of the path of charged particle sure for various values of direct and reverse magnetis-
in a uniform magnetic file is directly proportional to ing field. Following graph has been plotted for B ver-
the sus H. Choose the any wrong statement
a) charge on fie particle b) Momentum of
particle
c) energy of particle d) Strength of field

m
26. The material suitable for making electromagnets
should have
a) There is a limit of direct and reverse external
a)
b)
c)
d)
high retentivity and high coercivity
low retentivity and low coercivity
high retentivity and low coercivity
low retentivity and high coercivity
ra magnetising field at which the magnetisation and
hence the magnetic induction saturates
b) Even after removing the external magnetising
field some residual magnetisation called ‘reten-
G
27. Curie temperature is the temperature above which tivity’ is left over the substance
c) On increasing the reverse magnetising field, the
a) a ferromagnetic material becomes para-magnetic retentivity decreases to zero for a value of mag-
b) a paramagnetic material becomes diamagnetic netising field which is known as ‘susceptibility’
u
c) a ferromagnetic material becomes diamagnetic d) On increasing the reverse magnetising field the
d) a paramagnetic material becomes ferro-magnetic retentivity decreases to zero for a value of mag-
netising field known as ‘coercivity’
Ed

28. The value of angle of dip is zero at the magnetic equa-


tor because on it 33. A small bar of some magnetic material is placed in an
a) V and H are equal external magnetic field. The bar get magnetised such
b) The value of V and H is zero that the magnetic lines of induction so produced have
c) The value of V is zero been shown in the adjoining diagram. The material is
d) The value of H is zero

29. The value of angle of dip is zero at the magnetic equa-


a
tor because on it
a) paramagnetic b) diamagnetic substance
a) V and H are equal
substance
b) The value of V and H is zero
c) ferromagnetic d) non-magnetic
c) The value of V is zero
substance substance
d) The value of H is zero
34. Niddles N1, N2 and N3 are made of a ferromagnetic,
30. A magnet suspended freely at the magnetic pole will a paramagnetic and a diamagnetic substance respec-
show its direction tively.
a) vertical A magnet when brought close to them will
b) horizontal a) attract N1 and N2 strongly, but repel N3
c) 45° from the horizontal b) attract N1 strongly , N2 weakly and repel N3
d) turn in any direction weakly

53 - Nikhil Kamboj — 8193989452 #edugramddun


Magnetism and Matter

c) attract N1 strongly, but repel N2 and N3 weakly is represented by the graph


d) attract all three of them

35. Among which the magnetic susceptibility does not de-


pend on the temperature? .
a) diamagnetism b) paramagnetism
c) ferromagnetism d) ferrite

36. Let the magnetic field on earth be modelled by that


of a point magnetic dipole at the centre of earth. The
angle of dip at a point on the geographical equator is 39. The intensity of magnetisation I is plotted against the
a) always zero magnetising field H for different substances. the curves
b) positive, negative or zero A and B shown in the adjoining figure are associated
c) unbounded
d) always negative

37. The angle of dip at a certain place where the horizon-


tal and vertical components of the earth’s magnetic
field are equal is
a) 30° b) 75° c) 60° d) 45°

m
38. The variation of magnetic susceptibility (q) with ab-
solute temperature (T) for a ferromagnetic substance with

Practice Questions and Numerical

MAGNETIC DIPOLE MOMENT AND FORCE

1. A steel wire of length l has a magnetic moment M .


ra 10 Am. Calculate the magnetic field at distance of 0.2
m from its centre at a point on its (i) axial line (ii)
equatorial line. [Ans. 2.84 × 10−5 T ; 1.14 × 10−5 T ]
G
It is bent into a semicircular arc. What is the new 7. Two small magnets are placed horizontally perpendic-
magnetic moment ? [Ans. 2 M/π] ular to magnetic meridian. Their north poles are at 30
cm east and 20 cm west from a compass needle. Com-
2. Two thin bar magnets of pole strengths 25 Am and
pare the magnetic moments of the magnets, if compass
48 Am respectively and lengths 0.20 m and 0.25 m
u
needle remains undeflected. [Ans. 8 : 27]
respectively are placed at right angles to each other
with the N-pole of first touching the S—pole of the 8. The magnetic field at a point on the magnetic equator
second. Find the magnetic moment of the system. is found to be 3.1×10−5 T. Taking the earth’s radius to
Ed

[Ans. 13 Am2 ] be 6400 km, calculate the magnetic moment of the as-
sumed dipole at earth’s centre. [Ans. 8.1 × 1022 Am2 ]
3. Two identical magnets with a length 10 cm and weight
50 gf each are arranged freely with their like poles fac- 9. A current of 5 A is flowing through a 10 turn
ing in a vertical glass tube. The upper magnet hangs circular coil of radius 7 cm. The coil lies in
in air above the lower one so that the distance be- XY plane. What is the magnitude and direction
tween the nearest poles of the magnets is 3 mm. De- of magnetic dipole moment associated with it ?
termine the pole strength of the poles of these magents. [Ans. 0.77 Am2 , along Z-axis]
[Ans. 6.64 Am] 10. The magnetic dipole moment of earth is 6.4 × 1021
Am2 . If we consider it to be due to a current loop
4. A wire of length 2 metre carries a current 1 ampere,
wound around the magnetic equator of the earth,
is bent to form a circle. Find the magnetic moment of
then what should be the magnitude of the current
the coil. [Ans. 1/p; As, M=IA]
? Take earth to be a sphere of radius 6400 km.
5. A circular coil of ′ N ′ turns and diameter ′ d′ carries a [Ans. 5 × 107 A]
current ′ I ′ . It is unwound and rewound to make an-
11. A short bar magnet has a magnetic moment of 0. 65
other coil of diameter ′ 2d′ , current ′ I ′ remaining the
J/T, then what will be the magnitude and direction
same. Calculate the ratio of the magnetic moments of
of the magnetic field produced by the magnet at a
the new coil and the original coil.
distance 8 cm from the centre of magnet on the axis.
MAGNETIC FIELD STRENGTH DUE TO [Ans. 2.5 × 10−4 T along SN direction]
BAR MAGNET
TORQUE ACTING ON MAGNETIC DIPOLE
AND P.E.
6. A bar magnet of length 10 cm has a pole strength of

54 - Nikhil Kamboj — 8193989452 #edugramddun


Magnetism and Matter

12. A magnetised needle of magnetic moment 4.8 × 10−2 lation to magnetic susceptibility.
J/T is placed at 30° with the direction Of uniform [Ans. Diamagnetic material µr = 1 + Xm]
magnetic field of magnitude 3 × 10−2 T. What is the
24. Relative permeability of a material µr = 400. Identify
torque acting on the needle ? [Ans. 7.2 × 10−4 N-m]
the nature of the magnetic material.
13. A bar magnet placed in a uniform magnetic field of [Ans. Ferromagnetic]
strength 0.3 T with its axis at 30° to the field expe-
25. Relative permeability (µr ) of a material has a value
riences a torque of 0.06 N-m. What is the magnetic
lying 1 ¡ µr ¡ 1 + e (where e is a small quan-
moment of the bar magnet? [Ans. 0.4 Am2 ]
tity). Identify the nature of the magnetic material.
14. A circular coil of 100 tums and having a radius of 0.05 [Ans. Paramagnetic]
m carries a current of 0.1 A. Calculate the work re-
quired to tum the coil in an extenal magnetic field of 26. The susceptibility of a magnetic material is – 2.6 ×
1.5 T through 180° about an axis perpendicular to the 10−5 . Identify the type of magnetic material and state
magnetic field. The plane of the coil is initially at right its two properties. [Ans. diamagnetic]
angles to the magnetic field. [Ans. 0.24 J] 27. The susceptibility of a magnetic material is 2.6×10−5 .
15. A magnetic needle lying parallel to a magnetic field Identify the type of magnetic material and state its two
needs 2 J work to turn it through 60°. Calculate the properties. [Ans. paramagnetic]
torque required to maintain the needle in this position. 28. The relative magnetic permeability of a magnetic ma-
[Ans. 3.46 N-m] terial is 800. Identify the nature of magnetic material

16. A bar magnet with poles 3 cm apart and pole and state its two properties. [Ans. ferromagnetic]
strength 14.4 Am rests with its centre on a friction-

m
DESCRIPTIVE QUES.
less pivot. The magnet is held in equilibrium at an
angle of 60° to a uniform magnetic field of intensity
0.20 T by applying a force F at right angles to its 29. What happens if a bar magnet is cut into two pieces
axis, 0.1 m from its pivot. What is the value of force (i) transverse to its length (ii) along its length?
F ? [Ans. 0.432 N]
MAGNETIC PROPERTIES OF MATERIALS
ra [Ans. (i) magnetic moment reduced to half and
magnetic pole strength remain same
(ii) magnetic moment reduce to half, and magnetic
pole strength reduced to half]
17. A magnet of magnetic moment 2.5 Am2 weighs 66 g. If
G
30. What are magnetic lines of force? Draw the magnetic
density of material of the magnet is 7500 kg/m3 , find
field lines for a bar magnet.
the intensity of magnetisation. [Ans. 2.84 × 105 A/m]
18. Assume that each iron atom has a permanent magnetic 31. Write the four important properties of magnetic field
lines due to bar magnets.
u
moment = 1.85 × 10−23 A/m2 . The number density
of atoms in iron is 8.52 × 1022 m−3 . Find maximum 32. Derive an expression for the intensity of magnetic field
magnetisation and maximum magnetic induction in at an axial point of a short magnetic dipole.
Ed

the iron bar. [Ans. 1.58 × 106 A/m; 1.985 T]


33. Derive an expression for the torque on a magnetic
19. A magnetising field of 1500 A/m produces a flux of Dipole placed in external magnetic field and hence de-
2.4×10−5 weber in a bar of iron of cross-sectional area fine magnetic dipole moment.
0.5 cm2 . Calculate the permeability and susceptibility
of the iron bar used. [Ans. 255 ; 254] 34. State Gauss’s law of magnetism. Hence explain why
magnetic monopole do not exist.
20. An iron rod of 0.2 cm2 area of cross-section
is subjected to a magnetising field of 1200 35. Differentiate Diamagentic, Paramagentic and Ferro-
A/m. If susceptibility of iron is 599, calcu- magnetic Substance.
late (i) permeability (ii) magnetic flux produced. 36. At which place on the earth, the earth’s magnetic field
[Ans. 7.54 ×10−4 T/A m; 1.81 × 10−5 Wb] becomes (i) horizontal to surface (ii) Perpendicular to
21. Calculate the magnetic field intensity at a distance surface?
of 20 cm. from a pole of strength 40 Am in 37. Is the steady electric current the only source of mag-
air. Find the magnetic induction at the same point. netic field? Justify your answer.
[Ans. 79.54 A/m ; 10−4 Wb/m2 ]
38. Where on the surface of Earth is the vertical compo-
22. A circular current loop of magnetic moment M is in
nent of Earth’s magnetic field zero?
an arbitrary orientation in an external magnetic field
B. What is the work done to rotate the loop by 30° 39. Where on the surface of Earth is the horizontal com-
about an axis perpendicular to its plane? [Ans. Zero] ponent of Earth’s magnetic field zero?
23. Relative permeability of a material, µr = 0.5. Identify 40. Where on the surface of Earth is the Earth’s magnetic
the nature of the magnetic material and write its re- field perpendicular to the surface of the Earth?

55 - Nikhil Kamboj — 8193989452 #edugramddun


Magnetism and Matter

41. Define magnetic susceptibility of a material. Name (i) permanent magnets and
two elements, one having positive susceptibility and (ii) electromagnets.
the other having negative susceptibility. What does 44. Out of the two magnetic materials, ‘A’ has relative
negative susceptibility signify? permeability slightly greater than unity while ‘B’ has
42. What is Magnetic Permeability. What can be said less than unity. Identify the nature of the materials
about the value of magnetic permeability for Diamag- ‘A’ and ‘B’. Will their susceptibilities be positive or
netic, paramagnetic and ferro magnetic substances. negative?
43. Write two characteristic properties each to select ma- 45. Write two properties of material suitable for making
terials suitable for (a) permanent magnets (b) an electromagnet.

m
ra
u G
Ed

56 - Nikhil Kamboj — 8193989452 #edugramddun


Electromagnetic Induction
Important Formulae and Notes

▶ Magnetic flux, ϕ = N (B. ⃗ = N BA cos θ


⃗ A) µ0 N 2 A
▶ In case of a long solenoid, L =
where, θ is the angle which normal to the area makes l
with B. where N = total number of turns of the solenoid.
dϕ ▶ ϕ = MI
▶ Induced e.m.f., e = −
dt
▶ e = Blv dϕ dI
▶ e= =M
where v = velocity in a direction perpendicular to B dt dt
l = length of conductor µ0 N1 N2 A
▶ M=
▶ ϕ = LI l
where L = self inductance of the coil. where N1 , N2 = total no. of turns in the two solenoids
dl l = length of longer solenoid
▶ e=L A = πr2 = area of cross section of inner solenoid.
dt
Multiple Choice Questions

1. Whenever the magnetic flux linked with an electric cir- c) the number of turns in the coil of the galvanome-
cuit changes, an emf is induced in the circuit. This is ter are changed

m
called d) None of these
a) electromagnetic induction
7. The polarity of induced emf is given by
b) lenz’s law
c) hysteresis loss a) Ampere’s circuital law
d) kirchhoff’s laws

2. According to Faraday’s law of electromagnetic induc-


tion
ra b)
c)
d)
Biot-Savart law
Lenz’s law
Fleming’s right hand rule

8. The self inductance of a coil is a measure of


G
a) electric field is produced by time varying mag- a) electrical inertia b) electrical friction
netic flux. c) induced e.m.f. d) induced current
b) magnetic field is produced by time varying elec-
u
tric flux. 9. The coils in resistance boxes are made from doubled
c) magnetic field is associated with a moving insulated wire to nullify the effect of
charge. a) heating b) magnetism
Ed

d) None of these c) pressure d) self induced e.m.f.

3. A moving conductor coil produces an induced e.m.f. 10. Two pure inductors each of self inductance L are con-
This is in accordance with nected in series, the net inductance is

a) Lenz’s law b) Faraday’s law a) L b) 2 L c) L/2 d) L/4


c) Coulomb’s law d) Ampere’s law 11. Lenz’s law is a consequence of the law of conservation
of
4. Eddy currents do not cause
a) charge b) mass
a) damping b) heating c) energy d) momentum
c) sparking d) loss of energy
12. A magnet is moved towards a coil first quickly then
5. 1 henry is equal to : slowly, then the induced e.m.f. is
a) weber/ampere b) weber/Volt a) larger in case (i)
c) weber - ampere d) None of these b) smaller in case (i)
c) equal to both the cases
6. A coil of insulated wire is connected to a battery. If d) larger or smaller depending upon the radius of
it is taken to galvanometer, its pointer is deflected, the coil
because 13. When current in a coil changes from 5 A to 2 A in
a) the induced current is produced 0.1 s, average voltage of 50 V is produced. The self-
b) the coil acts like a magnet inductance of the coil is
Electromagnetic Induction

a) 1.67 H b) 6 H a) moving towards the solenoid


c) 3 H d) 0.67 H b) moving into the solenoid
c) at rest inside the solenoid
14. The self inductance associated with a coil is indepen- d) moving out of the solenoid
dent of
21. The steady current flows from A to B is as shown in
a) current b) induced voltage the figure. The direction of the induced current in the
c) time d) resistance of a coil

15. The north pole of a bar magnet is rapidly introduced loop is


into a solenoid at one end (say A). Which of the fol-
a) clockwise.
lowing statements correctly depicts the phenomenon
b) anticlockwise.
taking place?
c) straight line.
a) No induced emf is developed. d) no induced e.m.f. produced.
b) The end A of the solenoid behaves like a south
pole. 22. In electromagnetic induction, the induced charge is in-
c) The end A of the solenoid behaves like north dependent of
pole. a) change of flux b) time
d) The end A of the solenoid acquires positive c) resistance of the coil d) None of these
potential.
23. An induced e.m.f. is produced when a magnet is

m
16. A metal plate can be heated by plunged into a coil. The strength of the induced e.m.f.
is independent of
a) passing either a direct or alternating current
a) the strength of the magnet
through the plate.
b) number of turns of coil
b) placing in a time varying magnetic field.
c) placing in a space varying magnetic field, but
does not vary with time.
d) both (a) and (b) are correct.
ra c)
d)
the resistivity of the wire of the coil
speed with which the magnet is moved

24. In a coil of self-induction 5 H, the rate of change of


current is 2 As−1 . Then emf induced in the coil is
G
17. Identify the wrong statement. a) 10 V b) -10 V c) 5 V d) -5 V
a) Eddy currents are produced in a steady magnetic
field 25. Faraday’s laws are consequence of the conservation of
b) Eddy currents can be minimized by using lami- a) charge b) energy
u

nated core. c) magnetic field d) both (b) and (c)


c) Induction furnace uses eddy current to produce
26. Direction of current induced in a wire moving in a
Ed

heat.
magnetic field is found using
d) Eddy current can be used to produce braking
force in moving trains. a) Fleming’s left hand rule
b) Fleming’s right hand rule
18. A coil having 500 sq. loops of side 10 cm is placed c) Ampere’s rule
normal to magnetic flux which increases at a rate of 1 d) Right hand clasp rule
T/s. The induced emf is 27. Which of the following statements is not correct?
a) 0.1 V b) 0.5 V c) 1 V d) 5 V a) Whenever the amount of magnetic flux linked
with a circuit changes, an emf is induced in
19. A coil of 100 turns carries a current of 5 mA and cre- circuit.
ates a magnetic flux of 10−5 weber. The inductance b) The induced emf lasts so long as the change in
is magnetic flux continues.
a) 0.2 mH b) 2.0 mH c) The direction of induced emf is given by Lenz’s
c) 0.02 mH d) 0.002 H law.
d) Lenz’s law is a consequence of the law of conser-
20. The north pole of a long bar magnet was pushed slowly vation of momentum.
into a short solenoid connected to a short galvanome-
ter. The magnet was held stationary for a few seconds 28. Lenz’s law is a consequence of the law of conservation
with the north pole in the middle of the solenoid and of
then withdrawn rapidly. The maximum deflection of a) charge b) energy
the galvanometer was observed when the magnet was c) induced emf d) induced current

58 - Nikhil Kamboj — 8193989452 #edugramddun


Electromagnetic Induction

29. A solenoid is connected to a battery so that a steady 38. A square of side L metres lies in the x-y plane in a
current flows through it. If an iron core is inserted into region, where the magnetic field is given by B = B0(li
the solenoid, the current will + 3j + 4k) T, where Bo is constant. The magnitude of
a) increase flux passing through the square is [NCERT Exemplar]
b) decrease a) 2BoL2 Wb. b) 3BoL2 Wb.

c) remain same c) 4BoL2 Wb. d) 29 BoL2 Wb.
d) first increase then decrease
39. A loop, made of straight edges has six comers at A(0,
30. In the given figure current from A to B in the straight 0, 0), B(L, 0, 0) C(L, L, 0), D(0, L, 0), E(0, L, L)
wire is decreasing. The direction of induced current in and F(0,0, L). A magnetic field B = Bo (i+k)T is
present in the region. The flux passing through the
loop ABCDEFA (in that order) is [NCERT Exemplar]
the loop is A a) BoL2 Wb. b) 2BoL2 Wb.

a) clockwise b) anticlockwise c) 2BoL2 Wb. d) 4BoL2 Wb.
c) changing d) nothing can be said
40. An e.m.f is produced in a coil, which is not connected
31. Which of the following does not use the application of to an external voltage source. This is not due to
eddy current? a) the coil being in a time varying magnetic field.
a) Electric power meters b) the coil moving in a time varying magnetic field.
b) Induction furnace c) the coil moving in a constant magnetic field.

m
c) LED lights d) the coil is stationary in external spatially vary-
d) Magnetic brakes in trains ing magnetic field, which does not change with
time.
32. If number of turns in primary and secondary coils is
increased to two times each, the mutual inductance
a) becomes 4 times
c) becomes A times
b) becomes 2 times
d) remains unchanged 4
ra 41. There are two coils A and B as shown in Figure. A
current starts flowing in B as shown, when A is moved
towards B and stops when A stops moving. The cur-
rent in A is counterclockwise. B is kept stationary
when A moves. We can infer that [NCERT Exemplar]
G
33. When the rate of change of current is unity, the in-
duced emf is equal to
a) thickness of coil
a) there is a constant current in the clockwise di-
u
b) number of turns in coil
c) coefficient of self inductance rection in A.
d) total flux linked with coil b) there is a varying current in A.
Ed

c) there is no current in A.
34. Two inductors of inductance L each are connected in d) there is a constant current in the counterclock-
series with opposite magnetic fluxes. The resultant wise direction in A.
inductance is (Ignore mutual inductance)
42. Same as question 4 except the coil A is made to rotate
a) zero b) L c) 2L d) 3L about a vertical axis (Figure). No current flows in B
if A is at rest. The current in coil A, when the current
35. If I current is flowing inductance L, then the dimension in B (at t = 0) is counterclockwise and the coil A is
of LI2 is equivalent to: as shown at this instant, t = 0, is [NCERT Exemplar]
a) charge b) force
c) momentum d) energy

36. The phase difference between the flux linkage and the a) constant current clockwise.
emf in a rotating coil in a uniform magnetic field is: b) varying current clockwise.
a) zero b) p/2 c) p/4 d) p c) varying current counterclockwise.
d) constant current counterclockwise.
37. A choke is used as a resistance in :
43. The magnetic flux linked with a coil of N turns of area
a) dc circuits of cross section A held with its plane parallel to the
b) ac circuits field B is
c) both ac and dc circuits N AB N AB
d) neither (a) nor (b) a) b) NAB c) d) zero
2 4

59 - Nikhil Kamboj — 8193989452 #edugramddun


Electromagnetic Induction

44. Two identical coaxial coils P and Q carrying equal a) f ∝ N b) f ∝ c) f ∝ N 2 d) f ∝


amount of current in the same direction are brought N −1 N −2
nearer. The current in
a) P increases while in Q decreases 51. The magnetic flux (ϕ) lined with a coil is related to its
b) Q increases while in P decreases area (s) as:
c) both P and Q increases a) ϕ ∝ s b) ϕ ∝ s2
d) both P and Q decreases c) ϕ ∝ s1/2 d) ϕ ∝ s−1/2

45. A cylindrical bar magnet is rotated about its axis (Fig- 52. If the magnetic flux linked with a coil through which
ure). A wire is connected from the axis and is made to a current off ampere is set up is ϕ, then the coefficient
touch the cylindrical surface through a contact. Then of self-inductance of the coil is:
[NCERT Exemplar] a) l/ϕ b) ϕ/l
c) ϕl d) None of these

53. In a uniform magnetic field B, a wire in the form of a


semicircle of radius r rotated about the diameter of the
circle with angular frequency ‘w’. The axis of rotation
is perpendicular to the field. If the total resistance of
the circuit is R the mean power generated per period
of rotation is :
Bπr2 ω (Bπr2 ω)2

m
a) a direct current flows in the ammeter A. a) b)
b) no current flows through the ammeter A. 2R 8R
(Bπrω)2 (Bπrω 2 )2
c) an alternating sinusoidal current flows through c) d)
the ammeter A with a time period T = 2pw 2R 8R
d) a time varying non-sinosoidal current flows
through the ammeter

46. The magnetic field is parallel to a surface, then the


magnetic flux through the surface is :
ra 54. The role of inductance is equivalent to:
a) inertia
c) energy
b) force
d) momentum
G
55. A metal conductor of length 1 m rotates vertically
a) zero b) small but not zero about one of its ends at angular velocity 5 rad s-1 .
c) infinite d) large but not infinite If the horizontal component of earth’s magnetism is 2
× 10-5 T, then e.m.f. developed between the two ends
47. In the expression e = −dϕ/dt, the -ve sign signifies:
u
of the conductor is:
a) The induced emf is produced only when mag- a) 5 µV b) 50 µV c) 5 mV d) 50 mV
netic flux decreases
Ed

b) The induced emf opposes the change in the mag- 56. The laws of electromagnetic induction have been used
netic flux in the construction of a
c) The induced emf is opposite to the direction of a) galvanometer b) voltmeter
the flux c) electric motor d) generator
d) None of these
57. Two coils are placed closed to each other. The mutual
48. An e– and a p– are moving parallel to each other in a inductance of the pair of coils depends upon
magnetic field. The magnetic force acting on the p+ a) the rate at which currents are changing in the
is: two coils.
a) 1840 times that on e– b) relative position and orientation of two coils.
b) less than that of e– c) the material of the wires of the coils.
c) same as that of e– d) the currents in the two coils.
d) slightly more than that of e–
58. Two identical coaxial circular loops carry a current i
49. A transformer is used to light 100 W and 110 V lamp each circulating in the same direction. If the loops ap-
from a 220 V mains. If the main current is 0.5 A. Then proach each other, you will observe that the current
the efficiency of the transformer is: in
a) 11% b) 50% c) 80% d) 90% a) each increases
b) each decreases
50. The magnetic flux (ϕ) linked with a coil is related to c) each remains the same
the number of turns (N) of the coil as d) one increases whereas that in the other decreases

60 - Nikhil Kamboj — 8193989452 #edugramddun


Electromagnetic Induction

59. Coil of L = 8.4 mH and R = 6 Ω is connected to a 12 V a) T b) Tn−2 c) Wb d) Wb


battery. The current in the coil is 1.0 A at approx’time m−2
of:
61. The magnetic flux linked with a coil is inversely pro-
a) 500 s b) 20 s c) 35 ms d) 1 ms portional to the?
a) magnetic field b) area of cross section
60. The SI unit of magnetic flux is: c) number of turns d) none of these

Multiple Choice Questions

MAGNETIC FLUX AND FARADAY’S LAWS the arm M N of length 20 cm is moved with a veloc-
ity of l0 m/s, calculate the emf induced in the arm.
1. The magnetic field is parallel to a surface, then Find the value of induced current if resistance of arm
what will be the magnetic flux through the surface? M N is 5 Ω and other arms have negligible resistance.
[Ans. zero] [Ans. 1 V, 0.2 A]
2. A coil with an average diameter of 0.02 m is placed
perpendicular to a magnetic field of 6000 T. If the in-
duced e.m.f is 11 V when magnetic field is changed to
1000 T in 4 s, what is the number of turns in the coil.
[Ans. 28]

m
3. An air cored solenoid of length 50 cm and area of cross
section 28 cm2 has 200 turns and carries a current of
5.0 A. On switching off, the current decreases to zero
in 0.1 ms. Find the average e.m.f. induced across the 9. A circular copper disc of l0 cm radius rotates at 20 π
ends of the solenoid. [Ans. 1.4 V]
4. The area of coil of 25 turns is 1.6 cm2 . This coil
is inserted in 0.3 sec in a magnetic field of 1.8
Wb/m2 such that its plane is perpendicular to the
ra radian/sec about an axis through its centre and per-
pendicular to the disc. A uniform magnetic field of 0.2
T acts perpendicular to the disc. Calculate the poten-
tial difference developed between axis of the disc and
G
flux lines of the field. Calculate the e.m.f. induced the rim. What is the induced current if resistance of
in the coil. Also, calculate the total charge that disc is 2 ohm ? [Ans. 0.0629 V ; 0.0314 A]
passes through the wire, if its resistance is 10 Ω. 10. A metre gauge train is running due north with a con-
[Ans. 2.4 × 10−2 V, 7.2 × 10−4 C] stant speed of 90 km/h on a horizontal track. If the
u

5. A uniform magnetic field B exists in a direction per- vertical component of earth’s magnetic field is 3×10−5
pendicular to the plane of a square frame made of Wb/m2 , calculate the e.m.f. induced across the axle
copper wire. The wire has a diameter of 2 mm and of the train of length 1.25 m. [Ans. 9.38 × 104 V]
Ed

a total length of 40 cm. The magnetic field changes 11. A jet plane is travelling west at 450 m/s. If the hor-
with time at a steady rate dB/dt = 0.02 T/s. Find the izontal component of earth’s magnetic field at that
current induced in the frame. Resistivity of copper = place is 4 × 10−4 T and the angle of dip is 30°, find
1.7 × 10−8 ohm-m. [Ans. 9.3 × 10−2 A] the e.m.f. induced between the ends of wings having
6. A wire 88 cm long bent into a circular loop is placed a span of 30 m. [Ans. 3.12 V]
perpendicular to the magnetic field of flux density 2.5 12. An aeroplane is travelling west at the speed of 500
Wb/m2 . Within 0.5 sec, the loop is changed into a m/s. What is the voltage difference between the ends
square of each side 22 cm and flux density is increased of the wings 25 m long, if the earth’s magnetic field at
to 3.0 Wb/m2 . Calculate the value of e.m.f. induced. the location has a magnitude of 5 × 10−5 T and dip
[Ans. 0.018 V] angle is 30◦ ? [Ans. 3.125 V]
7. A coil of 100 loops is pulled in 0.01 s between the 13. When a wheel with metal spokes 1.2 m long rotates
poles of a magnet, where its area intercepts the flux in a magnetic field of flux density 5 × 10−5 T normal
of 300 µ Wb, to a place where it intercepts the flux of to the plane of the wheel, an e.m.f. of 10−2 V is in-
100 µ Wb. Calculate the e.m.f. induced in the coil. duced between the rim and the axle. Find the rate of
[Ans. 2 V] rotation of the wheel. [Ans. 44.2 rps]
MOTIONAL E.M.F. 14. A horizontal straight wire of length L extending from
east to west is falling with speed v at right angles to
8. A rectangular conductor LMNO is placed in a uniform the horizontal component of Earth’s magnetic field B.
magnetic field of O/5 T, Fig.. The field is directed (a) Write the expression for the instantaneous value of
perpendicular to the plane of the conductor. When the emf induced in the wire.

61 - Nikhil Kamboj — 8193989452 #edugramddun


Electromagnetic Induction

(b) What is the direction of the emf? 26. A coil of 100 turns carries a current of 5 mA and cre-
(c) Which end of the wire is at the higher potential? ates a magnetic flux of 10−5 weber. What will be the
[Ans. e = BLv; west to east; East] value of inductance? [Ans. 0.02 mH; LI=Nϕ]
15. Two identical loops, one of copper and the other of alu- 27. In a coil of self-induction 5 H, the rate of change of
minium, are rotated with the same angular speed in current is 2 As−1 . What will be emf induced in the
the same magnetic field. Compare (i) the induced emf coil? [Ans. -10V]
and (ii) the current produced in the two coils. Justify MUTUAL INDUCTION
your answer.
[Ans. (i) e = N BAwsinwt in both; (ii) I = e/R =
eA/rl, more induced current in copper] 28. Over a solenoid of 50 cm length and 2 cm radius hav-
ing 500 turns, is wound another wire of 50 turns near
16. A metal conductor of length 1 m rotates vertically the centre. Calculate mutual inductance of the two
about one of its ends at angular velocity 5 rad/s . coils. If current in primary changes from 0 to 5 A in
If the horizontal component of earth’s magnetism is 2 0.02 s, what is the emf induced in secondary Coi1 ?
× 10−5 T, then what is the e.m.f. developed between [Ans. 7.396 × 10−5 H; 19.74 mV]
the two ends of the conductor? [Ans. 5 µV]
29. An air cored solenoid is of length 0.3 m, area of cross
17. A coil having 500 square loops of side 10 cm is placed secfion 1.2 × 10−3 m2 and has 2500 turns. Around
normal to magnetic flux which increases at a rate of 1 its central section, a coil of 350 turns is wound. The
T/s. What will be the induced emf? solenoid and the coil are electrically insulated from
SELF INDUCTION each other. Calculate the emf induced in the coil if
the initial current of 3 A in the solenoid is reversed in

m
18. What is self induction of a coil, in which magnetic flux 0.25 s. [Ans. 0.106 V]
of 40 mili weber is produced when 2 A current flows 30. Find the mutual inductance between the two coils if
through it? [Ans. 2 × 10−2 Wb] current of 10 ampere in the primary coil changes the
flux by 500 Wb per turn in the secondary col of 200
19. An inductor of 5 H inductance carries a steady
current of 2 A. How can a 50 V self induc-
tance e.m.f. be made to appear in the inductor?
[Ans. In 0.2 s, current is reduced to zero]
ra turns. Also find the induced emf across the ends of
the secondary coil if this cahnge occurs in 0.5 sec.
[Ans. 104 H, 2 × 105 Volt]
31. Two solenoids A and B spaced close to each other and
20. The current in a solenoid of 240 turns, having a length
G
of 12 cm and radius of 2 cm, changes at the rate of 0.8 sharing the same cylindrical axis have 400 and 700
A/s. Find the emf induced in it. [Ans. 6 × 10−4 V] turns respectively. A current of 3.5 A in coil A pro-
duced an average flux of 90 µT/m2 through each turns
21. Magnetic flux of 5 microweber is linked with a coil of A and a fulx of 90µT/m2 through each turns of B.
u
when a current of 1 mA flows through it. What is self Calculate
inductance of the coil? [Ans. 5 mH] (a) mutual inductance of two solenoids.
22. The self inductance of an inductance coil having 100 (b) the self inductance of A. What emf is induced in
Ed

turns is 20 mH. Calculate the magentic flux through B when the current in A increases at the rate of 0.5
the cross section of the coil corresponding to a cur- A/s. [Ans. 1.8 × 10−2 H; 3.43 × 10−2 H; 9 × 10−3 V]
rent of 4 miliampere. Also find the total flux. 32. Two identical coaxial circular loops carry a current i
[Ans. 8 × 10−5 Wb; 8 × 10−3 Wb] each circulating in the same direction. If the loops ap-
23. Three inductances are connected as shown in fig.. Cal- proach each other, what will you observe in the value
culate the resultant inductance. [Ans. 1.0 H] of current? [Ans. each decreases]
ELECTRIC GENERATOR

33. An AC generator consists of a coil of 100 turns and


cross sectional area of 3m2 , rotating at a constant
angular speed of 60 rad/sec in a uniform magnetic
24. What is the magnetic flux through each turn of field of O.04 T. The resistance of the coil is 500 Ω.
solenoid of self inductance 8 × 10−5 H, when a current Calculate (i) maximum current drawn from the gen-
of 3.0 A flows through it? Assume that the solenoid erator and (ii) max. power dissipation in the coil.
has 1000 turns and is wound from wire of diameter l.0 [Ans. 1.44 A ; 518.4 W]
mm. What is the cross sectional area of the solenoid? 34. An AC generator consists ofa coil of 2000 turns each
[Ans. 2.4 × 10−7 W b; 6.37 × 10−5 m2 ] of area 80 cm2 and rotating at an angular speed of
25. When current in a coil changes from 5 A to 2 A in 200 rpm in a uniform magnetic field of 4.8 × 10−2 T.
0.1 s, average voltage of 50 V is produced. What is Calculate the peak and rms values of e.m.f induced in
self-inductance of the coil? [Ans. 1.67 H] the coil. [Ans. 16.1 V, 11.4 V]

62 - Nikhil Kamboj — 8193989452 #edugramddun


Electromagnetic Induction

35. A flat coil of 500 turns each of area 50 cm2 rotates in the solenoid, will the current increase or decrease? Ex-
a uniform magnetic field of 0.14 Wb/m2 at an angular plain.
speed of 150 rad/sec. The coil has a resistance of 5 Ω. 43. 11 kW of electric power can be transmitted to a distant
The induced e.m.f. is applied to an external resistance station at (i) 220 V or (ii) 22,000 V. Which of the two
of 10 ohm. Calculate the peak current through the modes of transmission should be preferred and why?
resistance. [Ans. 3.5 A] Support your answer with possible calculations.
36. A coil has 50 tums and its area is 500 cm2 . It is rotat- 44. What is induced emf? Write Faraday’s laws of elec-
ing at the rate of 50 r.p.s. at right angles to magnetic tromagnetic induction. Express it mathematically.
field of 0.5 wb/m2 . calculate the maximum value of
45. How is the mutual inductance of a pair of coils affected
electromotive force developed across the ends of the
when:
coil. [Ans. 392.8 Volt]
(a) the separation between the coils is increased?
SUBJECTIVE QUES (b) the number of turns of each coil is increased?
(c) a thin iron sheet is placed between the two coils,
37. Define self-inductance of a coil. Write its S.I. unit. other factors remaining the same? Explain your an-
swer in each case.
38. Name and define the unit used for measuring the co-
46. Derive an expression for the emf induced across the
efficient of mutual inductance. State the relation of
ends of a straight conductor of length ’L’ moving at
this unit with the units of magnetic flux and electric
right angles to a uniform magnetic field ’B’ with a
current.
uniform speedy.
39. State Lenz’s Law. A metallic rod held horizontally

m
47. A magnet is moved towards a coil first quickly then
along the east-west direction is allowed to fall under slowly, In which case the induced emf is larger?
gravity. Will there be an emf induced at its ends?
Justify your answer. 48. Write the factors on which self inductance of a coil
depends.
40. Explain by giving examples that Lenz’s law is a con-
sequence of the law of conservation of energy.
41. Define mutual inductance. A pair of adjacent coils has
a mutual inductance of 1.5 H. If the current in one coil
ra 49. A solenoid is connected to a battery so that a steady
current flows through it. If an iron core is inserted
into the solenoid, How will the induced current be ef-
fected?
changes from 0 to 20 A in 0.5 s, what is the change of
G
50. If number of turns in primary and secondary coils is
flux linkage with the other coil?
increased to two times each. What will be new mutual
42. A solenoid is connected to a battery so that a steady inductance?
current flows through it. If an iron core is inserted into
u
Ed

63 - Nikhil Kamboj — 8193989452 #edugramddun


Alternating Current
Important Formulae and Notes

▶ During growth of current in RL circuit (c)


I = I0 [1 − e−(R/L)r] Erms Erms
Irms = =p
▶ Mean Value of Alternating current Z R2 + XL2
▶ A.C. Circuit containing R and C only
2I0
(Im )half cycle =± = ±0.637I0 (a) Alternating current in R − C circuit leads the ap-
π
plied alternating emf by a phase angle ϕ, where
(Im )full cycle = 0 XC 1
tan ϕ = =
R ωCR
▶ Root mean square Value of Alternating EMF
(b) Impedance of RC circuit
E0
q
Erms = √ = 0.707E0 Z = R2 + XC2 = R2 + (1/ωC)2
p
2
(c)
▶ Root mean square Value of Alternating Current Erms Erms
Irms = =p
Z R2 + XC2
I0
Irms = √ = 0.707I0 ▶ A.C. Circuit containing R, L, and C

m
2
(a) Impedance of RLC circuit, Z =
R + (XL − XC )
p
2 2
Erms
▶ Irms = 1
R where XL = ωL and XC =
where R = Effective resistance to AC
▶ If NO phase change is introduced, i.e., if E =
E0 sin ωt, then I = I0 sin ωt
▶ A.C. circuit containing Inductor only
ra (b) Irms =
Erms
Z
ωC

(c) Phase angle ϕ by which alternating current


lags behind the alternating voltage is, tanϕ =
G
Xl − XC
(a) If, E = E0 sin ωt ; than I = I0 sin (ωt − 900 )
R
AC through an inductor lags behind the alt. emf
▶ At resonance, XL = XC ,
by phase angel of 902 1 1
ωL = or ω = √
(b) Inductive reactance, XL = ωL = 2πνL
u
ωC LC
Erms ω 1
∴ Irms = ∴ Resonance frequency ν = = √
XL 2π 2π LC
Ed

(c) Average power consumed/cycle = Erms Irms cos900 = ▶ At series resonance frequency, Z = R, Irms =
Zero. Erms
=max.
▶ A.C. Circuit containing Capacitance only R
▶ At parallel resonance frequency, Irms = 0
(a) If E = E0 sin ωt r
1 L
then I = I0 sin (ωt + 900 ) ▶ Q factor of resonance ; Q =
1 1 R C
(b) Capacitative reactance, XC = = ▶ Energy stored in capacitor,
ωC 2πνC
Erms Q2 1 1
Irms = UC = = CV 2 = QV
XC 2C 2 2
(c) Average power/cycle consumed by capacitor = 1
▶ Energy stored in inductor, UL = LI 2
Erms Irms cos900 = Zero 2
▶ Average power consumed/cycle in pure inductor is
▶ A.c. circuit containing R and L only Zero.
(a) Alternating current in RL circuit lags behind the ▶ Average power consumed/cycle in an ideal capacitor is
applied alternating e.m.f. by a phase angle ϕ, zero.
where
XL ωL ▶ The natural frequency of energy oscillations in LC cir-
tan π = = 1
R R cuit is ν = √
2π LC
(b) Impedance of RL circuit ▶ Average power dissipated/cycle in a non-inductive cir-
q cuit (XL = XC )
Z = R2 + XL2 = R2 + (ωL)2
p
P = Erms Irms
Alternating Current

▶ Average power dissipated/cycle in RLC circuit P = (a) Efficiency of transformer,


Erms Irms cosϕ output power Es I − s
η= =
true power R Input power Ep Ip
▶ Power factor, cos ϕ = = =
apparant power Z (b) When efficiency of transformer is 100 percent
R Es ns Ip
= = = K, transformer ratio.
R2 + (XL − XC )2
p
Ep np Is
▶ Transformer

Multiple Choice Questions

1. If the frequency of an A.C. is made 4 times of its initial 10. The heat produced in a given resistance in a given time
value, the inductive reactance will by the sinusoidal current I0 sinwt will be the same as
a) be 4 times b) be 2 times that of a steady current of magnitude nearly
c) be half d) remain the same a) 0.71 I0 b) 1.412 I0
p
c) I0 d) I0
2. In an L.C.R. series AC circuit, the current
11. Alternating currents can be produced by
a) is always in phase with the voltage
b) always lags the generator voltage a) dynamo b) choke coil
c) always leads the generator voltage c) transformer d) electric motor
d) None of these

m
12. With increase in frequency of an A.C. supply, the in-
ductive reactance
3. A capacitor acts as an infinite resistance for
a) decreases
a) DC b) AC b) increases directly with frequency
c) DC as well as AC d) neither AC nor DC

4. Phase difference between voltage and current in a ca-


pacitor only AC circuit is
a) π b) π/2 c) 0 d) π/3
ra c)
d)
increases as square of frequency
decreases inversely with frequency

13. Of the following about capacitive reactance which is


correct?
G
a) The reactance of the capacitor is directly pro-
5. An LCR series circuit, connected to a source E, is at portional to its ability to store charge
resonance. Then the voltage across b) Capacitive reactance is inversely proportional to
the frequency of the current
u
a) R is zero
b) R equals applied voltage c) Capacitive reactance is inversely proportional to
c) C is zero the resistance
Ed

d) L equals applied voltage d) Capacitive reactance is independent to the fre-


quency of the current
6. At resonance frequency the impedance in series LCR 14. Current in a circuit is wattless if
circuit is
a) inductance in the circuit is zero
a) maximum b) minimum b) resistance in the circuit is zero
c) zero d) infinity c) current is alternating
d) resistance and inductance both are zero
7. A capacitor has capacitance C and reactance X, if
capacitance and frequency become double, then reac- 15. Power factor is one for
tance will be a) pure inductor
a) 4X b) X/2 c) X/4 d) 2X b) pure capacitor
c) pure resistor
8. In LCR circuit if resistance increases quality factor d) either an inductor or a capacitor.
a) increases finitely b) decreases finitely 16. An A.C. source is connected to a resistive circuit.
c) remains constant d) None of these Which of the following is true?
a) Current leads ahead of voltage in phase
9. The peak value of the AC current flowing throw a re-
b) Current lags behind voltage in phase
sistor is given by
c) Current and voltage are in same phase
a) I0 = e0 /R b) I = e/R d) Any of the above may be true depending upon
c) I0 = e0 d) I0 = R/e0 the value of resistance

65 - Nikhil Kamboj — 8193989452 #edugramddun


Alternating Current

17. In which of the following circuits the maximum power 26. The impedance of a series LCR circuit is
dissipation is observed? s
1 1
a) Pure capacitive circuit a) R + XL + XC b) 2 + 2 + R2
XC XC
b) Pure inductive circuit q
c) XL − XC + R2 d) R2 + (XL − XC )2
p
c) Pure resistive circuit 2 2

d) None of these
27. The voltage across a resistor, and inductor and a ca-
18. When an ac voltage of 220 V is applied to the capacitor pacitor connected in series to an AC source are 20 V,
C, then 15 V, 30 V respectively. The resultant voltage in the
a) the maximum voltage between plates is 220 V circuit is
b) the current is in phase with the applied voltage a) 5 V b) 20 V c) 25 V d) 65 V
c) the charge on the plate is not in phase with the
applied voltage 28. When an alternating voltage E = E0 sin ωt is ap-
d) power delivered to the capacitor per cycle is zero plied to a circuit, a current I = I0 sin(ωt + π/2) flows
through it. The average power dissipated in the circuit
19. In general in an alternating current circuit is
a) the average value of current is zero a) Er ms × Ir ms b) E0 I0
b) the average value of square of the current is zero E0 I0
c) √ d) Zero
c) average power dissipation is zero 2
d) the phase difference between voltage and current

m
is zero 29. Choose the quantity whose SI unit is not ohm
a) Resistance b) Reactance
20. The graph between inductive reactance and frequency c) Capacitance d) Impedance
is
a) parabola
c) hyperbola
b) straight line
d) an arc of a circle

21. For minimum dissipation of energy in the circuit the


ra 30. The peak value of 220 V AC is
a) 220 V
220
b) √ V
2
c) 440 V

d) 220 2
V
G
power factor should be 31. Which of the following does not have the dimension of
a) large b) small time?
c) moderate d) can not say L R √
a) RC b) c) d) LC
R L
u
22. A bulb connected in series with a solenoid is lit by
an AC source. If a soft iron core is introduced in the 32. The resistance of a coil for direct current is 10 ohm.
solenoid then, When AC is sent through the same coil, its resistance
Ed

would be
a) the bulb will glow brighter
b) the bulb will glow dimmer a) 10 Ω b) >10 Ω
c) no effect on the light produced c) <10 Ω d) cannot say
d) bulb may glow more brighter or dimmer
33. The average value of AC voltage E = E0 sin ωt over
the time interval t = 0 to t = π/ω
23. Power factor of an ac circuit is a measure of:
a) −2E0 /π b) E0 /π
a) virtual power
2E0
b) power lost in the circuit c) d) Zero
c) mean power π
d) all the above 34. The alternating current from a source is represented
by I = 0.5 sin 314t. The frequency of AC is
24. The dimensional formula of L/R is similar to that of:
a) 314 Hz b) 100 Hz c) 50 Hz d) Zero
a) frequency b) time
c) length d) none of these 35. Q factor of resonance is given by
r r r r
1 L 1 C 1 R 1 L
25. In a circuit, the phase difference between the alternat- a) b) c) d)
ing current and the source voltage is π/3. Which of R C R L L C C R
the following cannot be the elements of the circuits? 36. The power factor of an AC circuit is given by cosϕ =
a) only C b) only L R Z R R
c) Only R d) All the above a) b) c) d)
Z R XL XC

66 - Nikhil Kamboj — 8193989452 #edugramddun


Alternating Current

37. When AC source is connected across series RLC com- 46. An iron core transformer with a turns ratio of 8 : 1 has
bination, maximum power loss will occur provided 120 V applied across the primary. The voltage across
a) current and voltage are in phase the secondary is
b) current from source is minimum a) 15 V b) 120 V c) 180 V d) 960 V
c) inductance is minimum
d) capacitance is maximum 47. A.C. power is transmitted from a power house at a
high voltage as
38. In RLC series AC circuit, Impedance cannot be in- a) the rate of transmission is faster at high voltages
creased by b) it is more economical due to less power loss
c) power cannot be transmitted at low voltages
a) increasing frequency of source
d) a precaution against theft of transmission lines
b) decreasing frequency of source
c) increasing the resistance 48. High Voltage Transmission line is preferred as
d) increasing the voltage of the source
a) Its appliances are costly
b) Thin power cables are required
39. In electric sub-station in township, large capacitor
c) Idle current very low
banks are used
d) Power loss is very less
a) to reduce power factor
b) to improve power factor 49. The battery of 12 V is connected to primary of a trans-
c) to decrease current former with turns ration ns /np = 10. Voltage across
d) to increase current in the circuit secondary would be

m
a) 120 V b) 1.2 V c) 12 V d) Zero
40. The capacitor of capacitance C has reactance X. If
capacitance and frequency become double, then the Es ns
50. The relation = is applicable only to
capacitative reactance will be Ep np
a) 2X b) 4X c) X/2 d) X/4

41. In series RLC circuit, Quality factor can be imporved


by
ra a)
b)
c)
d)
AC generator
DC generator
induction coil
Step up/step down tranformer
G
a) decreasing L b) Increasing C 51. Out of the following, choose the wrong statement :
c) decreasing R d) decreasing R and L
a) A transformer cannot work on dc
b) A transformer cannot change the frequecy of AC
u
42. The transformer voltage induced in the secondary coil
c) A transformer can produce AC power
of a transformer is mainly due to
d) In a transformer, when AC voltage is raised n
a) a varying electric field times, the AC reduces to 1/n time
Ed

b) a varying magnetic field


c) the vibrations of the primary coil 52. The core of a transformer is laminated, so as to
d) the iron core of the transformer a) make it light weight
b) make it robust and strong
43. Quantity that remains unchanged in a transformer is c) increase the secondary voltage
a) voltage b) current d) reduce energy loss due to eddy current
c) frequency d) None of these 53. In a transformer the no of turns of primary and sec-
ondary coil are 500 and 400 respectively. If 220 V ac
44. Transformers are used is supplied to the primary coil, the ratio of currents in
a) in DC circuit only primary and secondary coils is
b) in AC circuits only a) 4:5 b) 5:4 c) 5:9 d) 9:5
c) in both DC and AC circuits
d) neither in DC nor in AC circuits 54. Which one of the following statements is true?
a) An inductor has infinite resistance in d.c. circuit
45. The transformation ratio in the step-up transformer is b) An inductor and a capacitor both cannot con-
a) one duct in d.c. circuit
b) greater than one c) A capacitor can conduct in a d.c. circuit but not
c) less than one an inductor
d) the ratio greater or less than one depends on the d) An inductor can conduct in a d.c. circuit but
other factor not a capacitor.

67 - Nikhil Kamboj — 8193989452 #edugramddun


Alternating Current

55. In a transformer having 100% efficiency, the input ing all power losses, the input power must be
power is 60 W. The number of turns of primary coils
a) P b) 2P/3 c) P/2 d) 3P/2
is 100 and the number of turns of secondary coils is
300. Then the output power is:
57. A transformer has an efficiency of 80%. It works at 4
a) 60 W b) 120 W c) 180 W d) 240 W kW, 100 V. If secondary voltage is 240 V, the current
in primary coil is :
56. The ratio of the secondary to the primary turns in a
transformer is 3: 2 and the output power is P. Neglect- a) 10 A b) 4 A c) 0.4 A d) 40 A

Practice Questions and Numerical

AC CIRCUIT CONTAINING RESISTANCE henry. Write equation for instantaneous current


ONLY through the circuit. What will be the reading of
a.c. galvanometer connected in the circuit ? .
1. An alternating voltage given by E = 280sin50πt is [Ans. I = 2.2 sin (100 πt − π/2; 1.555 A]
connected across a pure resistor of 40 ohm. Find fre-
quency of source and rms current through the bulb. 11. An inductance of negligible resistance, whose reac-
[Ans. 25 Hz, 4.95 A] tance is 22 ohm at 200 Hz is connected to a 220 V,
50 Hz powerline. What is the value of inductance and
2. A light bulb is rated 200 W for 220 V supply at 50
reactance ? [Ans. 0.0175 H ; 5.5 Ω]
Hz. Calculate resistance of the bulb and rms current
through the bulb. [Ans. 242 Ω; 0.91 A] 12. An inductor of unknown value, a capacitor of 100 µF

m
and a resistor of 10 ohms are connected in series to
3. A resistance of 20 Ω is connected to a source of al-
a 200 V. 50 Hz a.c. source. It is found that the
ternating current rated 110 V, 50 Hz. Find (a) the
power factor of the circuit is unity. Calculate the in-
rms current, (b) the maximum instantaneous current
ductance of the inductor and the current amplitude.
in the resistor and (C) the time taken by the current
ra [Ans. 101.3 mH; I0 = 28.28A]
to change from its maximum value to the rms value.
[Ans. 5.5 A; 7.8 A; 2.5 ×10−3 s] AC CIRCUIT CONTAINING CAPACITOR
ONLY
4. A 100 ohm iron is connected to a 110 V 60 hertz
wall plus What is (i) peak pot. diff. (ii) average po- 13. A capacitor of capacitance 10 p.F is con-
G
tential diff. over a half cycle and (iii) rms current? nected to an oscillator giving an output volt-
[Ans. (i) 141.44 V (ii) 14.144 A (iii) 2000.5 W] age E = 10 sin Ωt volt. If Ω = 10 rad/s,
5. A sinusoidal voltage V = 200 sin 314 t, is applied to a find the peak current and instantaneous current.
resistor of 10 ohm. Calculate (i) rms value of voltage [Ans. 1.0 mA; I= 10−3 sin(Ωt + π/2) J]
u

(ii) rms current and (iii) power dissipated as heat.


14. A 10 µF capacitor is connected with 1 henry induc-
[Ans. (i) 141.44 v (ii) 14.144 A (iii) 2000.5 W]
tance in series with a 50 hertz source of a.c. Calculate
Ed

6. A heating element is marked 210 V, 630 W. Find the the impedance of the combination. [Ans. 4.0 Ω]
resistance of the element when connected to a 210 V
15. A 3 µF capacitor is connected to a 220 V. 50 Hz a.c.
dc source. [Ans. 70 ohms]
source. Calculate the rms value of current through
AC CIRCUIT CONTAINING INDUCTANCE the circuit. Also, find the peak value of voltage across
ONLY the capacitor. [Ans. 0.207 A ; 311.1 V]
7. An inductor of inductance 200 mH is connected to 16. A capacitor of 10µF is connected to an a.c. source of
an a.c. source of peak emf 210 V and frequency 50 e.m.f E = 220 sin 100 πt. Write the equation of in-
Hz. Calculate the peak current. What is the instan- stantaneous current through the circuit. What will be
taneous value of voltage when current is at its peak the reading of a.c. ammeter connected in the circuit ?
value? [Ans. 3.3 A; Zero] [Ans. I = 0.691 sin (10 πt + π/2; 0.489 A]
8. A pure inductor of self inductance 1 H is connected 17. A current of 30 mA is taken by a 4 µF capacitor con-
across an alternating voltage of 115 V and frequency nected across an alternating current line having fre-
60 Hz. Calculate the (i) inductive reactance (ii) ef- quency of 500 Hz. Calculate reactance of the capacitor
fective current (iii) peak current (iv) average power and voltage across the capacitor. [Ans. 79.6 Ω ; 2.4 V]
consumed. [Ans. 377.1 Ω, 0.3 A, 0.43 A, Zero] 18. A 30 µF capacitor is connected to a 220 V. 50 Hz
9. How much inductance should be connected to 200 V, source. Find its capacitative reactance, rms cur-
S0 c/s supply so that a maximum current of 0.5 A rent, peak current and impendance of the circuit.
flows through it ? [Ans. 1.79 H] [Ans. 106.1 Ω; 2.07 A; 2.93 A; 106.1 Ω]
10. Alternating e.m.f of E = 220 sin 100 πt is ap- 19. A 15.0 µF capacitor is connected to
plied to a circuit containing an inductance of (1/π) 220 V, 50 Hz source. Find the ca-

68 - Nikhil Kamboj — 8193989452 #edugramddun


Alternating Current

pacitive reactance and the rms current. and Y ? (ii) When X and Y are connected in series
1 across the same source, what will be the current ?
[Ans. XC = = 212.33 ohms; Irms = 1.04 A]
2πf c [Ans. X is resistor, Y is capacitor ; 0.353 A]
AC CIRCUIT CONTAINING R AND L 29. Find the capacity of a condenser to run a 30 volt,
10 watt lamp when connected in series with an al-
20. A 12 ohm resistance and an inductance of 0.05/π henry ternating e.m.f. of 220 volt and frequency 50 c/s.
are connected in series. Across the ends of the circuit [Ans. 4.86 µF ]
is connected a 130 V a.c. supply of 50 Hz. Calculate 30. What will be the impedance of the adjoining circuit,
current in the circuit. What is phase diff. between Fig. for (i) direct current source, (ii) an a.c. √source of
current and voltage ? [Ans. 10 A; 22.6°] frequency 25/π kilohertz? [Ans. (i) ∞(ii)10 5Ω]
21. The impedance of a coil is 141.4 ohm at 50 hertz and
its resistance is 100 ohm. What will be its inductance
? [Ans. 0.318 H]
22. When 100 volt d.c. is applied across an inductor, a
current of 1 A flows through it. If the same inductor
31. A circuit consists of a 2 µF capacitor and a resis-
is connected to 100 V a.c. source, the current reduces
tor of 1 KΩ. An a.c. source of l2 V, 50 hz is con-
to 0.5 A. Why is the current reduced in the latter case
nected acrosss the circuit. Calculate (i) current flow-
? Calculate the value of the reactance of the inductor?
ing (ii) voltage across capacitor (iii) Phase angle be-
[Ans. 173.2 Ω]
tween voltage and current (iv) average power supplied.

m
23. When a series combination of inductance and resis- [Ans. 6.4 × 10−3 A; 10.2 V; 58◦ , 0.041 W]
tance are connected with a 10 V, 50 Hz a.c. source,
32. When a circuit element
√ X is connected across
√ an a.c.
a current of l A flows in the circuit. The voltage
source of e.m.f 220 2 volt, current of 2 A flows
leads the current by a phase angle of π/3 radian.
through it in phase with the voltage. When another el-
Calculate the values of resistance and inductance.
[Ans. 5 Ω, 27.57 mH]
24. A 80 V 800 W henter is to be operated on a 100 V-
50 Hz a.c. supp—y. Calculate the inductance of the
ra ement Y is connected across the same a.c. source, the
same current flows through the circuit, but it leads the
voltage by π/2 radian. Name the circuit elements X
and Y . Find the current that flows in the circuit when
choke required. [Ans. 0.019 H]
G
series combination of X and Y is connected across the
25. The inductance of a resistance coil is 0.5 henry. same source. [Ans. X-Resistance, Y-capacitor; 1 A]
How much potential difference will be developed AC CIRCUIT CONTAINING R, L, AND C
across it on passing an alternating current of 0.2
u
amp. if the frequency of current be 50 hertz
33. A 50 µF capacitor, 0.05 H inductor and 48 Ω re-
? What will be the phase difference between
sistor are connected in series with an a.c. source
the potential difference and current in the coil?
Ed

of emf E = 310 sin 314t. Calculate reactance


[Ans. 31.4 volt, voltage leads current by 90°]
of the circuit; What is its nature ? What is
26. A coil of inductance 0.5 H and resistance 100 Ω is phase angle between current and applied e.m.f ?
connected to 200 V, 50 Hz a.c. supply. Find the max- [Ans. 48 Ω, capacitative; 45°, current leads]
imum current in the coil Also, find the time lag be-
34. A 25.0 µF capacitor, a 0.10 henry inductor and a 25.O
tween the maximum voltage and maximum current.
ohm resistor are connected in series with an a.c. source
[Ans. 1.52 A; 3.194 ×10−3 s]
whose e.m.f. is given by E = 310 sin 314 t.
AC CIRCUIT CONTAINING R AND C ONLY (i) what is the frequency of the e.m.f. ? (ii)
calculate (a) the reactance of the circuit (b) the
27. In a series RC circuit, R = 30 ohm, C = 0.25 µF V = impedance of the circuit, and (c) current in the circuit.
100 V, Ω = 10000 rad/s. Find the current in the cir- [Ans. (i) 50 Hz (ii) (a) 95.84 Ω (b) 99.1 Ω (c) 2.21 A]
cuit and calculate the voltage across the resistor and 35. A 40 ohm resistor, 3 mH inductor and 2 µF capaci-
capacitor. Is the algebraic sum of these voltages more tor arc connected in series to a 110 V, 5000 Hz a.c.
than the source voltage? If Yes, resolve the paradox. source. Calculate the value of current in the circuit.
[Ans. 0.25 A ; 7.5 V, 100 V, Yes] [Ans. 1.25 A]
28. When an electric device X is connected to a 220 volt, 36. A series LCR circuit having L = 10 mH, C =
50 hertz a.c. supply, the current is 0.5 amp, and is (400/π 2 µF ) and R = 55 ohm is connected to 220
in same phase as the applied voltage. When another V variable frequency a.c. supply. (i) Find frequency
device Y is connected to the same supply, the elec- of source, for which average power absorbed by the
tric current is again 0.5 amp, but it leads the poten- circuit is maximum (ii) Calculate
√ the amplitude of
tial difference by π/2. (i) What are the devices X current. [Ans. (i) 250 Hz (ii) 4 2 A]

69 - Nikhil Kamboj — 8193989452 #edugramddun


Alternating Current

37. If the frequency of an A.C. is made 4 times of its ini- the current that flows in the circuit, when the current
tial value, What will be the value of new inductive is in phase with the voltage. [Ans. 25 µF ; 2 A]
reactance? [Ans. 4 times that of initial] 47. Obtain the resonant frequency and Q factor of a series
38. A capacitor has capacitance C and reactance X, if LCR circuit with L = 3 H. C = 27 µF and R = 7.4
capacitance and frequency become double, then what ohm. [Ans. 17.7 Hz ; 45.05]
will be the value of new reactance. [Ans. (X/4] ENERGY STORED IN CONDENSER,
ELECTRIC RESONANCE INDUCTOR AND LC OSCILLATION

48. An e.m.f. E = 100 sin 314t is applied across a pure


39. A variable frequency 230 V alternating voltage source
capacitor of 637 µF . Calculate the instantaneous
is connected across a series combination of L =
power P and maximum energy stored in the capac-
5H, C = 80µF and R = 40Ω.
itor. [Ans. 1000 sin 628 t watt; 3.185 J]
(i) Calculate the angular frequency of the source
which drives the circuit in resonance (ii) Obtain the 49. A capacitor of capacitance 54 µF is charged to a po-
impedance of the circuit and amplitude of current at tential of 25 V and then connected to a 15 mH inductor
resonance frequency (iii) Obtain r.m.s. potential drop to produce oscillations. What was the energy stored
across the three elements of the circuit at resonating in the capacitor initially and to what wavelength will
frequency. (iv) How do you explain the observation the circuit respond ? [Ans. 1.69 × 10−2 J; 176.8 Hz]
that the algebraic sum of voltages across the three el- POWER ASSOCIATED WITH AC CIRCUITS,
ements obtained is greater than the supplied voltage. POWER FACTOR
[Ans. (i) 50 rad/sec (ii) 40 Ω; 8.13 A (iii)

m
VL = VC = 1427.5V, VR = 230V ] 50. A series LCR circuit is made by taking R = 100 Ω,L
40. An inductance of 2 H, a capacitance of 18 µF and re- = 5/π H and C = 100/π µF . The series combination
sistance of 10kΩ are connected in series to an ac source is connected across an a.c. source of 220 V, 50 Hz.
of 20 V with adjustable frequency. (a) What frequency Calculate impedance of the circuit and peak value of
should be chosen to maximise the current in the cir-
cuit ? (b) What is the value of this maximum current?
[Ans. 26.5 Hz; 2.828 mA]
41. A series LCR circuit with C = 80 µF , L = 5H and
ra current flowing in the circuit. What is power factor
of the circuit ? Compare it with √
one at resonance fre-
quency. [Ans. 141.4 Ω; 2.2 A, 1/ 2]
51. A resistor R and an element X are connected in se-
ries to a.c. source of voltage. The voltage is found to
G
R = 40Ω is connected to a variable frequency 240 V
a.c. source. Calculate (i) angular frequency of the lead the current in phase by π/4. If X is replaced
source which drives the circuit in resonance. (ii) cur- by another element Y , the voltage lags behind the
rent at the resonating frequency. (iii) rms pot. drop current by π/4. Identify elements X and Y . When
both, X and Y are connected in series with R, to
u
across the capacitor. [Ans. 50 rad/s ; 6 A ; 1500 V]
the same source, will the power dissipated in the cir-
42. In a series LCR circuit, the resonance frequency is 800 cuit be maximum or minimum ? Justify your answer.
Hz. The half power points are obtained at frequencies
Ed

[Ans. X-inductor, Y-capacitor ; maximum]


745 and 855 Hz. Calculate the Q factor of the circuit
and the band width. [Ans. 7.27 ; 110 Hz] 52. An inductor of unknown value, a capacitor of 100 µF
and a resistor of 10 ohm are connected in series to 200
43. A capacitor, a resistor and a 40 mH inductor are con- V, 50 Hz a.c. source. It is found that power factor of
nected in series to an a.c. source of frequency 60 Hz. the circuit is unity. Calculate inductance of the induc-
Calculate the capacitance of the capacitor, if current tor and current amplitude. [Ans. 101.3 mH; 28.28 A]
is in phase with the voltage. [Ans. 175.7 µF ]
53. You are given three circuit elements X, Y and Z.
44. A capacitor of 50 µF , a resistor of 10 ohm and an in- When the element X is connected across an a.c. source
ductor L are in seris with an a.c. source of frequency of a given voltage, the current and voltage are in same
50 Hz. Calculate the value of L if phase angle between phase. When element Y is connected in series with X
current and voltage is zero. [Ans. 0.2 H] across the source, voltage is ahead of the current by
45. A resistor of resistance 400 ohm and a capacitor of re- π/4. But the current is ahead of voltage by π/4, when
actance 200 ohm are connected in series to a 220 V, 50 Z is connected in series with X, across the source.
Hz source. If the current in the circuit is 0.49 A, find Identify the circuit elements X, Y and Z. When all
the voltage across the resistor and capacitor. What is the three elements are connected in series across the
the value of inductance required so that voltage and same source, determine the impedance of the circuit.
current are in phase? [Ans. 196 V, 98 V, 0.64 H] Draw a plot of current versus the frequency of ap-
46. A capacitor of unknown capacitance, a resistance of plied source and mention the significance of this plot.
100 ohm and an inductor of inductance L = 4/π 2 [Ans. X| > R; Y | > L; Z| > C; Z = R]
henry are connected in series across an a.c. source of 54. An a.c. circuit containing 800 mH inductor and
200 V, 50 Hz. Calculate the value of capacitance and a 60 µF capacitor is in series with 15 ohm re-

70 - Nikhil Kamboj — 8193989452 #edugramddun


Alternating Current

sistance. They are connected to 230 V, 50 65. Calculate the current drawn by the primary of a trans-
Hz a.c. supply. Obtain average power trans- former which steps down 200 V to 20 V to operate a
ferred to each element and total power absorbed. device of resistance 20 ohm. Assume the efficiency of
[Ans. transformer to be 80%. [Ans. 0.125 A]
PL = 0, PC = 0, PR = 20.05 watt, P = 20.05 watt] 66. The ratio of the number of tums in primary and sec-
55. A group of electric bulbs have a power rating of 300 ondary coil of a step up transformer is 1:200. It is con-
watt. An a.c. voltage V = 141.4 sin (314t + π/3 ) is nected to a.c. mains of 220 volt. Calculate the voltage
applied to the group. Calculate the effective current. developed in the secondary. Determine the maximum
[Ans. 3 A] current in secondary coil when a current of 2 ampere
56. A 60 cycle AC circuit has a resistance of 200 ohms flows through the primary. [Ans. 44 kV, 0.01 amp]
and inductance of 10 mH. What is the power factor ? 67. A 60 W load is connected to the secondary coil of
What capacitance placed in the circuit will make the an ideal transformer whose primary coil draws line
power factor unity? [Ans. 0.99, 703 µF ] voltage (220 V). If a current of 0.54 A flows in
the load, What is the current in the primary coil?
57. In LCR series circuit with 100 Ω resistance is con-
Comment on the type of transformer being used.
nected to an a.c. source of 200 V and angular
[Ans. Step up transformer; Ip = 0.27A]
frequency 300 rad/s. When only the capacitance
is removed the current lags behind the voltage by 68. A transformer is used to light 100 W and 110 V lamp
60◦ . When only the inductance is removed, the cur- from a 220 V mains. If the main current is 0.5 A. Then
rent leads the voltage by 60°. Calculate the cur- what is the efficiency of the transformer? [Ans. 90 %]
rent and the power dissipated in the LCR circuit. 69. An iron core transformer with a turns ratio of 8 : 1

m
[Ans. 2 A; 400 W] has 120 V applied across the primary. What will be
58. In LCR circuit if resistance increases, how will quality the voltage across it?
ω0 L
factor varies? [Ans. decreases finitely; as Q = ]
R
59. The instantaneous current and voltage of an a.c. cir-

V. What is the power dissipation in the circuit?


[Ans. 1000 W]
ra
cuit are given by i = 10 sin 300t A and V = 200 sin 300t
DESCRIPTIVE QUES

70. Why is the use of a.c. voltage preferred over d.c. volt-
age? Give two reasons.
71. A.C. power is transmitted from a power house at a
G
60. The instantaneous current and voltage of an a.c. cir- high voltage. Why?
cuit are given by i = 10 sin 314t A and v = 50 sin 314t
72. (a) An a.c. source generating a voltage v = vm sin w
V. What is the power dissipation in the circuit?
t is connected to a capacitor of capacitance C. Find
[Ans. p = 250 W]
u
the expression for the current, i, flowing through it.
TRANSFORMER (b) Plot a graph of v and i to show that the current is
p/2 ahead of the voltage.
Ed

61. In an ideal transformer, number of turns in primary (c) A resistor of 200Ω and a capacitor of 15.0 µF are
and secondary are 200 and 1000 respectively. If connected in series to a 220 V, 50 Hz a.c. source. Cal-
the power input to the primary is 10 kW at 200 culate the current in the circuit and the rms voltage
V, calculate (i) output voltage (ii) primary current. across the resistor and the capacitor.
[Ans. 1000 V, 50 A] 73. Define the term root mean square (rms) value of a.c..
Derive the relation between rms value and the peak
62. The primary of a transformer has 200 turns and sec-
value of a.c..
ondary has 1000 turns. The power output from sec-
ondary at 1000 V is 9 kW. Calculate primary volt- 74. Distinguish between the terms ’average value’ and ’rms
age and heat loss in primary. Take resistance of pri- value’ of an alternating current.
mary coil 0.2 Ω and efficiency of transformer = 90%. 75. An a.c. source of voltage V = V0 sin wt is connected
[Ans. 200 V ; 500 W] to a series combination of L, C and R.
63. A transformer has 200 primary tums and 150 sec- (i) Use the phasor diagram to obtain expressions for
ondary turns. If the operating voltage for the load con- impedance of the circuit and phase angle between volt-
nected to the secondary is measured to be 300 V, what age and current.
is the voltage supplied to the primary? [Ans. 400 v] (ii) Find the condition when current will be in phase
with the voltage. What is the circuit in this condition
64. The output voltage of an ideal transformer, connected called?
to a 240 V a.c. mains is 24 V. When this transformer
is used to light a bulb with rating 24 V, 24 W, cal- 76. Define capacitor reactance. Write its S.I. units.
culate the current in the primary coil of me circuit. 77. Define ‘quality factor’ of resonance in series LCR cir-
[Ans. 0.1 A] cuit. What is its SI unit?

71 - Nikhil Kamboj — 8193989452 #edugramddun


Alternating Current

78. Prove that (i) an ideal inductor (ii) an ideal Capacitor in turn across (i) ac source and (ii) a ‘dc’ battery.
; does not dissipate power in an a.c. circuit. What change would you notice in each case if the ca-
79. Define the term ‘wattless current’. pacitance of the capacitor is increased?
80. An electric lamp having coil of negligible inductance 84. State the principle of working of a transformer. Can
connected in series with a capacitor and an a.c. source a transformer be used to step up or step down a d.c.
is glowing with certain brightness. How does the voltage? Justify your Answer.
brightness of the lamp change on reducing the (i) ca-
pacitance, and (ii) the frequency? Justify your An- 85. Mention various energy losses in a transformer.
swer. 86. (a) Draw a labelled diagram of a step-up transformer.
81. A light bulb is rated 200 W for 220 V ac supply of 50 Obtain the ratio of secondary to primary voltage in
Hz. Calculate (i) the resistance of the bulb; (ii) the terms of number of turns and currents in the two coils.
rms current through the bulb. (b) A power transmission line feeds input power at
82. An alternating voltage given by V = 140 sin 314 t 2200 V to a step-down transformer with its primary
is connected across a pure resistor of 50 ohms. Find windings having 3000 turns. Find the number of turns
(i) the frequency of the source. (ii) the rms current in the secondary to get the power output at 220 V.
through the resistor. 87. Mention the two characteristic properties of the ma-
83. A lamp is connected in series with a capacitor. Predict terial suitable for making core of a transformer.
your observation when this combination is connected

m
ra
u G
Ed

72 - Nikhil Kamboj — 8193989452 #edugramddun


Electromagnetic Waves
Important Formulae and Notes

dϕE ▶ Average Energy density of electric field


▶ Displacement Current (ID ) = ϵ0
  dt
dE d V ϵ0 A dV q 1

E

1
= ϵ0 A = ϵ0 A = uE = ϵ0 E 2 = ϵ0 √0 = ϵ0 E02
dt dt d d dt 2 2 2 4
dV
ID = C
dt where E and E0 are r.m.s. value and amplitude of
# #
I
▶ B.dl = µ0 (I + ID ) electric field.

µ0 2I ▶ Average Energy density


√ of magnetic field
▶ B= , Where B is Magnetic field induction at 1 2 (B0 / 2)2 B02
4π r uB = B = =
point outside a straight wire at a perpendicular dis- 2µ0 2µ0 4µ0
tance r from a straight wire carrying current I. where B and B0 are r.m.s. and amplitude of magnetic
field.
▶ Energy of photo, E = hν = hc/λ √
▶ E0 = B0 c, where E0 = Amplitude (or maximum ▶ e = e0 / 2
value) of electric field. ▶ Total average density = uE + uB = 2uE
B0 = Amplitude (or maximum value) of magnetic 1 B2
field. = ϵ0 E02 = 0

m
2 2µ0

Multiple Choice Questions

1. In electromagnetic waves the phase difference between a) 1.8 × 108 m/s b) 1.8 × 104 m/s
electric and magnetic field vectors are
a) zero b) π/4 c) π/2 d) π
ra c) 1.8 × 106 m/s

6. The quantity
a)

µ0 ϵ0 presents
speed of sound
d) 1.8 × 102 m/s

2. Which of the following has minimum wavelength?


G
b) speed of light in vacuum
a) Blue light b) g-rays
c) speed of e.m. waves
c) infrared rays d) microwave
d) inverse of speed of light in vacuum
u
3. The correct option, if speeds of gamma rays, X-rays 7. In electromagnetic wave if ue and um are mean electric
and microwave are Vg, Vx an Vm respectively will be. and magnetic energy densities respectively, then
Ed

a) Vg > Vx > Vm b) Vg < Vx < Vm a) ue = um b) ue > um


c) Vg > Vx > Vm d) Vg = Vx = Vm 1
c) ue < um d) (ue )2 = (um )2
2
4. Which of the following has maximum penetrating
power? 8. Which of the following is called heat radiation?
a) Ultraviolet radiation b) Microwaves a) X-rays b) g-rays
c) g-rays d) Radio waves c) Infrared radiation d) Microwave

5. Electromagnetic waves travelling in a medium having 9. From Maxwell’s hypothesis, a charging electric field
relative permeability ţr = 1.3 and relative permittiv- gives rise to
ity Er = 2.14. The speed of electromagnetic waves in a) an electric field. b) an induced emf.
medium must be c) a magnetic field. d) a magnetic dipole.

Practice Questions and Numerical

DESCRIPTIVE QUES respect of their frequencies ;


X-rays, Microwaves, UV rays and radiowaves.

1. Name the part of the electromagnetic spectrum of


wavelength 10-2 m and mention its one application. 3. Name the electromagnetic radiation to which waves of
wavelength in the range of 10-2 m belong. Give one
2. Write the following radiations in ascending order in use of this part of EM spectrum.
Electromagnetic Waves

4. Name the part of electromagnetic spectrum which is magnitude.


suitable for 10. Identify the following electromagnetic radiations as
(i) radar systems used in aircraft navigation per the wavelengths given below. Write one applica-
(ii) treatment of cancer tumours. tion of each.
5. How does a charge q oscillating at certain frequency (a) 1 mm (b) 10−12 m (c) 10−8 m
produce electromagnetic waves? Sketch a schematic 11. (a) When the oscillating electric and magnetic fields
diagram depicting electric and magnetic fields for are along the x- and indirection respectively point out
an electromagnetic wave propagating along the Z- the direction of propagation . of electromagnetic wave.
direction. Express the velocity of propagation in terms of the am-
6. How are infrared waves produced? Why are these re- plitudes of the oscillating electric and magnetic fields.
ferred to as ‘heat waves’? Write their one important (b) How do you show that the em wave carries energy
use. and momentum?
7. A capacitor is connected in series to an ammeter across 12. Answer the following questions :
a d.c. source. Why does the ammeter show a momen- (a) Name the em waves which are produced during
tary deflection during the charging of the capacitor? radioactive decay of a nucleus. Write their frequency
What would be the deflection when it is fully charged? range.
(b) Welders wear special glass goggles while working.
8. Name the types of e.m. radiations which Why? Explain.
(i) are used in destroying cancer cells, (c) Why are infrared waves often called as heat waves?
(ii) cause tanning of the skin and Give their one application.
(iii) maintain the earth’s warmth.

m
Write briefly a method of producing any one of these 13. Name the type of e.m. waves having a wavelength
waves. range of 0.1 m to 1 mm. How are these waves gener-
ated? Write their two uses.
9. Electromagnetic waves with wavelength
(i) l1 is suitable for radar systems used in aircraft nav-
ra 14. Name the type of e.m. waves having a wavelength
igation. range 10−7 m to 10−9 m. How are these waves gener-
(ii) l2 is used to kill germs in water purifiers. ated? Write their two uses.
(iii) l3 is used to improve visibility in runways during 15. (i) A plane electromagnetic wave travels in vacuum
fog and mist conditions. along x-direction. What can you say about the direc-
G
Identify and name the part of the electromagnetic tion of electric and magnetic field vectors?
spectrum to which these radiations belong.Also ar- (ii)How are infrared waves produced? Write their one
range these wavelengths in ascending order of their important use.
u
Ed

74 - Nikhil Kamboj — 8193989452 #edugramddun


Ray Optics and Optical Instruments
Important Formulae and Notes

R (c) Power of spherical refracting surface


▶ Focal length, f =
2
where R = Radius of curvature µ2 − µ1
P =
▶ Mirror formula R
1 1 1
+ = ▶ Lens maker’s formula,
u v f
where u = object distance from centre of curvature
1 1 1
 
v = image distance from centre of curvature = (µ − 1) −
I f R1 R2
▶ Linear magnification, m =
O where f = focal length of lens
where I = size of image
O = size of object µ = refractive index of lens
R1 and R2 = radii of curvature of two refracting sur-
▶ (a) Refractive index faces of a thin lens
sin∠i 1 1 1
a
µb = (Snell’s Law) ▶ Lens formula, − =
sin∠r u v f
where a µb = refractive index of medium b
w.r.t. medium a ▶ Power of lens
1

m
∠i = angle of incidence (a) P =
∠r = angle of refraction f
c 
1 1

(b) Refractive index, µ = (b) P = (µ − 1) −
v R1 R2
where c = velocity of light
(c) Refractive index, a µb =
Real depth
Apparent depth
(d) Refractive index, a µc =a µb ×b µc
where b and c = material of two refracting slabs
ra ▶ Two lenses placed in contact
(a)
1
f
=
1
f1
+
1
f2
where f1 and f2 = focal length of
two lenses f = focal length of the combination
G
a = medium in which slabs are placed (b) Power of combination, P = P1 + P2
1 (c) Magnification of combination,
(e) In total internal reflection a µb =
sin C m = m1 × m2
t ▶ Prism
▶ Lateral shift, d = sin(i − r)
u
cos r
where t = thickness (a) Angle of prism, ∠A = ∠r1 + ∠r2
 of slab 
1 where ∠r1 = angle of refraction at 1st surface of
▶ Normal shift, d = t 1 − a
Ed

µb the prism
▶ Image formed between two inclined plane mirrors ∠r2 = angle of refraction at 2nd surface of the
prism
3600
(a) n = −1 (b) ∠A + δ = ∠i + ∠e
θ where ∠δ = angle of deviation
3600
(when is an even number) ∠i = angle of incidence
θ
360 0 ∠e = angle of emergence
(b) n = (c) In minimum deviation position,
θ
3600 sin (A + δm )/2
(when is an odd number) Refractive index, µ =
θ sin A/2
▶ Spherical refracting surface where δm = angle of minimum deviation
(a) When object is situated in the rarer medium, (d) Angle of deviation, δ = A(µ − 1)
(e) Angular dispersion, θ = δv − δr
µ1 µ2 µ2 − µ1 where δy = angle of deviation of violet colour
+ =
u v R δy = angle of deviation of yellow colour
where µ1 = refractive index of rarer medium ▶ Magnifying power of simple microscope
µ2 = refractive index of spherical refracting sur-
face
 
D
M = 1+
(b) When object is situated in denser medium f
µ2 µ1 µ1 − µ2
− + = where D = least distance of distinct vision
u v R
Ray Optics and Optical Instruments

▶ Magnifying power of compound microscope, (b) When the final image is formed at least distance
v0

D
 of distinct vision, magnifying power of telescope,
M= 1+
u0 fe f0

fe

M =− 1+
where v0 = distance of image from object lens fe D
u0 = distance of object from object lens
fe = focal length of eye in lens where f0 = focal length of objective
▶ Astronomical telescope
(a) Magnifying power in normal adjustment,
f0
M =−
fe

Multiple Choice Questions

1. A converging lens is used to form an image on a screen. d) The magnification for virtual image formed by a
When the upper half of the lens is covered by an concave lens is negative
opaque screen.
7. The apparent flattening of the sun at sunset and sun-
a) half the image will disappear. rise is due to
b) incomplete image will be formed.
c) intensity of image will decrease but complete im- a) refraction
b) diffraction

m
age is formed.
c) total internal reflection
d) intensity of image will increase but image is not
d) interference
distinct.
8. We combine two lenses, one is convex and other is con-
2. In optical fibres, the refractive index of the core is
a)
b)
c)
greater than that of the cladding.
equal to that of the cladding.
smaller than that of the cladding.
ra cave having focal lengths /, and f2 and their combined
focal length is F. Combination of the lenses will behave
like concave lens, if
a) f1 > f2 b) f1 = f2
d) independent of that of cladding.
G
c) f1 < f2. d) f1 ≤ f2
3. An object is placed at a distance of 0.5 m in front of a 9. The focal length of a biconvex lens of radii of each
plane mirror. The distance between object and image surface 50 cm and refractive index 1.5, is
will be
u
a) 40.4 cm b) 75 cm
a) 0.25 m b) 0.5 m c) 1.0 m d) 2.0 m c) 50 cm d) 80 cm
Ed

4. Air bubble in water behaves as 10. A metal coin is at bottom of a beaker filled with a
a) sometimes concave, sometimes convex lens liquid of refractive index = 4/3 to height of 6 cm. To
b) concave lens an observer looking from above the surface of liquid,
c) convex lens coin will appear at a depth
d) always refracting surface a) 1.5 cm b) 6.75 cm
c) 4.5 cm d) 7.5 cm
5. Which of the following is not due to total internal re-
flection? 11. If a convex lens of focal length 80 cm and a concave
lens of focal length 50 cm are combined together, what
a) Working of optical fibre will be their resulting power?
b) Difference between apparent and real depth of
pond a) + 6.5 D b) – 6.5 D
c) + 7.5 D d) – 0.75 D
c) Mirage on hot summer days
d) Brilliance of diamond 12. A convex lens and a concave lens, each having the
same focal length of 25 cm, are put in contact to form
6. Identify the wrong sign convention a combination of lenses. The power of the combination
a) The magnification for virtual image formed by a (in dioptres) is
convex lens is positive a) zero b) 25
b) The magnification for real image formed by a c) 50 d) infinity
convex lens is negative
c) The height measured normal to the principal 13. The refractive
√ index of the material of an equilateral
axis upwards is positive prism is 3. What is the angle of minimum deviation?

76 - Nikhil Kamboj — 8193989452 #edugramddun


Ray Optics and Optical Instruments

a) 45° b) 60° c) 37° d) 30° 23. Two beams of red and violet color are made to pass
separately through a prism (angle of the prism is 60°).
14. In the formation of a rainbow, the light from the sun In the position of minimum deviation, the angle of re-
on water droplets undergoes fraction will be
a) dispersion only. b) only TIR. a) 30° for both the colors
c) dispersion and TIR. d) scattering. b) greater for the violet color
c) greater for the red color
15. When a ray of light enters from one medium to an- d) equal but not 30° for both the colors
other, then which of the following does not change?
a) Frequency b) Wavelength 24. Which of the following colours of white light deviated
c) Speed d) Amplitude most when passes through a prism?
a) Red light b) Violet light
16. A rod of length 10 cm lies along the principal axis of c) Yellow light d) Both (a) and (b)
a concave mirror of focal length 10 ’em in such a way
that its end closer to the pole is 20 cm away from the 25. An under-water swimmer cannot see very clearly even
mirror. The length of the image is in absolutely clear water because of
a) 10 cm b) 15 cm a) absorption of light in water
c) 2.5 cm d) 5 cm b) scattering of light in water
c) reduction of speed of light in water
17. For a total internal reflection, which of the following d) change in the focal length of eye lens
is correct?

m
26. An object is immersed in a fluid. In order that the
a) Light travels from rarer to denser medium.
object becomes invisible, it should
b) Light travels from denser to rarer medium.
c) Light travels in air only. a) behave as a perfect reflector.
b) absorb all light falling on it.
d) Light travels in water only.

18. Critical angle of glass is j1 and that of water is j2 . The


critical angle for water and glass surface would be (ţg
= 3/2, ţw = 4/3).
ra c)
d)
have refractive index one.
have refractive index exactly matching with that
of the surrounding fluid.
G
27. When light is refracted, which of the following does
a) less than j2 b) between j1 and j2 not change ?
c) greater than j2 d) less than j1
a) Wavelength b) Frequency
19. Mirage is a phenomenon due to c) Velocity d) Amplitude
u

a) refraction of light 28. If the light moving in a straight line bends by a small
b) reflection of light but fixed angle, it may be a case of
Ed

c) total internal reflection of light


a) reflection b) refraction
d) diffraction of light
c) diffraction d) both (a) and (b)
20. A convex lens is dipped in a liquid whose refractive
29. A ray of light incident at an angle j on a refracting
index is equal to the refractive index of the lens. Then
face of a prism emerges from the other face normally.
its focal length will
If the angle of the prism is 5° and the prism is made
a) become zero of a material of refractive index 1.5, the angle of inci-
b) become infinite dence is [NCERT Exemplar]
c) become small, but non-zero
a) 7.5° b) 5°
d) remain unchanged
c) 15° d) 2.5°
21. Which of the following forms a virtual and erect image 30. A short pulse of white light is incident from air to
for all positions of the object? a glass slab at normal incidence. After travelling
a) Concave lens b) Concave mirror through the slab, the first colour to emerge is [NCERT
c) Convex mirror d) Both (a) and (c) Exemplar]
a) blue b) green
22. Two lenses of focal lengths 20 cm and – 40 cm are held
c) violet d) red
in contact. The image of an object at infinity will be
formed by the combination at 31. An object approaches a convergent lens from the left
a) 10 cm b) 20 cm of the lens with a uniform speed 5 m/s and stops at
c) 40 cm d) infinity the focus. The image

77 - Nikhil Kamboj — 8193989452 #edugramddun


Ray Optics and Optical Instruments

a) moves away from the lens with an uniform speed d) In the side mirror, the speed of the approach-
5 m/s. ing car would appear to increase as the distance
b) moves away from the lens with an uniform between the cars decreases.
acceleration
35. Consider an extended object immersed in water con-
c) moves away from the lens with a non-uniform
tained in a plane trough. When seen from close to the
acceleration.
edge of the trough the object looks distorted. Which
d) moves towards the lens with a non-uniform of the following is not correct.
acceleration.
a) the apparent depth of the points close to the edge
32. You are given four sources of light each one provid- are nearer the surface of the water compared to
ing a light of a single colour – red, blue, green and the points away from the edge.
yellow. Suppose the angle of refraction for a beam of b) the angle subtended by the image of the object
yellow light corresponding to a particular angle of in- at the eye is smaller than the actual angle sub-
cidence at the interface of two media is 90°. Which of tended by the object in air.
the following statements is correct if the source of yel- c) some of the points of the object far away from
low light is replaced with that of other lights without the edge may not be visible because of total in-
changing the angle of incidence? [NCERT Exemplar] ternal reflection.
a) The beam of red light would undergo total inter- d) water in a trough acts as a lens and magnifies
nal reflection. the object.
b) The beam of red light would bend towards nor-
mal while it gets refracted through the second 36. A magnifying glass is used, as the object to be viewed

m
medium. can be brought closer to the eye than the normal near
c) The beam of blue light would undergo total in- point. This results in
ternal reflection. a) a larger angle to be subtended by the object at
d) The beam of green light would bend away from the eye and hence viewed in greater detail.
the normal as it gets refracted through the sec-
ond medium.
33. The radius of curvature of the curved surface of a
plano-convex lens is 20 cm. If the refractive index
ra b) the formation of a real inverted image.
c) increase in the field of view.
d) infinite magnification at the near point.

37. Virtual image formed by convex mirror has magnifica-


G
of the material of the lens be 1.5, it will [NCERT Ex- tion
emplar] a) positive b) negative
a) act as a convex lens only for the objects that lie c) 0 d) none of above
on its curved side.
u

b) act as a concave lens for the objects that lie on 38. A concave lens of refractive index 1.5 is immersed in a
its curved side. medium of refractive index 1.65. What is the nature
of the lens?
Ed

c) act as a convex lens irrespective of the side on


which the object lies. a) converging b) diverging
d) act as a concave lens irrespective of side on which c) mixed d) none of above
the object lies.
39. An air bubble is formed inside water. Does it act as a
34. A car is moving with at a constant speed of 60 km converging lens or a diverging lens?
h-1 on a straight road. Looking at the rear view mir- a) Plano convex b) converging
ror, the driver finds that the car following him is at c) diverging d) none of these
a distance of 100 m and is approaching with a speed
of 5 km h-1. In order to keep track of the car in the 40. A convex lens is placed in contact with a plane mirror.
rear, the driver begins to glance alternatively at the A point object at a distance of 20 cm on the axis of
rear and side mirror of his car after every 2 s till the this combination has its image coinciding with itself.
other car overtakes. If the two cars were maintain- What is the focal length of the lens?
ing their speeds, which of the following statement (s) a) 10 cm b) 20 cm
is/are correct? [NCERT Exemplar] c) 40 cm d) 400 cm
a) The speed of the car in the rear is 65 km h-1.
b) In the side mirror the car in the rear would ap- 41. The light reflected by a plane mirror may form a real
pear to approach with a speed of 5 km h-1 to the image
driver of the leading car. a) if the rays incident on the mirror are diverging
c) In the rear view mirror the speed of the ap- b) if the rays incident on the mirror are converging
proaching car would appear to decrease as the c) if the object is placed very close to the mirror
distance between the cars decreases. d) under no circumstances

78 - Nikhil Kamboj — 8193989452 #edugramddun


Ray Optics and Optical Instruments

42. In image formation from spherical mirrors, only parax- 49. Critical angle of light passing from glass to water is
ial rays are considered because they minimum for
a) are easy to handle geometrically a) red colour b) green colour
b) contain most of the intensity of the incident light c) yellow colour d) violet colour
c) form nearly a point image of a point source
d) show minimum dispersion effect 50. A glass slab of thickness 4 cm contains the same num-
ber of waves as 5 cm of water when both are traversed
43. For reflection through spherical surfaces, the normal by the same monochromatic light. If the refractive
at the point of incidence is index of water is 4/3, what is that of glass?
a) perpendicular to the principle axis and passes
a) 5/3 b) 5/4
through the centre of curvature
c) 16/15 d) 1.5
b) perpendicular to the focal plane and passes
through the pole.
51. A point object is placed at the centre of a glass sphere
c) perpendicular to the tangent plane at pole and of radius 6 cm and refractive index 1.5. The distance
passes through the focus. of the virtual image from the surface of the sphere is
d) perpendicular to the tangent plane at the point
of incidence and passes through the centre of a) 2 cm b) 4 cm c) 6 cm d) 12 cm
curvature.
52. A planoconvex lens of focal length 16 cm, is to be made
44. A person is six feet tall. How tall must a plane mirror of glass of refractive index 1.5. The radius of curvature
be if he is able to see his entire length? of the curved surface should be

m
a) 3 ft b) 4.5 ft a) 8 cm b) 12 cm c) 16 cm d) 24 cm
c) 7.5 ft d) 6 ft
53. Electromagnetic radiation belonging to ......... region
45. An object is placed 40 cm from a concave mirror of of spectrum is called light.
focal length 20 cm. The image formed is
a)
b)
c)
real, inverted and same in size
real, inverted and smaller
virtual, erect and larger
ra a) 100 nm to 400 nm
c) 750 nm to 10 nm
b) 400 nm to 750 nm
d) 1000 nm to 1400 nm

54. The turning back of light into the same medium after
G
d) virtual, erect and smaller
incident on a boundary separating two media is called
46. Which of the following is incorrect statement? a) reflection of light b) refraction of light
a) the magnification produced by a convex mirror c) dispersion of light d) interference of light
u
is always less than one
b) a virtual, erect, same-sized image can be ob- 55. A point source of light is placed in front of a plane
tained using a plane mirror mirror. Then
Ed

c) a virtual, erect, magnifield image can be formed a) all the reflected rays meet at a point when pro-
using a concave mirror duced backward
d) a real, inverted, same-sized image can be formed b) only the reflected rays close to the normal meet
using a convex mirror. at a point when produced backward.
c) only the reflected rays making a small angle
47. Total internal reflection can take place only if
with the mirror, meet at a point when produced
a) light goes from optically rarer medium (smaller backward.
refractive index) to optically denser medium d) light of different colours make different images.
b) light goes from optically denser medium to rarer
medium 56. The field of view is maximum for
c) the refractive indices of the two media are close
a) plane mirror b) concave mirror
to different
c) convex mirror d) cylindrical mirror
d) the refractive indices of the two media are widely
different
57. A virtual image larger than the object can be obtained
48. When the angle of incidence of a light ray is greater by
than the critical angle it gets a) concave mirror b) convex mirror
a) critically refracted c) plane mirror d) concave lens
b) totally reflected
c) total internally reflected 58. An object is placed 40 cm from a concave mirror of
d) totally refracted focal length 20 cm. The image formed is

79 - Nikhil Kamboj — 8193989452 #edugramddun


Ray Optics and Optical Instruments

a) real, inverted and same in size a) its wavelength and frequency both increase
b) real, inverted and smaller b) its wavelength increases but frequency remains
c) virtual, erect and larger unchanged
d) virtual, erect and smaller c) its wavelength decreases but frequency remains
unchanged
59. All of the following statements are correct except d) its wavelength and frequency both decrease
a) the magnification produced by a convex mirror 66. Total internal reflection can take place only if
is always less than one
b) a virtual, erect, same-sized image can be ob- a) light goes from optically rarer medium (smaller
tained using a plane mirror refractive index) to optically denser medium
c) a virtual, erect, magnifield image can be formed b) light goes from optically denser medium to rarer
using a concave mirror medium
d) a real, inverted, same-sized image can be formed c) the refractive indices of the two media are close
using a convex mirror. to different
d) the refractive indices of the two media are widely
different
60. The image formed by a concave mirror is
a) always real 67. The difference between reflection and total internal re-
b) always virtual flection is that
c) certainly real if the object is virtual a) the laws of reflection hold true for reflection but
d) certainly virtual if the object is real not for total internal reflection.

m
b) total internal reflection can take place only when
61. In image formation from spherical mirrors, only parax- light travels from a rarer medium to a denser
ial rays are considered because they medium while reflection can take place vice-versa
also.
a)
b)
c)
d)
are easy to handle geometrically

form nearly a point image of a point source


show minimum dispersion effect
ra
contain most of the intensity of the incident light c) reflection can take place when light travels from
a rarer medium to denser medium and vice-versa
but total internal reflection can take place only
when it travels from an optically denser to an
G
optically rarer medium.
62. For reflection through spherical surfaces, the normal
at the point of incidence is d) reflection is a natural phenomena while total in-
ternal reflection is man-made.
a) perpendicular to the principle axis and passes
u
through the centre of curvature 68. When the angle of incidence of a light ray is greater
b) perpendicular to the focal plane and passes than the critical angle it gets
through the pole. a) critically refracted
Ed

c) perpendicular to the tangent plane at pole and b) totally reflected


passes through the focus. c) total internally reflected
d) perpendicular to the tangent plane at the point d) totally refracted
of incidence and passes through the centre of
curvature. 69. Which of the following phenomena is used in optical
fibres ?
63. Which of the following (referred to a spherical mirror) a) Total internal reflection
depends on whether the rays are paraxial or not? b) Scattering
c) Diffraction
a) Pole b) Focus
d) Refraction
c) Radius of curvature d) Principal of axis
70. Critical angle of light passing from glass to water is
64. Which of the following is correct for the beam which minimum for
enters the medium?
a) red colour b) green colour
a) Travel as a cylindrical beam c) yellow colour d) violet colour
b) Diverge
c) Converge 71. The speed of light in an isotropic medium depends on
d) Diverge near the axis and converge near the a) the nature of the source
periphery b) its wavelength
c) its direction of propagation
65. When light is refracted into a medium, d) its intensity

80 - Nikhil Kamboj — 8193989452 #edugramddun


Ray Optics and Optical Instruments

72. A parallel beam of light is incident on a converging a) f0 + 2fe b) f0 × fe


lens parallel to its principal axis. As one moves away c) f0 − fe d) f0 + fe
from the lens on the other side on its principal axis,
the intensity of light 77. An astronomical refractive telescope has an objective
a) remains constant of focal length 20 m and an eyepiece of focal length 2
b) continuously increases cm. Which one of the following is not correct?
c) continuously decreases a) The length of the telescope tube is 20.02 m.
d) first increases then decreases b) The magnification is 1000.
c) The image formed is inverted.
73. The rays of different colours fail to converge at a point d) An objective of a larger aperture will increase
after going through a converging lens. This defect is the brightness and reduce chromatic aberration
called of the image.
a) spherical aberration b) distortion 78. An astronomical refractive telescope has an objective
c) coma d) chromatic aberration of focal length 20 m and an eyepiece of focal length 2
cm. Then
74. The focal length of a converging lens are fV and fR
a) the magnification is 1000
for violet and red light respectively. Then
b) the length of the telescope tube is 20.02 m
a) fV > fR c) the image formed of inverted
b) fV = fR d) all of these
c)

m
fV < fR
d) any of the three is possible depending on the 79. The astronomical telescope consists of objective and
value of the average refractive index eyepiece. The focal length of the objective is
a) equal to that of the eyepiece.
75. A narrow beam of white light goes through a slab hav-
ing parallel faces
a) the light never splits in different colours
b) the emergent beam is white
ra b)
c)
d)
shorter than that of eyepiece.
greater than that of eyepiece.
five times shorter than that of eyepiece.

80. Tom’ lenses of focal lengths ± 15 cm and ± 150 cm


c) the light inside the slab is split into different
G
colours are available for making a telescope. To produce the
d) the light inside the slab is white largest magnification, the focal length of the eyepiece
should be
u
76. The length of an astronomical telescope for normal a) + 15 cm b) + 150 cm
vision (relaxed eye) will be c) – 150 cm d) – 15 cm
Ed

Practice Questions and Numerical

REFRACTIVE INDEX prism vary, if the incident violet light is replaced with
red light? [Ans. decreases]
1. A microscope is focused on a coin lying at the bottom 5. State the conditions for the phenomenon of total in-
of a beaker. The microscope is now raised up by 1 ternal reflection to occur.
cm. To what depth should the water be poured into 6. When light travels from a rarer to a denser medium,
the beaker so that coin is again in focus? (Refractive the speed decreases. Does this decrease in speed im-
index of water is 3/4). [Ans. 4 cm] ply a decrease in the energy carried by the light wave?
2. The bottom of a container is a 4.0 cm thick glass Justify your answer. [Ans. No]
(µ=1.5) slab. The container contains two immiscible 7. When an object is placed between f and 2f of a concave
liquids A and B of depths 6.0 cm and 8.0 cm respec- mirror, would the image formed be
tively. What is the apparent position of a scratch on (i) real or virtual and (ii) diminished or magnified?
the outer surface of the bottom of the glass slab when [Ans. (i) Real (ii) magnified]
viewed through the container? Refractive indices of A
8. What is the focal length of a biconvex lens of radii
and B are 1.4 and 1.3 respectively. [Ans. 4.98cm]
of each surface 50 cm and refractive index 1.5?
3. How does the power of a convex lens vary, if [Ans. 50 cm]
the incident red light is replaced by violet light?
9. A concave lens of refractive index 1.5 is immersed in a
[Ans. Power increased]
medium of refractive index 1.65. What is the nature
4. How does the angle of minimum deviation of a glass of the lens? [Ans. converging]

81 - Nikhil Kamboj — 8193989452 #edugramddun


Ray Optics and Optical Instruments

10. A Plano-convex lens of focal length 16 cm, is to be 21. A rod of length 10 cm lies along the principal axis of
made of glass of refractive index 1.5. What should a concave mirror of focal length 10 cm in such a way
be the radius of curvature of the curved surface. that its end closer to the pole is 20 cm away from
[Ans. 8 cm] the mirror. What will be the length of the image?
11. Define refractive index of a transparent medium. Ex- [Ans. 5cm]
plain its relation with color(wavelength) and velocity 22. An object is held in front of a concave mirror of focal
of incident light. length 15 cm. The image formed is 3 times the size
12. (i) What is the relation between critical angle and re- of the object. Calculate two possible distances of the
fractive index of a material? object from the mirror. [Ans. -10cm ;-20cm]
(ii) Does critical angle depend on the colour of light? 23. Two objects A and B, when placed one after another
Explain. in front of a concave mirror of focal length 10 cm, form
13. A convex lens made up of glass of refractive index 1.5 images of same size. Size of object A is four times that
is dipped, in turn, of B. If object A is placed at a distance of 50 cm from
(i) a medium of refractive index 1.6, the mirror. what should be the distance of B from the
(ii) a medium of refractive index 1.3. mirror ? [Ans. — 20 cm]
(a) Will it behave as a converging or a diverging lens 24. An object of size 10 cm is placed at a distance of
in the two cases? 50 cm from a concave mirror of focal length 15 cm.
(b) How will its focal length change in the two media? Calculate location, size and nature of the image.
REFLECTION OF LIGHT [Ans. - 21.4 cm, - 4.28 cm ; Real]
25. An object 2 cm high is placed at a distance of

m
14. Light of wavelength 6000 Åfalls on a plane reflecting 16 cm from a concave mirror, which produces a
surface. What are the wavelength and frequency of real image 3 cm high. What is the focal length
reflected light. If angle between incident ray and re- of the mirror? Find the position of the image?
flected ray is 60°, what is the angle of incidence ? [Ans. f= -9-6 cm, v = - 24 cm]
[Ans. 6000 Å, 5 × 1014 s Hz ; 30◦ ]
ra
15. A ray of light is incident at an angle of 60° on a hor-
izontal plane mirror. Through what angle should the
26. An object is placed in front of a concave mirror of focal
length 20 cm. The image formed is three times the size
of the object. Calculate two possible distances of the
object from the mirror. [Ans. -40/3 cm and -80/3 cm]
mirror be tilted to make the reflected ray horizontal ?
G
[Ans. 15°] 27. A square object is placed l5 cm from a convex mir-
16. A point object is held between two plane mirrors in- ror of radius of curvature 90 cm. Calculate the
clined at 45°. What is the number of images seen ? position of the image and is areal magnification.
u
[Ans. 7] [Ans. 11.25 cm behind the mirror; 9/16]
17. A 12 m tall tree is to be photographed with a pin hole 28. The image formed by a convex mirror of focal length
camera. It is situated 15 m away from the pin hole. 30 cm is a quarter of the Object. What is the distance
Ed

How far should the screen be placed from the pin hole of the object from the mirror? [Ans. -90 cm]
to obtain a 12 cm tall image of the tree ? [Ans. 15 cm]
29. Calculate the distance of an object of height from a
18. A metal coin is at bottom of a beaker filled with a concave mirror of focal length 10 cm so as to obtain a
liquid of refractive index = 4/3 to height of 6 cm. To real magnification 2. [Ans. -15 cm]
an observer looking from above the surface of liquid,
30. An object is placed in front of a concave mirror of
coin will appear at what depth? [Ans. 4.5 cm]
radius of curvature 40 cm at a distance of 10cm.
19. A microscope is focused on a coin lying at the bottom Find the position, nature and magnification of Image.
of a beaker. The microscope is now raised up by 1 [Ans. - 20 cm, Virtual, erect behind the mirror, m =
cm. To what depth should the water be poured into + 2]
the beaker so that coin is again in focus? (Refractive
31. When an object is placed between f and 2f of a concave
index of water is 3/4). [Ans. 4 cm]
mirror, would the image formed be
20. The bottom of a container is a 4.0 cm thick glass (i) real or virtual and
(µ=1.5) slab. The container contains two immiscible (ii) diminished or magnified?
liquids A and B of depths 6.0 cm and 8.0 cm respec- [Ans. (i) Real (ii) magnified]
tively. What is the apparent position of a scratch on
LAWS OF REFRACTION, REAL AND
the outer surface of the bottom of the glass slab when
APPARENT DEPTHS
viewed through the container? Refractive indices of A
and B are 1.4 and 1.3 respectively. [Ans. 4.98 cm] 32. A mark is made on the bottom of a beaker and a mi-
REFLECTION AT SPHERICAL MIRRORS croscope is focused on it. The microscope is raised
through 1.5 cm. To what height water must be poured

82 - Nikhil Kamboj — 8193989452 #edugramddun


Ray Optics and Optical Instruments

into the beaker to bring the mark again into focus ? 43. An object is placed 50 cm from the surface of a glass
Given that it for water is 4/3. [Ans. 6 cm] sphere of radius 10 cm along the diameter. Where will
33. Calculate the speed and wavelength of light (i) in the final image be formed after refraction at both the
glass (ii) in air, when light waves of frequency surfaces µ of glass = 1.5. [Ans. 20 cm]
6 × 1014 Hz travel from air to glass of µ = 1.5. 44. A convex refracting surface of radius of curvature
[Ans. (i) 20 cm separates two media of refractive indices 4/3
2 × 108 m/s; 3.3 × 10−7 m (ii)3 × 108 m/s; 5 × 10−7 m] and 1.60. An object is placed in the first medium
34. A ray of light is incident at an angle of 60° on one (µ = 4/3) at a distance of 200 cm from the refract-
face of a rectangular glass slab of thickness 0.1 m, and ing surface. Calculate the position of image formed.
refractive index 1.5. Calculate the lateral shift pro- [Ans. At 234.15 cm in denser medium]
duced. [Ans. 0.0513] 45. A sphere of glass (µ = .5) is of 20 cm diameter. A
TOTAL INTERNAL REFLECTIONS parallel beam enters it from one side. Where will it
get focussed on the other side ? [Ans. 5 cm]
35. State the conditions for the phenomenon of total in- 46. A beam of light strikes a glass sphere of diameter 15
ternal reflection to occur. cm converging towards a point 30 cm behind the pole
of the spherical surface. Find the position of the im-
36. In Fig., find the maximum angle i for which light suf- age, if µ of glass is 1.5. [Ans. 15 cm from the pole]
fers total internal reflection at the vertical surface.
[Ans. 48.6°] 47. One end of a horizontal cylindrical glass rod (µ = 1.5)
of radius 5 cm is rounded in the shape of a hemi-
37. A small bulb (assumed to be a point source) is placed sphere. An object 0.5 mm high is placed perpendicular

m
at the bottom of a tank containing water to a depth to the axis of the rod at a distance of 20 cm from the
of 80 cm. Find out the area of the surface of wa- rounded edge. Locate the image of the object and find
ter through which light from the bulb can emerge. its height. [Ans. 30 cm ; 0.5 mm]
Take the value of refractive index of water to be 4/3.
48. An air bubble is formed inside water. Does it act as a
[Ans. 25837.7 sq. cm]
38. A point source of monochromatic light ’S’ is kept at
the centre of the bottom of a cylinder of radius 15 cm.
The cylinder contains water (refractive index 4/3) to
ra converging lens or a diverging lens? [Ans. diverging]
49. A convex lens is placed in contact with a plane mirror.
A point object at a distance of 20 cm on the axis of
this combination has its image coinciding with itself.
G
a height of 7 cm. Draw the ray diagram and calcu-
late the area of water surface through which the light What is the focal length of the lens? [Ans. 20 cm]
emerges in air. [Ans. 198 cm2 ] 50. A point object is placed at the centre of a glass sphere
39. A right prism is to be made by selecting a proper ma- of radius 6 cm and refractive index 1.5. What is the
u
terial and the angles A and B (B ¡ A), as shown in Fig.. distance of the virtual image from the surface of the
It is desired that a ray of light incident normally on sphere? [Ans. 6 cm]
AB emerges parallel to the incident direction after two 51. What will be the minimum length of a plane mirror
Ed

internal reflections. (a) What should be the minimum required to view the full image of a person 6 feet tall?
refractive index µ for this to be possible ? (b) For µ [Ans. 3 ft]
= 5/3, is it possible to achieve
√ this with the angle A LENS MAKER’S FORMULA
equal to 60 degrees ? [Ans. 2; No]
40. ln Fig., light rays of blue, green and red wavelengths 52. The radii of curvature of a double convex lens of glass
are incident on an isosceles right angled prism. Ex- (µ = 1.5) are in the ratio of 1 : 2. This lens renders
plain with reason which ray of light will be transmitted the rays parallel coming from an illuminated filament
through the face AC. The refractive index of the prism at a distance of 6 cm. Calculate the radii of curvature
for red, Green and blue light are 1.39, 1.424 and 1.476 of its surfaces. [Ans. 4.5 cm ; - 9 cm]
respectively. [Ans. Red rays will be transmitted]
53. A converging lens has a focal length of 20 cm in air.
41. Velocity of light in a liquid is 1.5 X 108 m/s and in lt is made of a material of refractive index 1.6. lf it is
air, it is 3 × 108 m/s. If a ray of light passes from immersed in a liquid of refractive index l.3, what will
this liquid to air, calculate the value of critical angle. be its new focal length? [Ans. 52 cm]
[Ans. 30°]
54. The radii of curvature of each surface of a convex
REFRACTION AT SPHERICAL SURFACES lens is 20 cm and the refractive index of the mate-
rial of the lens is 3/2 (i) Calculate its focal length (ii)
42. A small air bubble in a glass sphere of radius 2 cm If this is cut along the plane AB. What will be the
appears to be l cm from the surface when looked at. focal length of the each of the two halves so formed
along diameter. If the refractive index of glass is l.5, ? (iii) What happens if the lens is cut along CD?
find the true position of the air bubble. [Ans. 1.2 cm] [Ans. 20 cm ;40 cm ; 20 cm]

83 - Nikhil Kamboj — 8193989452 #edugramddun


Ray Optics and Optical Instruments

55. A convex lens made up of glass of refractive index 1.5 dex l.7, calculate the focal length of the lens in liquid.
is dipped in turn (i) in a medium of refractive index [Ans. - 63.75 cm]
1.65 (ii) in a medium of refractive index 1.33.
66. A converging lens of refractive index 1.5 is kept in a
(a) Will it behave as converging or diverging lens in
liquid medium having same refractive index. What
the two cases ?
would be the focal length of the lens in this medium?
(b) How will its focal length change in the two media?
[Ans. infinite (∞)]
[Ans. (a) diverging lens in medium of ref. index 1.65;
converging lens in medium of ref. index 1.33 (b) f1 = LENS FORMULA, LINEAR MAGNIFICATION
— 5.5f , f2 = 3.9f ] AND POWER
56. A converging lens of refractive index 1.5 and of focal
67. From the ray diagram shown in Fig. calculate the focal
length 15 cm in air, has the same radii of curvature for
length of concave lens. [Ans. -20 cm]
both sides. If it is immersed in a liquid of refractive
index l.7, find the focal length of the lens in the liquid. 68. A convex lens is used to throw on a screen l0 m from
[Ans. - 63.75 cm] the lens, a magnified image of an object. If the mag-
57. The radii of curvature of the surfaces of a double con- nification is to be l9, find the focal length of the lens.
vex lens are 20 cm and 30 cm. What will be its focal [Ans. 0.5 m]
length and power in air and water respectively ? Re- 69. A screen is placed 80 cm from an object. The image of
fractive indices for glass and water are 3/2 and 4/3 the object on the screen is formed by a convex lens at
respectively. [Ans. 24 cm, 4.17 D ; 96 cm, 1.04 D] two different locations separated by 10 cm. Calculate
58. A convex lens made up of glass of refractive index 1.5 the focal length of the lens used. [Ans. 19.7 cm]

m
is dipped, in tum in (i) medium A of refractive index 70. The image obtained with a convex lens is erect and
l.65, (ii) medium B of refractive index l.33. Explain its length is four times the length of the object. If the
giving reasons, whether, it will behave as a converg- focal length of the lens is 20 cm. Calculate the object
ing lens or a diverging lens in each of these media. and image distances. [Ans. -15 cm, — 60 cm]
[Ans. (i) diverging (ii) converging]
59. A double convex lens of glass of refractive index 1.6
has its both surfaces of equal radii Of curvature of 30
cm each. An object of height 5 cm is placed at a dis-
ra 71. An illuminated object and a screen are placed 90 cm
apart. Determine the focal length and nature of the
lens required to produce a clear image on the screen
twice the size of the object. [Ans. 20 cm, convex lens]
G
tance of 12.5 cm from the lens. Calculate the size of
the image formed. [Ans. 10 cm] 72. The radius of curvature of the faces of a double con-
vex lens are l0 cm and 15 cm. If focal length of lens is
60. Convex lens is made of glass of refractive index 1.5. If 12 cm, find the refractive index of the material of the
the radius of curvature of each of the two surfaces is
u
lens. [Ans. 1.5]
20 cm, find the ratio of the powers of the lens, when
placed in air to its power, when immersed in a liquid 73. A biconvex lens has a focal length 2/3 times the radius
of curvature of either surface. Calculate the refractive
Ed

of refractive index l.25. [Ans. 5/2]


index of the lens material. [Ans. 1.75]
61. A convex lens of focal length 20 cm and made of glass
(µ = 1.5) is immersed in water of µ = 1.33. Calculate 74. How does the power of a convex lens vary, if
change in focal length of the lens. [Ans. 58.23 cm] the incident red light is replaced by violet light?
[Ans. increase]
62. A thin converging lens made of glass of refractive in-
dex 1.5 acts as a concave lens of focal length 50 cm, COMBINATION OF LENSES
when immersed in a liquid of refractive index 15/8.
Calculate the focal length of converging lens in air. 75. Find the focal length and power of a convex lens,
[Ans. 20 cm] which when placed in contact with a concave lens of
63. Find the radius of curvature of the convex surface focal length 25 cm forms a real image 5 times the
of a plano convex lens, whose focal length is 0.3 m size of the object placed 20 cm from the combination.
and refractive index of the material of the lens is 1.5. [Ans. 10 cm, 10 D]
[Ans. 0.15 m] 76. Find the focal length and nature of lens which should
64. A converging lens has a focal length of 20 cm in air. be placed in contact with a lens of focal length 10 cm
It is made of a material of refractive index 1.6. If it is so that the power of the combination becomes 5 diop-
immersed in a liquid of refractive index 1.3, what will tre. [Ans. - 20 cm; concave]
be its new focal length ? [Ans. 52 cm]
77. Two lenses, one diverging of power 2 dioptre and the
65. A converging lens of refractive index 1.5 and focal other converging of power 6 dioptre are combined to-
length 15 cm in air has same radii of curvature for gether. Calculate focal length and power of the com-
both sides. It is immersed in a liquid of refractive in- bination. [Ans. 25 cm, 4D]

84 - Nikhil Kamboj — 8193989452 #edugramddun


Ray Optics and Optical Instruments

78. Two lenses of power + 10 D and - 5 D are placed in angle 60°. Calculate the angle of deviation and angle
contact, (i) Calculate the focal length of the combi- of incidence. [Ans. 37.2°, 48.6°]
nation (ii) where should an object be held from the 90. A glass prism has a refracting angle of 60°. The an-
combination so as to obtain a virtual image of magni- gle of minimum deviation is 40°. If velocity of light
fication 2 ? [Ans. (i) 20 cm (ii) - 10 cm] in vacuum is 3 × 108 m/s. Calculate the velocity
COMBINATION OF LENS AND MIRROR of light in glass. What is the angle of incidence ?
[Ans. 1.958 × 108 m/s, 50◦ ]
79. A point object is placed 60 cm in front of a convex lens 91. Show that the angle of deviation produced by thin
of focal length 15 cm. A plane mirror is placed 10 cm prism is reduced to one fourth (w.r.t. air) when it is
behind the convex lens. Where is the image formed by immersed in water. Given a µg = 3/2 and a µw = 4/3.
the system? [Ans. At optical centre of convex lens] 92. The refractive index of the
√ material of a prism of 60°
80. A convex lens of focal length 15 cm, and a concave mir- angle for yellow light is 2. Calculate angle of mini-
ror of radius of curvature 20 cm are placed co-axially mum deviation, angle of incidence and angle of refrac-
10 cm apart. An object is placed in front of convex tion. [Ans. 30°, 45°, 30°.]
lens so that there is no parallax between the object
93. A glass prism whose refractive index is 1.53 and re-
and its image formed by the combination. Find the
fracting angle is 60° is held in a liquid of refractive
position of the object. [Ans. At 30 cm from lens]
index 1.33. Calculate the angle of minimum deviation
81. Fig. shows a plane mirror M placed at a distance of 10 in this case. [Ans. 10° 12’]
cm from a concave lens L. A point object is placed at a
94. A ray of light is inclined to one face of the prism at
distance of 60 cm from the lens. The image formed due

m
an angle of 60°. If angle of prism is 60° and the ray is
to refraction by the lens and reflection by the mirror
deviated through an angle of 42°, find the angle which
is 30 cm behind the mirror. What is the focal length
the emergent ray makes with the second face of the
of this lens ? [Ans. - 30 cm]
prism. [Ans. 18°]
82. If a convex lens of focal length 80 cm and a concave
lens of focal length 50 cm are combined together, what
will be their resulting power? [Ans. – 0.75 D]
83. A convex lens and a concave lens, each having the
same focal length of 25 cm, are put in contact to form
ra 95. A glass prism has a refracting angle of 60°. The an-
gle of minimum deviation is 40°. Find the refractive
index. At what angle should the ray be incident so as
to suffer minimum deviation ? [Ans. 1.532, 50°]
G
96. A glass prism of angle 72° and refractive index 1.66 is
a combination of lenses. What will be the power of
immersed in a liquid of µ = 1.33. Calculate the angle
the combination (in dioptres).
of minimum deviation. [Ans. 22.38°]
[Ans. Zero]
97. The refractive indices of a prism for red, violet and
u
84. Two lenses of focal lengths 20 cm and – 40 cm are held
yellow lights are l.52, 1.62 and 1.59 resp. What is
in contact. Where will be image of an object at infin-
the dispersive power of the prism? If mean deviation
ity will be formed by the combination?
is 40°. What is angular dispersion produced by the
Ed

[Ans. 40 cm]
prism? [Ans. 0.169 ; 6°76”]
PRISM
98. Find the angle of flint glass prism which produces the
same angular dispersion for c and F wavelengths in
85. (a) Deduce the expression, by drawing a suitable ray 10° crown glass prism.
diagram, for the refractive index of triangular glass For crown glass : µf = 1.53230, µc = 1.5145
prism in terms of the angle of minimum deviation (D) For flint glass : µ′f = 1.6637, µ′c = 1.6444. [Ans. 4.4°]
and the angle of prism (A).
99. The refractive indices of crown and flint glasses for
(b) Draw a plot showing the variation of the angle of
violet and red light are 1.523, 1.513, 1.773 and 1.743
deviation with the angle of incidence.
respectively. Find the dispersive powers of the glasses.
86. The refractive
√ index of the material of an equilateral [Ans. 0.0191 0.04]
prism is 3. What is the angle of minimum deviation?
[Ans. 60°, At minimum deviation position] 100. The minimum deviations suffered by red, Yellow
and violet beams passing through an equilateral
87. How does the angle of minimum deviation of a glass transparent prism are 38.4°, 38.7° and 39.2° respec-
prism vary, if the incident violet light is replaced with tively. Calculate the dispersive powers of the medium.
red light? [Ans. decreases] [Ans. 0.018]
88. Calculate the refractive index of the material of an 101. Determine the angle of flint glass prism, which should
equilateral prism for
√ which angle of minimum devia- be combined with a crown glass prism of 5° so as to
tion is 60◦ . [Ans. 3] give dispersion, but no deviation. Given
89. A ray of light suffers minimum deviation, while passing For crown glass : µv = 1.5323, µr = 1.515
through a prism of refractive index 1.5 and refracting For flint glass : µ′v = 1.688, µ′r = 1.650. [Ans. -3.88◦ ]

85 - Nikhil Kamboj — 8193989452 #edugramddun


Ray Optics and Optical Instruments

102. Calculate the angle of a prism of dispersive power 0.02l w. Calculate distance between objective and eye piece.
and refractive index 1.53 to form an achromatic com- [Ans. 12.5 cm]
bination with prism of angle 4.2°, and dispersive power 113. A compound microscope uses an objective lens of
0.045, having refractive index 1-65. Find also the net focal length 4 cm and eye lens of focal length l0
deviation. [Ans. — 11.04°, — 3.12°] cm. An object is placed at 6 cm from the objective
DEFECTS OF VISION lens. Calculate the magnifying power of compound
microscope. Also, calculate the length of microscope.
103. The far point of a myopic person is 150 cm in front [Ans. 7 ; 22 cm]
of the eye. Calculate the focal length and power of 114. The total magnification produced by a compound mi-
the lens required to enable him to see distant objects croscope is 20, while that produced by eye piece alone
clearly. [Ans. - 1.5 m, - 0.67 D] is 5. When the microscope is focussed on a cer-
tain object. the distance between objective and eye
104. A short sighted person is wearing specs of power
piece is l4 cm. If distance of distinct vision is 20cm,
- 3.5 D. His doctor prescribes a correction of +
what are the focal lengths of objective and eye piece ?
2.5 D for his near vision. What is focal length
[Ans. 2 cm ; 5 cm]
of his distance viewing pan and near vision pan ?
[Ans. - 28.5 cm ; l6.7 cm] TELESCOPE
105. A person cannot see the objects distinctly. when
placed at a distance less than 100 cm. What is the 115. Draw a ray diagram of an astronomical telescope in
power of the spectacles that he should use to see the normal adjustment position. State two drawbacks
clearly, the objects placed at 25 cm? [Ans. + 3 D] of this type of telescope.

m
106. The distance of distinct vision of a person is 50 cm. He 116. The magnifying power of an astronomical telescope
wants to read a book placed at 25 cm. What should in the normal adjustment position is I00. The dis-
be the focal length of the spectacles? [Ans. + 50 cm] tance between the objective and eye piece is 101 cm.
Calculate the focal lengths of objective and eye piece.
107. A person has nominal for point (infinity) and nor-
mal near point (25 cm). He intends to read a book
ra [Ans. 100 cm and 1 cm]
using a magnifying glass of f = 5 cm. What is 117. A refracting telescope has an objective of focal length
the (i) Closest and (ii) farthest distance at which l m and an eye piece of focal length 20 cm. The final
image of the sun 10 cm in diameter is formed at a dis-
he can read the book through the magnifying glass.
G
[Ans. - 4.l7 cm ; - 5 cm] tance of 24 cm from eye piece. What angle does the
sun subtend at the objective? [Ans. 0.0455 rad]
MICROSCOPE
118. A giant refracting telescope at an observatory has an
objective lens of focal length 15 m. If an eye piece lens
u
108. Draw course of rays through a compound microscope. of focal length 1 cm is used, find the angular magnifi-
Deduce an expression for its magnifying power. How cation of the telescope.
can the magnifying power be increased?
Ed

If this telescope is used to view the moon, what is the


109. An object is to be seen through a simple microscope diameter of image of moon formed by objective lens?
of power l0 D. Where should an object be placed to The diameter of the moon is 3.42 × 106 m and radius
produce maximum angular magnification ? Least dis- of lunar orbit is 3.8 × 108 m. [Ans. 1500 ; 13.5 cm]
tance of distinct vision is 25 cm. [Ans. - 7.1 cm] 119. A refracting telescope has an objective of focal length
110. The focal lengths of the objective and eye piece Of a 30 cm and an eye piece of focal length 3 cm. It is fo-
microscope are 2 cm and 5 cm respectively, find the cussed on a scale distant 2 m. For seeing with relaxed
distance between them is 20 cm. Find the distance of eye, calculate the separation between the objective and
the object from the objective when the final image seen eye piece. [Ans. 37.97 cm]
by the eye is 25 cm from the eye piece. What is the 120. A telescope consists of two lenses of focal lengths 20
magnifying power? [Ans. µ0 = −2.3cm, m = 41.5] cm and 5 cm. Obtain its magnifying power when fi-
111. The focal lengths of the eye piece and objective of a nal image is (i) at infinity (ii) a 25 cm from the eye.
compound microscope are 5 cm and l cm respectively, [Ans. (i) - 4 (ii) — 4.8]
and the length of the tube is 20 cm. Calculate mag- 121. A reflecting type telescope has a concave reflector of
nifying power of microscope when the final image is radius of curvature 120 cm. Calculate focal length of
formed at infinity. The least distance of distinct vi- eye piece to secure a magnification of 20. [Ans. 3 cm]
sion is 25 cm. [Ans. 70]
122. How would you combine two lenses of focal
112. A compound microscope is made using a lens of focal lengths 25 cm and 2.5 cm to make a telescope?
length l0 mm as objective and another lens of focal What is the magnifying power of this telescope?
length 15 mm as eye piece. An object is held at 1.1 [Ans. Lenses should be held at a distance of 27.5 cm
cm from the objective and final image is obtained at from each other, M = - 10]

86 - Nikhil Kamboj — 8193989452 #edugramddun


Ray Optics and Optical Instruments

123. Two boys, one 52 inches tall and the other 55 inches 125. The image of the moon is focussed by a converging
tall, are standing at distances 4 m and 5 m respec- lens of focal length 50 cm on a plane screen. The im-
tively from an eye. Which boy will appear taller? age is seen by an unaided eye from a distance of 25 cm
[Ans. First boy] Find the angular magnification achieved due to the
124. The angular magnification of a telescope is 300. What converging lens? [Ans. 2]
should be the diameter of the objective, if our eyes at 126. Two lenses of focal lengths ± 15 cm and ± 150 cm
the eye ring, are just able to collect all the light re- are available for making a telescope. To produce the
fracted from the objective. Take diameter of pupil of largest magnification, What should be the focal length
eye = 3 mm. [Ans. 90 cm] of the eyepiece should be? [Ans. + 15 cm]

m
ra
u G
Ed

87 - Nikhil Kamboj — 8193989452 #edugramddun


Wave Optics
Important Formulae and Notes

▶ Constructive interference (d) Angular position of nth bright band is given by,

(a) Resultant amplitude = a1 + a2 θ=
d
(b) Resultant intensity p
p (e) Distance between two virtual sources d = d1 d2
(Imax = (a1 + a2 )2 = ( (I1 + I2 )2
p

= I1 + I2 + 2I1 I2 ▶ Distance between two virtual images of the slits in bi


(c) Phase difference prism experiment d = 2u(µ − 1)α
ϕ = 0, 2π, 4π, ..., n(2π) where n = 0, 1, 2, ... ▶ Diffraction from a single slit (Fraunhofer diffraction)
(d) Path difference
(a) Angular position of the nth secondary minimum,
x = 0, λ, 2λ, ..., nλ, where n = 0, 1, 2, 3, ...

▶ Destructive interference θn =
a
(a) Resultant amplitude = a1 − a2 (b) Distance of the nth secondary minimum from the
(b) Resultant intensity nDλ
centre of screen, Yn =
a
p
(Imin = (a1 − a2 )2 = ( (I1 − I2 )2
p

= I1 + I2 − 2I1 I2 (c) Angular positions of the nth secondary maxi-


(2n + 1)λ
(c) Phase difference mum, θn′ =

m
ϕ = π, 3π, 5π, ..., (2n − 1)π where n = 1, 2, ... 2a
(d) Distance of nth secondary maximum from the
(d) Path difference
λ 2λ λ centre of the screen,
x= , , , ..., (2n − 1) ,
2 2 2 (2n + 1)Dλ
▶ Average intensity =


Imax
Imin
=
(a1 + a2 )
(a1 − a2 )2
2
Imax + Imin
2
ra yn′ =
2a

(e) Width of a secondary maximum or minimum,


β=

G
2
√ a

a1
+ 1 I1 + I2 + 2 I1 I2
=
a2 2Dλ
2 = √ (f) Width of central maximum, β0 =
I1 + I2 − 2 I1 I2 a
a1
a2 − 1
▶ Law of Malus, I = I0 cos θ 2
u
Imax − Imin
▶ Band visibility = where I = intensity of light transmitted from analyser
Imax + Imin I0 = intensity of incident plane polarised light
λD θ = angle between plane of transmission of analyser
Ed

▶ Band width, x =
d and polarise
where d = distance between two slits D = distance of
screen from slits ▶ Brewster’s law, µ = tan ip
where µ = refractive index ip = polarising angle
▶ Young’s Double slit experiment
xd θ
(a) λ = ▶ Specific rotation, S =
D lc
′ x where l = length of solution
(b) New band width, x = c = concentration of solution
µ
x λ θ = rotation produced in the plane of polarisation of
(c) Angular band width θ = = light by the solution.
D d
Multiple Choice Questions

1. Resolving power of telescope can be increased by in- a) corpuscular nature of light.


creasing b) quantum nature of light.
c) transverse wave nature of light.
a) the wavelength
d) longitudinal wave nature of light.
b) the diameter of objective
c) the diameter of eyepiece
d) the focal length of 3. The wavefront due to a source situated at infinity is
a) spherical b) cylindrical
2. Polarisation of light proves c) planar d) circular
Wave Optics

4. A laser beam is coherent because it contains 13. When unpolarised light beam is incident from air onto
a) incoherent waves of several wavelengths. glass (n = 1.5) at the polarising angle.
b) incoherent waves of a single wavelength. a) Reflected beam is polarised completely
c) coherent waves of several wavelengths b) Reflected and refracted beams are partially
d) coherent waves of a single wavelength polarised
c) Refracted beam is plane polarised
5. If two sources have a randomly varying phase differ- d) Whole beam of light is refracted
ence F(t), the resultant intensity will be given by
√ 14. Resolving power of microscope depends upon
I0
a) I0 2 b) a) wavelength of light used (directly proportional)
2
I0 b) wavelength of light used (inversely proportional)
c) 2I0 d) √
2 c) frequency of light used
d) focal length of objective
6. According to Huygens’ principle, light is a form of
15. The phenomenon of interference is based on
a) particle b) rays
c) wave d) radiation a) conservation of momentum.
b) conservation of energy.
7. Two coherent monochromatic light beams of intensi- c) conservation of momentum and energy.
ties I and 41 superimpose. The maximum and mini- d) quantum nature of light
mum possible intensities in the resulting beam are:
16. A double slit interference experiment is carried out in

m
a) 5I and I b) 5I and 3I air and the entire arrangement is dipped in water. The
c) 3I and I d) 9I and I fringe width
a) increases
8. What is path difference for destructive interference?
b) decreases
a) nl
c) (2n + 1)
λ
2
b) n(l +1)
d) (n + 1)
λ
2
ra c)
d)
remains unchanged.
fringe pattern disappears

17. In Young’s double slit experiment, if the monochro-


G
9. When exposed to sunlight, thin films of oil on water matic source of yellow light is replaced by red light,
of ten exhibit brilliant colours due to the phenomenon the fringe width
of
a) increases b) decreases.
a) interference b) diffraction c) remains unchanged. d) the fringes disappear
u
c) dispersion d) polarisation
18. In Young’s double-slit experiment, the intensity at the
10. What happens, if the monochromatic light used in central maximum is I0 if one of the slit is covered, then
Ed

Young’s double slit experiment is replaced by white the intensity at the central maximum become:
light? I0 I0
a) b) √
a) No fringes are observed. 2 2
I0
b) All bright fringes become while. c) d) I0
c) All bright fringes have colour between violet and 4
red. 19. The angle of incidence at which reflected light is totally
d) Only the central fringe is white and all other polarised for reflection from air to glass (refractive in-
fringes are coloured dex n) is
1
 
11. When compact disk is illuminated by a source of white a) sin (n)
−1
b) sin −1
n
light, coloured lines are observed. This is due to 1
 
a) dispersion b) diffraction c) tan −1
d) tan (n))
−1
n
c) interference d) refraction
20. In Young’s double-slit experiment, the intensity is I at
a point, where the path difference is (l – wavelength
12. An unpolarised beam of intensity I0 is incident on a λ
pair of nicols making angle of 60° with each other. The 6
of light used). If I0 denotes the maximum intensity
intensity of right emerging from the pair is I
I0 then is equal to
a) I0 b) I0
2 √
I0 I0 3 1 3 1
c) d) a) b) c) d) √
4 8 2 2 4 2

89 - Nikhil Kamboj — 8193989452 #edugramddun


Wave Optics

21. Which of the following is correct for light diverging 29. The angular resolution of a 10 cm diameter telescope
from a point source? at a wavelength of 5000 Å” is of the order of
a) The intensity decreases in proportion with the a) 106 rad b) 10−2 rad
distance squared. c) 10−4 rad d) 10−6 rad
b) The wavefront is parabolic.
c) The intensity at the wavelength does not depend 30. The velocity of light in air is 3 × 108 m/s and that
on the distance. in water is 2.2 × 108 m/s . The polarising angle of
d) None of these. incidence is
22. The refractive index of glass is 1.5 for light waves of X a) 45° b) 50° c) 53.74° d) 63°
= 6000 A in vacuum. Its wavelength in glass is
a) 2000 Å b) 4000 Å 31. An optically active compound
c) 1000 Å d) 3000 Å a) rotates the plane of polarised light
b) changes the direction of polarised light
23. The phenomena which is not explained by Huygen’s c) does not allow plane polarised light to pass
construction of wavefront through
a) reflection b) diffraction d) none of these
c) refraction d) origin of spectra
32. Consider sunlight incident on a slit of width 104 A.
24. A laser beam is used for locating distant objects be- The image seen through the slit shall

m
cause
a) be a fine sharp slit white in colour at the centre.
a) it is monochromatic b) a bright slit white at the centre diffusing to zero
b) it is not chromatic intensities at the edges
c) it is not observed c) a bright slit white at the centre diffusing to re-
d) it has small angular spread.

25. Two slits in Young’s double slit experiment have


widths in the ratio 81 :1. The ratio of the amplitudes
of light waves is
ra gions of different colours.
d) only be a diffused slit white in colour. [NCERT
Exemplar]
G
33. In a Young’s double slit experiment, the source is white
a) 3 : 1 b) 3 : 2 c) 9 : 1 d) 6 : 1 light. One of the holes is covered by a red filter and
another by a blue filter. In this case
26. When interference of light takes place
a) there shall be alternate interference patterns of
u
a) energy is created in the region of maximum
red and blue.
intensity
b) there shall be an interference pattern for red dis-
b) energy is destroyed in the region of maximum
tinct from that for blue.
Ed

intensity
c) there shall be no interference fringes.
c) conservation of energy holds good and energy is
d) there shall be an interference pattern for red mix-
redistributed
ing with one for blue. [NCERT Exemplar]
d) conservation of energy does not hold good

27. In a double slit interference pattern, the first maxima 34. Consider the diffraction pattern for a small pinhole.
for infrared light would be As the size of the hole is increased

a) at the same place as the first maxima for green a) the size decreases.
light b) the intensity decreases.
b) closer to the centre than the first maxima for c) the size increases.
green light d) the intensity decreases.
c) farther from the centre than the first maxima for
green light 35. For light diverging from a point source
d) infrared light does not produce an interference a) the wavefront is spherical.
pattern b) the intensity increases in proportion to the dis-
tance squared.
28. To observe diffraction, the size of the obstacle c) the wavefront is parabolic.
a) should beX/2, where X is the wavelength. d) the intensity at the wavefront does not depend
b) should be of the order of wavelength. on the distance.
c) has no relation to wavelength.
d) should be much larger than the wavelength. 36. Plane of polarisation is ........ to plane of vibration.

90 - Nikhil Kamboj — 8193989452 #edugramddun


Wave Optics

π
a) Parallel b) perpendicular a) b) 0
c) at 45 degree d) none of above 2
c) π d) none of these

37. For a destructive interference, phase difference be- 39. What is the effect on the interference fringes in a
tween wave is Young’s double slit experiment, if the width of the
a) (2n + 1)p b) (2n – 1)/p source slit is increased slightly?
c) (2n + 1)/p d) (2n – 1)p a) the fringe pattern gets less and less sharp
b) the fringe pattern gets more and more sharp
38. What is a phase difference between two points on the c) more constructive interference will take place
same wavefront? d) none of above

Practice Questions and Numerical

HUYGEN’S PRINCIPLE if the light source is replaced by another monochro-


matic source of wavelength 7500 Åand the separation
1. (a) State Huygens’s principle. between the slits is doubled? [Ans. 0.5 mm]
12. The fringe width in YDSE is 2.4 × 10−4 m, when red
2. What type of wavefront will emerge from a
light of wavelength 6400 Åis used. By how much will
(i) point source, and (ii) distant light source?
it change, if blue light of wavelength 4000 Åis used?
[Ans. (i) Point source – Spherical wavefront (ii)
[Ans. 9 × 10−5 ]

m
Distant light source – Plane wavefront.]
13. In Young’s double slit experiment, using light of wave-
3. Differentiate between a ray and a wave front.
length 400 nm, interference fringes of width X are ob-
4. The optical path of monochromatic light is same if it tained. The wavelength of light is increased to 600
travels 2 cm thickness of glass or 2.25 cm of thickness nm and the separation between the slits is halved. If

refractive index of water? [Ans. 1.33]


5. Red light of wavelength 750 nm enters a glass
plate of refractive index 1.5. If velocity of
ra
of water. If refractive index of glass is 1.5, what is the one wants observed fringe width in the screen to be
the same in the two cases, find the ratio of distance
between the screen and the plane of the interfering
sources in the two arrangement. [Ans. 3 : 1]
G
light in vacuum is 3 × 108 m/s, Calculate ve- 14. In YDSE, the width of the fringes obtained with light
locity, wavelength and frequency of light in glass. of wavelength 6000 Åis 2 mm. Calculate the fringe
[Ans. 2 × 108 m/s; 500 nm; 4 × 1014 Hz] width if the entire apparatus is immersed in a liquid
medium of refractive index µ = 1.33. [Ans. 1.5 mm]
u
INTENSITIES OF MAXIMA AND MINIMA
15. The two slits in Young’s double slit experiment are
separated by a distance of 0.03 mm. When light of
6. Find the ratio of intensities at two points on a screen in
Ed

wavelength 5000 Åfalls on the slits, an interference


young’s double slit experiment, when waves from two pattern is produced on the screen 1.5 m away. Find
slits have path diff. of (i) 0 and (ii) λ/4. [Ans. 2 : 1] the distance of fourth bright fringe from the central
7. The ratio of intensities at maxima and minima is 25:16. maximum. [Ans. 0.1 m]
What will be the ratio of the widths of two slits in 16. A double slit is illuminated by light of λ = 6000 Å.
YDSE. [Ans. 81:1] The slits are 0.1 cm apart and the screen is placed 1
8. The intensity ratio in the interference pattern is 1:9. m away. Calculate (i) angular position of 10th maxi-
What is the amplitude ratio and the ratio of widths of mum in radian (ii) separation of two adjacent minima.
two slits? [Ans. 2:1, 4:1] [Ans. 6 × 10−3 rad (ii) 6 × 10−4 m]
9. Two interfering sources have an intensity ratio 16 : 17. In a Young’s double slit experiment, the angular width
l. Deduce amplitude ratio and ratio of intensity be- of a fringe formed on distant screen is 0.1°. The wave-
tween the maxima and minima in interference pattern. length of light used is 6000 Å. What is the spacing
[Ans. 4:1; 25:9] between the slits? [Ans. 3.44 × 10−4 ]
18. In Young’s experiment, two coherent sources are 1.5
10. The ratio of the intensities at minima to the maxima
mm apart and the fringes are obtained at a distance
in YDSE is 9 : 25. Find the ratio of widths of two
of 2.5 m from them. If the sources produce light of
slits. [Ans. 16 : 1]
wavelength 589.3 nm, find the number of fringes in
YOUNG’S DOUBLE SLIT EXPERIMENT the interference pattern, which is 4.9 × 10−3 m long.
[Ans. 5]
11. Yellow light of wavelength 6000 Åproduces fringes of 19. A central fringe of interference pattern produced by
width O.8 mm in YDSE. What will e the fringe width light of wavelength 6000 Åis shifted to the position of

91 - Nikhil Kamboj — 8193989452 #edugramddun


Wave Optics

5th bright fringe by introducing thin film of pt = l.5. 29. A slit of width d is illuminated by a monochro-
Calculate thickness of the film. [Ans. 6 micron] matic light of wavelength 700 nm at normal inci-
dence. Calculate the value of d for position of
20. In Young’s double slit experiment, monochromatic
(i) first minimum at an angle of diffraction of 30°
light of wavelength 630 nm illuminates the pair of
(ii) first maximum at an angle of diffraction of 30°.
slits and produces an interference pattern in which
[Ans. 14 × 10−7 m ; 21 × 10−7 m]
two consecutive bright fringes are separated by 8.1
mm. Another source of monochromatic light pro- 30. Light of wavelength 600 nm is incident normally on a
duces the interference pattern in which the two con- slit of width 3 mm. Calculate linear width of central
secutive bright fringes are separated by 7.2 mm. maximum on a screen kept 3 m away from the slit.
Find the wavelength of light from second source. [Ans. 1.2 mm]
What is the effect on interference fringes if monochro-
31. A parallel beam of light of wavelength 600 nm is inci-
matic source is replaced by a source of white light ?
dent normally on a slit of width d. If distance between
[Ans. 560 nm ; coloured fringes with white centre]
slit and screen is 0.8 m and distance of 2nd order mini-
21. In YDSE, light of wavelength 5000 Åis used. The mum from the centre of the screen is 9.6 mm, calculate
third bright band on the screen is formed at a dis- the width of the slit. [Ans. 0.1 mm]
tance of 1 cm from the central bright band. If the
screen is at a distance of 1.5 m from the centre of nar- 32. A plane wavefront (λ = 6 × 10−7 m) falls on a slit 0.4
row slits, calculate the separation between the slits. mm wide. A convex lens of focal length 0.8 m placed
[Ans. 2.25 ×10−4 m] behind the slit focusses the light on a screen. What
is the linear diameter of (i) first minimum (ii) second
22. How does the fringe width of interference fringes maximum ? [Ans. 2.4 mm; 6 mm]

m
change, when the whole apparatus of Young’s ex-
periment is kept in a liquid of refractive index 3/4? 33. A slit of width ‘a’ is illuminated by red light of wave-
λD length 6500 Å. For what value of ‘a’ will the (i) first
[Ans. Fringe width β = ] minimum fall at an angle of diffraction of 30° (ii)
d

λ
the slits is increased? [Ans. decreases; as θ = ]
d
ra
23. How does the angular separation of interference fringes
change in Young’s experiment, if the distance between
first maximum fall at an angle of diffraction of 30◦ ?
[Ans. (i) 1.3 × 10−6 m; (ii) 1.9 × 10−6 m]
34. A 0.02 cm wide slit is illuminated at normal inci-
dence by light having wavelength 6 × 10−7 m. (i)
G
24. State the reason, why two independent sources of light Find the width of the central band maximum on the
cannot be considered as coherent sources. screen which is 1 m away from slit. (ii) What would
be the width of central maximum, if the apparatus
25. How would the angular separation of interference
is immersed in water whose refractive index is 4/3 ?
fringes in Young’s double slit experiment change when
u
[Ans. (i) 0.6 cm (ii) 0.45 cm]
the distance between the slits and screen is doubled?
[Ans. no change as j=l/d] 35. Two wavelengths of sodium light 590 nm and 596 nm
Ed

are used, in turn, to study the diffraction taking place


DIFFRACTION OF LIGHT BY A SINGLE
SLIT
at a single slit of aperture 2×10−4 m. The distance be-
tween the slit and screen is 1.5 m. Calculate the sepa-
26. A parallel beam of light of wavelength 600 nm is in- ration between the positions of first maxima of diffrac-
cident normally on a slit of width ‘d’. If the distance tion pattern obtained in the two cases. [Ans. 9 nm]
between the slit and screen is 0.8 m and distance of 36. A parallel beam of light of wavelength 500 nm falls on
2nd order maximum from the centre of the screen is a narrow slit and the resulting diffraction pattern is
15 mm, calculate the width of the slit. [Ans. 80 µm] observed on a screen 1 m away. The first minimum is
27. Light of λ = 550 nm is incident as parallel beam on at a distance of 2.5 mm from the centre of the screen.
a slit of width 0.1 mm. Find the angular width and Calculate the width of the slit. [Ans. 0.2 mm]
linear width of the principal maximum in the diffrac- 37. In a single-slit diffraction experiment, the
tion pattern on a screen at a distance of 1.1 m from width of the slit is made double the orig-
the slit. Which of these widths will not change if the inal width. How does this affect the size
screen were moved to a distance of 2.2 m from the slit and intensity of the central diffraction band.
? [Ans. 1.1 × 10−2 rad ; 12.1 mm, Former will not [Ans. size become half and intensity become four
change] times as β = Dλ/dandI ∝ a2 ]
28. Light of wavelength 500 nm falls from a distant source
DESCRIPTIVE QUES
on a slit 0.5 mm wide. Find the distance between the
two dark bands on either side of central maximum, if
diffraction pattern is observed on a screen at 2 m from 38. (b) What do you understand by Diffraction. Draw di-
the slit. [Ans. 4 mm] agram and explain diffraction pattern by a single slit.

92 - Nikhil Kamboj — 8193989452 #edugramddun


Wave Optics

39. For Diffraction due to single slit, draw a plot of in- the interference pattern in a double slit experiment?
tensity distribution and explain clearly why the sec- (ii) When a tiny circular obstacle is placed in the path
ondary maxima become weaker with increasing order of light from a distance source, a bright spot is seen
(n) of the secondary maxima. at the centre of the shadow of the obstacle. Explain,
why.
40. State two conditions required for obtaining coherent
sources. In Young’s arrangement to produce interfer- 43. (a) Write the conditions under which light sources can
ence pattern, show that dark and bright fringes ap- be said to be coherent.
pearing on the screen are equally spaced. (b) Why is it necessary to have coherent sources in
order to produce an interference pattern?
41. Write the distinguishing features between a diffraction
44. How is a wavefront defined? Using Huygen’s construc-
pattern due to a single slit and the interference fringes
tion draw a figure showing the propagation of a plane
produced in Young’s double slit experiment?
wave reflecting at the interface of the two media. Show
42. Answer the following questions : that the angle of incidence is equal to the angle of re-
(i) In what way is diffraction from each slit related to flection.

m
ra
u G
Ed

93 - Nikhil Kamboj — 8193989452 #edugramddun


Dual Nature of Matter and Radiation
Important Formulae and Notes
e
▶ Thomson’s method to measure of electron (c) Number of photons of wavelength λ emitted in t
m seconds from a lamp of power P ,
e E2
z = PTλ
m 2V B 2 n=
hc
where e = charge of electron
m = mass of electron ▶ Photoelectric effect
E = electric field (a) Work function W = hν0
B = magnetic field where h = Planck’s constant
V = potential difference v0 = threshold frequency
▶ Millikan’s method to measure charge of electron (b) Einstein’s equation of photoelectric effect
Charge s (A) Maximum Kinetic energy of the emitted
6πη 9ηv 3 photo electrons
q=
E 2(ρ − σ)g
1 1 1
 
where v = terminal velocity mv 2 = hc −
E = electric field 2 λ λ0
ρ = density of oil

m
σ = density of gaseous medium where m = mass of the ejected photo elec-
η = coefficient of viscosity of gaseous medium tron
▶ Photons λ = wavelength of the incident radiation
λ0 = threshold wavelength
photon,
E = hν =
hc
λ
ra
(a) If the frequency of radiation is v, then energy of 1
(B) mvmax
2
2
= hν − hν0
where ν0 = maximum velocity of photo elec-
tron
v = frequency of incident radiation
G
where c = velocity of light
λ = wavelength of radiation v0 = threshold frequency
h = Planck’s constant 1
(c) (A) Stopping potential, Vs =
(b) Momentum of the photon work function
1
u
p = mc =
E
=
h (B) eVs = mvmax2

c λ 2
Ed

Multiple Choice Questions

1. Light of a particular frequency ν is incident on a metal 4. When light is incident on a metal surface the maxi-
surface. When the intensity of incident radiation is in- mum kinetic energy of emitted electrons
creased, the photoelectric current a) vary with intensity of light
a) decreases b) vary with frequency of light
b) increases c) vary with speed of light
c) remains unchanged d) vary irregularly
d) sometimes increases and sometimes decreases
5. The maximum energy of electrons released in a pho-
tocell is independent of
2. The photoelectric effect is based on the law of conser-
vation of a) the frequency of the incident light
b) the intensity of the incident light
a) momentum b) energy
c) the nature of the cathode
c) angular momentum d) mass
d) All of the above
3. The photoelectric effect can be understood on the ba- 6. Einstein’s photoelectric equation states that hn = W
sis of + Ek . In this equation, Ek refers to the
a) wave theory of light only a) kinetic energy of all the emitted electrons
b) electromagnetic theory of light only b) mean kinetic energy of the emitted electrons
c) quantum theory of light only c) maximum kinetic energy of the emitted electrons
d) None of these d) minimum kinetic energy of the emitted electrons
Dual Nature of Matter and Radiation

7. In the photoelectric effect, electrons are emitted a) amplitude is higher b) frequency is higher
a) at a rate that is proportional to the amplitude c) wavelength is longer d) wavelength is shorter
of the incident radiation
b) with a maximum velocity proportional to the fre-
16. The energy of photon of wavelength l is
quency of the incident radiation a) cλ/h b) hλ/c c) hc/λ d) c/hλ
c) at a rate that is independent of the emitter
d) only if the frequency of the incident radiations is 17. A photo sensitive metal is not emitting photo-electrons
above a certain threshold value when irradiated. It will do so when threshold is
crossed. To cross the threshold we need to increase
8. The minimum energy required to eject an electron, a) intensity b) frequency
from the metal surface is called c) wavelength d) None of these
a) atomic energy b) mechanical energy
c) electrical energy d) work function 18. According to Einstein’s photoelectric equation, the
plot of the kinetic energy of the emitted photo elec-
9. The work function for photoelectric effect trons from a metal Vs the frequency of the incident
radiation gives as straight the whose slope
a) is different for different metals
b) is same for all metals a) depends both on the intensity of the radiation
c) depends upon the intensity of incident light and the metal used
d) depends upon the frequency of incident light b) depends on the intensity of the radiation
c) depends on the nature of the metal used

m
10. Photoelectric effect is the phenomenon in which d) is the same for the all metals and independent
a) photons come out of a metal when it is hit by a of the intensity of the radiation
beam of electrons
19. ...... and ...... led to understanding of atomic struc-
b) photons come out of the nucleus of an atom un-
tures. (i) Fresnel diffraction (ii) cathode rays (iii)
der the action of an electric field
c) electrons come out of a metal with a constant
velocity
d) which depends on the frequency and intensity of
ra X-rays (iv) electrons
a) (i) and (ii)
c) (i) and (iv)
b) (ii) and (iii)
d) (iii) and (iv)
G
incident light wave
20. Sodium and copper have work functions 2.3 eV and
11. A photoelectric cell is a device which 4.5 eV respectively. Then the ratio of the wavelengths
is nearest to
a) converts light into electricity
u
b) converts electricity into light a) 1 : 2 b) 4 : 1 c) 2 : 1 d) 1 : 4
c) stores light
d) stores electricity 21. A strong argument for the particle nature of cathode
Ed

rays is that they


12. In Heinrich Hertz’s experiment on the production of a) produce fluoroscence
electromagnetic waves by means of spark discharge, b) travel through vacuum
it was observed that high voltage sparks across the c) get deflected by electric and magnetic fields
detector loop were ....... when the emitter plate was d) cast shadow
illuminated by ........ light
a) degraded, infra-red b) degraded, ultraviolet 22. In an electron gun the control grid is given a negative
c) enhanced, ultraviolet d) enhanced, infra-red potential relative to cathode in order to
a) decelerate electrons
13. Which of the following shows particle nature of light? b) repel electrons and thus to control the number
a) Refraction b) Interference of electrons passing through it
c) Polarization d) Photoelectric effect c) to select electrons of same velocity and to con-
verge them along the axis
14. Einstein’s photoelectric equation is Ek = hν - F . In d) to decrease the kinetic energy of electrons
this equation Ek refers to
a) kinetic energy of all the emitted electrons 23. When the speed of electrons increase, then the value
b) mean kinetic energy of emitted electrons of its specific charge
c) maximum kinetic energy of emitted electrons a) increases
d) minimum kinetic energy of emitted electrons b) decreases
c) remains unchanged
15. A photon will have less energy, if its d) increases upto some velocity

95 - Nikhil Kamboj — 8193989452 #edugramddun


Dual Nature of Matter and Radiation

24. Photoelectric effect is the phenomenon in which a) the radiations which appeared to be coming from
a) photons come out of a metal when it is hit by a the anode
beam of electrons b) the radiation which appeared to be coming from
b) photons come out of the nucleus of an atom un- the cathode
der the action of an electric field c) the protons coming from the cathode
c) electrons come out of a metal with a constant d) the protons coming from the anode
velocity 33. The presently accepted value of charge/mass (e/m)is
d) which depends on the frequency and intensity of
a) 1.66 x 10−19 c/kg b) 9.1 x 1011 c/kg
incident light wave
c) 1.76 x 1011 c/kg d) 9.1 x 1019 c/kg
25. The photoelectrons emitted from a metal surface are 34. In which of the following, emission of electrons does
such that their velocity not take place?
a) is zero for all a) Thermionic emission b) X-rays emission
b) is same for all c) Photoelectric d) Secondary emission
c) lies between zero and infinity emission
d) lies between zero and a finite maximum
35. Photoelectric emmision occurs only when the incident
26. In which of the following, emission of electrons does light has more than a certain minimum
not take place? a) power b) wavelength
a) Thermionic emission b) X-rays emission c) intensity d) frequency

m
c) Photoelectric d) Secondary emission
emission 36. Which of the following when falls on a metal will emit
photoelectrons ?
27. Photoelectric emmision occurs only when the incident a) UV radiations b) Infrared radiation
light has more than a certain minimum
a) power
c) intensity
b) wavelength
d) frequency
ra c) Radio waves d) Microwaves

37. Particle like behavior of light arises from the fact that
each quanta of light has definite ...X... and a fixed
value of ...Y.. just like a particle, Here, X and Y refer
G
28. When light is incident on a metal surface the maxi- to
mum kinetic energy of emitted electrons
a) frequency, energy b) shape, volume
a) vary with intensity of light c) energy, frequency d) energy, momentum
b) vary with frequency of light
u

c) vary with speed of light 38. The wave nature of light was established by (i)
d) vary irregularly Maxwell’s equations (ii) Fraunhoffer’s lines (iii) Hertz
experiment (iv) Einstein’s theory
Ed

29. A photoelectric cell is a device which a) (i) and (ii) only b) (ii) and (iv) only
a) converts light into electricity c) (i) and (iii) only d) (iii) and (iv) only
b) converts electricity into light
39. The work-function of a metal is
c) stores light
d) stores electricity a) the minimum current required to take out elec-
tron from the metal surface
30. The maximum energy of electrons released in a pho- b) the maximum frequency required to take out
tocell is independent of electron from the metal surface
a) the frequency of the incident light c) the minimum amount of energy required to take
b) the intensity of the incident light out the electron from the metal surface
c) the nature of the cathode d) None of these
d) All of the above 40. The work function of a metal is independent of (i) na-
ture of the surface of the metal (ii) dimensions of the
31. Cathode ray consists of metal (iii) properties of the metal (iv) abundance of
a) photons b) electrons the metal
c) protons d) a-particles a) (i) only b) (i) and (iii)
c) (ii) and (iii) d) (ii) and (iv)
32. A discharge takes place between the two electrodes on
applying the electric field to the gas in the discharge 41. In photoelectric effect, electrons are ejected from met-
tube. The cause of this fluorescence was attributed to als,if the incident light has a certain minimum

96 - Nikhil Kamboj — 8193989452 #edugramddun


Dual Nature of Matter and Radiation

a) wavelength b) frequency 52. In a photon-particle collision, the quantity that does


c) amplitude d) angle of incidence not remain conserved is
a) total energy b) total momentum
42. Which of the following metals is not sensitive to visible
c) number of photons d) None of these
light?
a) Caesium b) Sodium 53. Of the following properties, the photon does not pos-
c) Rubidium d) Cadmium sess
43. A photosensitive substance emits when illuminated by a) rest mass b) momentum
light. c) energy d) frequency
a) only protons b) only neutrons
c) electrons and protons d) only electrons 54. It is essential to consider light as a stream of photons
to explain
44. The photoelectric current does not depend upon the a) diffraction of light b) refraction of light
(i) frequency of incident light (ii) work function of the c) photoelectric effect d) reflection of light
metal (iii) stopping potential (iv) intensity of incident
light 55. Photoelectric effect was discovered by
a) (i) and (iv) only b) (ii) and (iii) only a) Hertz b) Hallwachs c) Lenard d) Millikan
c) only (d) (ii) only d) (iii) only

45. The stopping potential is directly related to


56. The momentum of a photon of wavelength l is

m
a) the work function of the metal a) hl b) h/l c) l/h d) h/cl
b) intensity of incident radiation
c) the saturation current for the given frequency 57. The photoelectrons emitted from a metal surface are
such that their velocity
d) the kinetic energy gained by the photoelectrons

46. The wave theory of light does not explain


a) polarisation b) diffraction
ra a)
b)
c)
d)
is zero for all
is same for all
lies between zero and infinity
lies between zero and a finite maximum
c) photocurrent d) interference
G
47. Photoelectric effect can be explained by 58. Photoelectric effect shows
a) wave theory of light a) wave like behaviour of light
u
b) Bohr’s theory b) particle like behaviour of light
c) quantum theory of light c) both wavelike and particle like behaviour
d) corpuscular theory of light d) neither wave like nor particle like behaviour of
Ed

light
48. In Einstein’s picture of Photoelectric emission, the
photoelectric emission does not take place by
59. A photoelectric cell converts
a) continuous emission of energy from radiation
a) light energy into heat b) light energy to sound
b) continuous absorption of energy from radiation
energy energy
c) discrete absorption of energy from radiation
d) discrete emission of energy from radiation c) light energy into elec- d) electric energy into
tric energy light energy
49. The particle nature of light is not confirmed by
60. The work functions of Silver and Sodium are 4.6 and
a) photoelectric effect
2.3 eV, respectively. The ratio of the slope of the stop-
b) scattering of X-rays by electrons
ping potential versus frequency plot for Silver to that
c) diffraction of electrons
of Sodium is
d) compton effect
a) 1 b) 2 c) 4 d) zero
50. Photons are deflected by
a) electric field only b) magnetic field only 61. In a photoelectric experiment the stopping potential
c) electromagnetic field d) None of these for the incident light of wavelength 4000Å is 2 volt.
If the wavelength be changed to 3000 Å, the stopping
51. Electrically, photons are potential will be
a) positively charged b) negatively charged a) 2 V b) zero
c) neutral d) may be + or - c) less than 2 V d) more than 2 V

97 - Nikhil Kamboj — 8193989452 #edugramddun


Dual Nature of Matter and Radiation

62. A photocell is illuminated by a small bright source a) 100 V b) 150 V c) 250 V d) 103 V
placed 1 m away. When the same source of light is
placed 2 m away, the number of electrons emitted by 72. When light is directed at the metal surface, the emit-
photocathode are reduced by a factor of ted electrons:
a) are called photons
a) 1/8 b) 1/16 c) 1/2 d) 1/4
b) have energies that depend upon the intensity of
63. Light from a hydrogen discharge tube is incident on light
the cathode of a photoelectric cell, the work function c) have random energies
of the cathode surface is 4.2 eV. In order to reduce the d) have energies that depend upon the frequency of
photocurrent to zero the voltage of the anode relative light
to the cathode must be made
73. Which of the following is not the property of photons
a) – 4.2 V b) – 9.4 V
c) – 17.8 V d) + 9.4 V a) charge b) rest mass
c) energy d) momentum
64. In Davison-Germer experiment, an electron beam is
74. The frequency and the intensity of a beam of light
incident on a crystal. The reflected beam consists of
falling on the surface of photoelectric material are in-
a) b) protons c) x-rays d) electrons creased by a factor of two. This will:
photons
a) increase the maximum K.E. of photo-electron as
65. In the Davisson and Germer experiment, the veloc- well as photoelectric current by a factor of two
ity of electrons emitted from the electron gun can be b) increase maximum K.E. of photoelectrons and
increased by would increase the photo current by a factor of

m
a) increasing the potential difference between the two
anode and filament c) increase the maximum K.E. of photo electrons
b) increasing the filament current by a factor of two and will no affect photoelec-
tric current
c) decreasing the filament current
d) decreasing the potential difference between the
anode and filament
66. Radiations of two photon’s energy, twice and ten times
ra d) No effect on both maximum K.E. and photoelec-
tric current
75. Ultra-violet radiation of 6.2 eV falls on an aluminium
G
the work function of metal are incident on the metal surface having work-function 4.2 eV. The kinetic en-
surface successively. The ratio of maximum velocities ergy (in J) of the fastest electron emitted is nearly.
of photo electrons emitted in two cases is a) 3 × 10−19 b) 3 × 10−15
a) 1 : 2 b) 1 : 3 c) 1 : 4 d) 1 : 1 c) 3 × 10−17 d) 3 × 10−21
u

67. When the minimum wavelength of X-rays is 2Å then 76. The de-Broglie wavelength of an electron moving with
the applied potential difference between cathode and a speed of 6.6 × 1015 m/s is nearly equal to
Ed

anticathode will be a) 10−11 m b) 10−9 m


a) 6.2 kV b) 2.48 kV c) 24.8 kV d) 62 kV c) 10−7 m d) 10−5 m
68. When the energy of the incident radiation is increased 77. Photoelectrons are being obtained by irradiating zinc
by 20%, the kinetic energy of the photoelectrons emit- by a radiation of 3100 Å. In order to increase the ki-
ted from a metal surface increased from 0.5 eV to 0.8 netic energy of ejected photoelectrons.
eV. The work function of the metal is a) the intensity of radiation should be increased
a) 0.65 eV b) 1.0 eV c) 1.3 eV d) 1.5 eV b) the wave length of radiation should be increased
c) the wavelength of radiation should be decreased
69. With which of the following particles moving with d) both wavelength and intesity of radiation should
same velocity de-Broglie wave length will be maxi- be increased
mum?
a) ß-particle b) α particle 78. The energy E and momentum p of a photon is given
c) electron d) proton by E = hv and p = h/l. The velocity of photon will
be
70. The momentum of a photon is 10−27 kg m/s. Its en- a) E/P b) (E/P)2 c) E/P d) (EP)3
p
ergy will be:
a) 3 × 10−19 J b) 3 × 10−34 j 79. For light of wavelength 5000 Å, the photon energy is
nearly 2.5 eV. For X-rays of wavelength 1 Å, the pho-
c) 3 × 10−27 J d) none of these
ton energy will be close to:
71. The wavelength associated with n electron is 1Å. The a) 2.5 × 5000 eV b) 2.5 ÷ 5000 eV
potential difference required for accelerating it is c) 2.5 × (5000)2 eV d) 2.5 ÷ (5000)2 eV

98 - Nikhil Kamboj — 8193989452 #edugramddun


Dual Nature of Matter and Radiation

Practice Questions and Numerical

DUAL NATURE is 6 V. If the wavelength of the incident light is in-


creased to 600 nm, calculate the new stopping poten-
1. (a) Define the terms, (i) threshold frequency and (ii) tial. [Ans. 4.97 V]
stopping potential in photoelectric effect. (b) Plot 11. If light of wavelength 412.5 nm is incident on each of
a graph of photocurrent versus anode potential for the metals given below, which ones will show photo-
a radiation of frequency u and intensities I1 and I2 electric emission and why ? [Ans. Na and K]
(I1 < I2 ).
12. Ultra-violet light ofwave length 800 Åand 700 Åwhen
2. (a) State two important features of Einstein’s photo- allowed to fall on hydrogen atoms in their ground state
electric equation. (b) Radiation of frequency 1015 Hz is found to liberate electrons with K.E. = 1.8 eV and
is incident on two photosensitive surfaces P and Q. 4.0 eV respectively. Find the value of Planck’s con-
There is no photoemission from surface P. Photoemis- stant. [Ans. 6.67 ×10−34 ]
sion occurs from surface Q but photoelectrons have
zero kinetic energy. Explain these observations and 13. Light of wavelength 2000 Åfalls on an aluminium
find the value of work function for surface Q. surface. In aluminium 4.2 eV an required to re-
move an electron. What is the kinetic energy of
3. Show graphically, the variation of the de- Broglie wave-
(a) the fastest (b) the Slowest emitted photoelec-
length (l) with the potential (V) through which an
trons, (c) what is the Stopping potential? (d)
electron is accelerated from rest.
what is the cut off wavelength for aluminium ?
4. Write Einstein’s photoelectric equation. State clearly [Ans. (a) 2 eV (b) zero (c) 2V (d) 2946.4 Å]

m
the three salient features observed in photoelectric ef-
fect, which can be explained on the basis of the above 14. The work function for caesium is 1.8 eV. Light of 4500
equation. Åis incident on it. Calculate (i) the maximum kinetic
energy of the emitted photoelectron (ii) maximum ve-
5. A proton and a deuteron are accelerated through the
same accelerating potential. Which one of the two has
(a) greater value of de-Broglie wavelength associated
with it, and
(b) less momentum?
ra locity of the emitted photoelelectron (iii) if the in-
tensity of the incident light is doubled, then find the
maximum kinetic energy of the emitted photoelectron.
[Ans. (i)
1.52 × 10−19 J (ii) 5.78 × 105 m/s (iii) 1.52 × 10−191 ]
G
Give reasons to justify your answer.
15. Find the frequency of light which ejects electrons
6. A proton and an alpha particle are accelerated through
from a metal surface, fully stopped by a retard-
the same potential. Which one of the two has
ing potential of 3.3 V. If Photoelectric emission be-
(i) greater value of de-Broglie wavelength associated
u
gins in this metal at a frequency of 8 × 1014 Hz,
with it, and
calculate the work function (in eV) for this metal.
(ii) less kinetic energy.
[Ans. 15.96 × 1014 Hz, 3.315 eV]
Give reasons to justify your answer.
Ed

7. Define the terms (i) ‘cut-off voltage’ and (ii) ‘thresh- 16. The electric field associated with a light wave is given
old frequency’ in relation to the phenomenon of photo- by E = E0 sin [( 1.57 × 107 m−1 )(ct − x)]. Find the
electric effect. Using Einstein’s photoelectric equation stopping potential when this light is used in an exper-
show how the cut-off voltage and threshold frequency iment on photoelectric effect with the emitter having
for a given photosensitive material can be determined work function 2.1 eV. [Ans. 1.0 v]
with the help of a suitable plot/graph. 17. A photon of wavelength 3310 Åfalls on a photo
8. Draw a graph showing the variation of stopping poten- cathode and an electron of energy 3 × 10−19 J is
tial with frequency of incident radiation for two photo- ejected. If the wavelength of the incident photon is
sensitive materials having work functions W1 and W2 changed to 5000 Å, the energy of the ejected elec-
(W1 ¿ W2). Write two important conclusions that can tron is 9.72 × 10−20 J. Calculate the value of Planck’s
be drawn from the study of these plots. constant and threshold wavelength of the photon.
[Ans. 6.62 × 10−34 Js; 6620 Å]
PHOTOELECTRIC EFFECT
18. Radiation of wavelength 180 nm ejects photo electrons
9. Given in Fig., is the graph between frequency (ν) of from a plate whose work function is 2.0 eV. If a uniform
the incident light and maximum kinetic energy (Ek ) magnetic field of flux density 5.0 × 10−5 T is applied
of the emitted photoelectrons. Find the values of (i) to the plate, what should be the radius of the path
threshold frequency and (ii) work function from the followed by electrons ejected normally from the plate
graph. [Ans. (i) 1015 Hz (ii) 4 eV] with maximum energy? [Ans. 0.149 cm]
10. When light of wavelength 400 nm is incident on the PHOTONS
cathode of a photocell, the stopping potential recorded

99 - Nikhil Kamboj — 8193989452 #edugramddun


Dual Nature of Matter and Radiation

19. Monochromatic light of frequency 6 × 1014 Hz associated with it? To which part of the electromag-
is produced by a laser. The power emitted netic spectrum does this value of wavelength corre-
is 2.0 × 10−3 W. Calculate the (i) energy of a spond? [Ans. 0.1533nm; X ray]
photon in the light beam and (ii) number of 31. An α-particle and a proton are accelerated from rest by
photons emitted on an average by the source. the same potential. Find
[Ans. (i) 39.78 ×10−20 (ii) 5.03 ×1015 photons/s] √ the ratio of their de-Broglie
wavelengths. [Ans. 1:2 2]
20. The minimum light intensity that can be perceived
32. An electron and photon each have a wavelength 1.00
by the eye is about 10−10 Wm−2 , Find the num-
nm. Find (i) their momenta (ii) the energy of
ber of photons of wavelength 5.84 × 10−7 m that
the photon and (iii) the kinetic energy of electron.
must enter the pupil, of area 10−14 m2 s−1 for vision.
[Ans. (i) 6.6 × 10−25 kg m/s, 6.6 × 10−25 kg m/s (ii)
[Ans. 3 × 104 photons]
1.98 × 10−16 J (iii) 2.39 × 10−19 J]
21. Find the number of photons emitted per minute by
33. An electron is accelerated through a potential differ-
a 25 W source of monochromatic light of wavelength
ence of 64 volts. What is the de-Broglie wavelength as-
5000 Å. [Ans. 3.77 × 1021 ]
sociated with it? To which part of the electromagnetic
22. A parallel beam of light is incident normally on a spectrum does this value of wavelength correspond ?
plane surface absorbing 40% of the light and reflect- [Ans. 1.53 Å, x-rays]
ing the rest. If the incident beam carries 10 watt
34. From a cliff that is 10.0 m above a lake, a boy (mass 40
of power, find the force exerted by it on the surface.
kg) jumps from rest, straight down into the water. At
[Ans. 5.33 × 10−8 N]
the instant, he strikes the water, what is his de-Broglie
23. Find the number of photons emitted per second by a wavelength? [Ans. 1.17 × 10−36 ]

m
40 W source of monochromatic light of wave-length
6000 Å. What is the photoelectric current assuming 35. The de-Broglie wavelength of an electron moving with
5% efficiency of photoelectric effect ? [Ans. 0.97 A] a velocity 1.5 × 108 m/s is equal to that of a photon.
Calculate the ratio of the kinetic energy of the electron
24. An X-ray tube produces a continuous spectrum of ra-

is the maximum energy of a photon in the radiation


ra
diation with its short wavelength end at 0.42 Å. What

? What is the order of accelerating voltage (for elec-


trons) required in such a case ? [Ans. 29.6 keV, 30 kV]
to that of photon. [Ans. 1/4]
36. Calculate the energy of an electron having de-Broglie
wavelength 5500 Å. [Ans. 4.9 × 10−6 eV]
37. An electron and a photon each have a wavelength
G
DE-BROGLIE HYPOTHESIS 2 nm. Find (i) their momenta (ii) the energy of
a photon and (iii) the kinetic energy of electron.
25. The threshold wavelength for two photosensitive sur- [Ans. (i) 3.3 × 10−25 kg m/s (ii) 6.2 × 10−2 eV (iii)
faces A and B are λ1 and λ2 respectively. What is 0.377 eV]
u

the ratio of the work functions of the two surfaces? 38. The wavelength of a photon is 1.4 A. It collides
[Ans. λ2/λ1] with an electron. Its wavelength after collision
is 4 Å. Calculate the energy of scattered electron.
Ed

26. A proton and an electron have same kinetic energy.


[Ans. 9.23 × 10−16 J]
Which one has greater de-Broglie wavelength and
why? [Ans. electron] 39. Find the energy that should be added to an electron of
27. An electron and alpha particle have the same de- energy 2 eV to reduce its de-Broglie wavelength from
Broglie wavelength associated with them. How 1 nm to 0.5 nm. [Ans. 6 eV]
are their kinetic energies related to each other? 40. The de-Broglie wavelength associated with proton
[Ans. K.E.e > K.E.α ] changes by 0.25%. If its momentum is changed by
28. The stopping potential in an experiment on photo- 9 × 10−26 kg m/s, find the initial momentum of elec-
electric effect is 1.5 V. What is the maximum kinetic tron. [Ans. 3.6 × 10−23 ]
energy of the photoelectrons emitted? [Ans. 1.5 eV] 41. An electron is accelerated through a potential differ-
29. An electron is accelerated through a potential differ- ence of 200 volts. What is the de-Broglie wavelength
ence of 100 volts. What is the de-Broglie wavelength associated with it? To which part of the electmmag-
associated with it? To which part of the electromag- netic spectrum does the value of wavelength corre-
netic spectrum does this value of wavelength corre- spond? [Ans. 0.87 Å; X-rays]
spond? [Ans. 0.123 nm, X ray] 42. Find the de-Broglie wavelength (in A) asscociated with
30. An electron is accelerated through a potential differ- a proton moving with a velocity 0.5 c, where c = 3×108
ence of 64 volts. What is the de-Broglie wavelength m/s. [Ans. 2.21 × 10−15 m]

100 - Nikhil Kamboj — 8193989452 #edugramddun


Atom
Important Formulae and Notes

▶ Rutherford’s atom model (b) Orbital velocity of electron in nth orbit


(a) Distance of closest approach,
e2 Z c Z
v= . = (c= speed of light)
1 2Ze 2
2ϵ0 h n 137 n
r0 =
4πϵ0 12 mu2
(c) Orbital frequency of electron
where Z = atomic number
e = magnitude of electric charge v me4 Z 2
ϵ0 = permitivity of free space f= = 2 3 3
2πr 4ϵ0 n h
m = mass of α−particle
u = initial velocity of α− particle
(d) Time period of electron
(b) Impact parameter,

1 2Ze2 cot θ2 h3 (4P ϵ0 )2


b= T = × n3
4πϵ0 12 mu2 4π 2 mZ 2 e4

where θ = angle between the initial path and final n3


or (1.52 × 10−16 ) ×
path after scattering Z2

m
(e) Wave number
▶ Energy Quantisation
1 me4 1 1
 
E = nhν = 2 3 − 2
λ 8ϵ0 ch n21 n2
where n = 1, 2, 3, ...
▶ Centripetal force,
mv 2
r
=
1 e2
4πϵ0 r2
ra where n2 = outer orbit (electron jumps from this
orbit)
n1 = inner orbit (electron falls in this orbit)
G
(f) Kinetic energy of electron
where m = mass of an electron
v = velocity of an electron 1 mZ 2 e4
K= mv 2 = 2 2 2
r = radius of circular orbit of an electron 2 8ϵ0 n h
u
▶ Electron can revolve in circular orbit without radiating
energy only when, (g) potential energy of electron
Ed

nh mZ 2 e4
Iω = U =−
2π 4ϵ20 n2 h2
where n = principle quantum number
I = moment of inertia of an electron (h) Total energy
ω = angular velocity of an electron mZ 2 e4
v (A) E = K + U = −
▶ Angular velocity, ω = 8ϵ20 n2 h2
r  2
where r = radius of circular orbits 2
−Z Rhc Z
(B) En = = −13.6 eV
▶ When electron jumps from nth outer orbit to pth in- n2 n
ner orbit, then energy radiated in the form of photon me4
where R = 2 3 (Rydberg’s constant)
is given by 8ϵ0 ch
En − Ep = hν for hydrogen atom, Z = 1
Thus, if n = 1, E1 = −Rch joule or −13.6eV
▶ Various parameters according to Bohr’s model Rch
n = 2, E2 = − joule or −3.4eV
(a) Radius of nth orbit 2
Rch
n = 3, E3 = − joule or −1.5eV
4πϵ0 n2 h2 ϵ0 n2 h2 9
r= = n = ∞, E∞ = 0
4π mZe
2 2 πmZe2
Multiple Choice Questions
Atom

1. According to classical theory, the path of an electron 10. The Lyman transitions involve
in Rutherford atomic model is a) largest changes of energy
a) spiral b) circular b) smallest changes of energy
c) parabolic d) straight line c) largest changes of potential energy
d) smallest changes of potential energy
2. Rutherford’s α-particle experiment showed that the
atoms have 11. The first model of atom was proposed by
a) Proton b) Nucleus c) Neutron d) Electrons a) Hans Geiger b) Ernst Rutherford
c) J.J. Thomson d) N.H.D Bohr
3. Rutherford’s atomic model was unstable because
a) nuclei will break down 12. The empirical atom model was given by
b) electrons do not remain in orbit a) J. J. Thomson b) Rutherford
c) orbiting electrons radiate energy c) Niels Bohr d) Sommerfeld
d) electrons are repelled by the nucleus
13. Which one did Rutherford consider to be supported
4. According to Bohr’s model of hydrogen atom by the results of experiments in which a-particles were
a) the linear velocity of the electron is quantised scattered by gold foil?
b) the angular velocity of the electron is quantised a) The nucleus of an atom is held together by forces
c) the linear momentum of the electron is quantised which are much stronger than electrical or grav-
d) the angular momentum of the electron is itational forces

m
quantised b) The force of repulsion between an atomic nucleus
and an a-particle varies with distance according
5. As the quantum number increases, the difference of to inverse square law
energy between consecutive energy levels c) a-particles are nuclei of Helium atoms
a)
b)
c)
d)
remain the same
increases
decreases
sometimes increases and sometimes decreases
ra d) Atoms can exist with a series of discrete energy
levels

14. According to the Rutherford’s atomic model, the elec-


G
trons inside the atom are
6. When hydrogen atom is in its first excited level, it’s a) stationary b) not stationary
radius is c) centralized d) None of these
a) four times, it ground state radius
u

b) twice times, it ground state radius 15. According to classical theory, the circular path of an
c) same times, it ground state radius electron in Rutherford atom model is
Ed

d) half times, it ground state radius a) spiral b) circular


c) parabolic d) straight line
7. Which of the following series in the spectrum of hy-
drogen atom lies in the visible region of the electro- 16. Rutherford’s a-particle experiment showed that the
magnetic spectrum? atoms have
a) Paschen series b) Balmer series a) Proton b) Nucleus c) Neutron d) Electrons
c) Lyman series d) Brackett series
17. Electrons in the atom are held to the nucleus by
8. The Balmer series for the H-atom can be observed
a) coulomb’s force b) nuclear force
a) if we measure the frequencies of light emitted c) vander waal’s force d) gravitational force
when an excited atom falls to the ground state
b) if we measure the frequencies of light emitted 18. The Rutherford a-particle experiment shows that most
due to transitions between excited states and the of the a-particles pass through almost unscattered
first excited state while some are scattered through large angles. What
c) in any transition in a H-atom information does it give about the structure of the
d) None of these atom?
a) Atom is hollow
9. In a hydrogen atom, which of the following electronic b) The whole mass of the atom is concentrated in
transitions would involve the maximum energy change a small centre called nucleus
a) n = 2 to n = 1 b) n = 3 to n = 1 c) Nucleus is positively charged
c) n = 4 to n = 2 d) n = 3 to n = 2 d) All of the above

102 - Nikhil Kamboj — 8193989452 #edugramddun


Atom

19. In Rutherford’s a -particle scattering experiment, 26. The electrons of Rutherford’s model would be ex-
what will be correct angle for a scattering for an im- pected to lose energy because, they
pact parameter b = 0 ?
a) move randomly
a) 90º b) 270º c) 0º d) 180º b) jump on nucleus
c) radiate electromagnetic waves
20. In the ground state in ...A... electrons are in stable
d) escape from the atom
equilibrium while in ...B... electrons always experi-
ences a net force. Here, A and B refer to
a) Dalton’s theory, Rutherford model 27. As one considers orbits with higher values of n in a hy-
b) Rutherford’s model, Bohr’s model drogen atom, the electric potential energy of the atom
c) Thomson’s model, Rutherford’s model a) decreases b) increases
d) Rutherford’s model, Thomson’s model c) remains the same d) does not increase
21. The significant result deduced from the Rutherford’s
scattering experiment is that 28. Which of the following parameters is the same for all
a) whole of the positive charge is concentrated at hydrogen-like atoms and ions in their ground states?
the centre of atom a) Radius of the orbit
b) there are neutrons inside the nucleus b) Speed of the electron
c) a-particles are helium nuclei c) Energy of the atom
d) electrons are embedded in the atom d) Orbital angular momentum of the electron

m
22. Electrons in the atom are held to the nucleus by
29. The angular speed of the electron in the nth orbit of
a) coulomb’s force b) nuclear force
Bohr hydrogen atom is
c) vander waal’s force d) gravitational force
a) directly proportional to n
23. In a Rutherford scattering experiment when a projec-
tile of charge Z1 and mass M1approaches a target nu-
cleus of charge Z2 and mass M2, the distance of closest
approach is r. The energy of the projectile is
ra b)
c)
d)
inversely proportional to n
inversely proportional to n2
inversely proportional to n3
G
a) directly proportional to Z1 Z2
30. According to Bohr’s model of hydrogen atom
b) inversely proportional to Z1
c) directly proportional to mass M1 a) the linear velocity of the electron is quantised
d) directly proportional to M1 × M2 b) the angular velocity of the electron is quantised
u
c) the linear momentum of the electron is quantised
24. According to classical theory, Rutherford’s atomic d) the angular momentum of the electron is
model is quantised
Ed

a) stable b) unstable
c) meta stable d) both (a) and (b)
31. As the quantum number increases, the difference of
25. Rutherford’s atomic model was unstable because energy between consecutive energy levels
a) nuclei will break down a) remain the same
b) electrons do not remain in orbit b) increases
c) orbiting electrons radiate energy c) decreases
d) electrons are repelled by the nucleus d) sometimes increases and sometimes decreases

Practice Questions and Numerical

1. There are materials which absorb photons of shorter energy possessed by the electron?
wavelength and emit photons of longer wavelength. 4. Using Bohr’s postulates of the atomic model, derive
Can there be stable substances which absorb photons the expression for radius of nth electron orbit. Hence
of larger wavelength and emit light of shorter wave- obtain the expression for Bohr’s radius.
length.
5. An a-particle moving with initial kinetic energy K to-
2. Derive the expression for the de-Broglie wavelength of wards a nucleus of atomic number z approaches a dis-
an electron moving under a potntial difference V volt. tance ‘d’ at which it reverses its direction. Obtain the
3. Using Rutherford model of the atom, derive the ex- expression for the distance of closest approach ‘d’ in
pression for the total energy of the electron in hydro- terms of the kinetic energy of a-particle K.
gen atom. What is the significance of total negative 6. Define ionisation energy. What is its value for a hy-

103 - Nikhil Kamboj — 8193989452 #edugramddun


Atom

drogen atom? to which these transitions belong.


7. Using Rutherford model of the atom, derive the ex- (ii) Find out the ratio of the wavelengths
pression for the total energy of the electron in hydro- of the emitted radiations in the two cases.
gen atom. What is the significance of total negative [Ans. (i) Balmer, Layman; (ii) 27:5]
energy possessed by the electron? 19. (i) In hydrogen atom, an electron undergoes transition
8. State Bohr postulates. Using Bohr’s postulates of the from third excited state to the second excited state
atomic model, derive the expression for radius of nth and then to the first excited state. Identify the spec-
electron orbit. Hence obtain the expression for Bohr’s tral series to which these transitions belong.
radius. (ii) Find out the ratio of the wavelengths
of the emitted radiations in the two cases.
9. Define the distance of closest approach. An α-particle [Ans. (i) Paschen, Balmer; (ii) 20:5]
moving with initial kinetic energy K towards a nucleus
of atomic number Z approaches a distance ‘d′ at which 20. In hydrogen atom, an electron undergoes transition
it reverses its direction. Obtain the expression for the from 3rd excited state to the first excited state and
distance of closest approach ‘d′ in terms of the kinetic then to the ground state. Identify the spectral series
energy of a-particle K. What will be distance of clos- to which these transitions belong.
est approach for an α-particle of doublet the kinetic (ii) Find out the ratio of the wavelengths
energy. of the emitted radiations in the two cases.
[Ans. (i) Balmer and Lyman series (ii)
10. Explain observations and conclusions of Gold foil ex- λB : λL = 4 : 1]
periment. Write two important limitations of Ruther-
21. What is the ratio of radii of the orbits corresponding
ford nuclear model of the atom.

m
to first excited state and ground state in a hydrogen
DISTANCE OF CLOSEST APPROACH AND atom? [Ans. 4:1]
IMPACT PARAMETER
22. Find the ratio of energies of photons produced due to
11. A beam of α-particles of velocity 2.1 × 107 m/s is scat- transition of an electron of hydrogen atom from its

est approach of α− particle to the gold nucleus. For


α-particle, 2e/m = 4.8 × 107 C/kg. [Ans. 2.5 × 1014 ]
12. In a head on collision between an α particle and a
ra
tered by a gold foil (Z = 79) Find the distance of clos- (i) second permitted energy level to the first level, and
(ii) the highest permitted energy level to the first per-
mitted level. [Ans. 3:1]
23. Find out the wavelength of the electron orbiting in (i)
G
gold nucleus, the minimum distance of approach is ground state of hydrogen atom (ii) first excited state
4 × 10−14 m. Calculate the energy of alpha particle. of hydrogen atom. [Ans. (i) 3.32Å (ii) 6.69Å]
Take Z = 79 for gold. [Ans. 5.688 MeV] 24. Calculate the shortest wavelength in the Balmer series
of hydrogen atom. In which region (infra-red, visible,
u
BOHR’S ATOM MODEL
ultraviolet) of hydrogen spectrum does this wavelength
13. What is the ratio of radii of the orbits corresponding
lie? [Ans. 3646 Å”; UV region]
to first excited state and ground state in a hydrogen
Ed

atom? [Ans. 4:1] 25. The electron, in a hydrogen atom, is in its second
excited state. Calculate the wavelength of the lines
14. The radius of innermost electron orbit of a hydrogen
in the Lyman series, that can be emitted through
atom is 5.3 × 10−11 m. What is the radius of orbit in
the permissible transitions of this electron. (Given
the second excited state? [Ans. 4.77 X 10−10 m]
the value of Rydberg constant, R = 1.1 × 107 m−1 )
15. The ground state energy of hydrogen atom is -13.6 eV. [Ans. λ = 102.3nm and 121nm]
What are the kinetic and potential energies of electron 26. The ground state energy of hydrogen atom is -13.6
in this state? [Ans. Ke = 13.6 eV, U = – 27.2 eV ] eV . If an electron makes a transition from an en-
16. (i) In hydrogen atom, an electron undergoes transition ergy level -0.85 eV to -3.4 eV , calculate the wave-
from third excited state to the second excited state length of spectral line emitted. To which series of
and then to the first excited state. Identify the spec- hydrogen spectrum does this wavelength belong ?
tral series to which these transitions belong. [Ans. 4853 Å, Balmer series]
(ii) Find out the ratio of the wavelengths 27. The ground state energy of hydrogen atom is -13.6
of the emitted radiations in the two cases. eV . lf an electron makes a transition from an en-
[Ans. Paschen, Balmer; 20:7] ergy level -0.85 eV to -1.51 eV , calculate the wave-
17. The ground state energy of hydrogen atom is -13.6 eV. length of the spectral line emitted. To which se-
What are the kinetic and potential energies of electron ries of hydrogen atom does this wavelength belong?
in this state? [Ans. 13.6 ev and -27.2 ev ] [Ans. 18750 Å, Paschen Series]
18. (i) In hydrogen atom, an electron undergoes transition 28. The ground state energy of hydrogen atom is -13.6
from 2nd excited state to the first excited state and eV (i) What are the potential energy and K.E. of
then to the ground state. Identify the spectral series electron is 3rd excited state ?(ii) If the electron

104 - Nikhil Kamboj — 8193989452 #edugramddun


Atom

jumps to the ground state from the third excited copper. [Ans. 1.22 x 104 V]
state, calculate the frequency of photon emitted. 33. Calculate shortest wavelength of Balmer series.
[Ans. (i) -1.7 eV ; 0.85 eV ; (ii) 3 x 1015 Hz] [Ans. 3646.8 Å]
29. At what speed must an electron revolve around the 34. The wavelength of Hβ line of Balmer series is 4861
nucleus of hydrogen atom so that it may not be pulled Å. Calculate the wavelength of Hα line of the series.
into the nucleus by electrostatic attraction? Given, [Ans. 6562 Å]
mass of electron = 9.1 × 10−31 kg, radius of orbit =
0.5 × 10−10 m. [Ans. 2.25 × 106 m/s] 35. A 12.9 eV beam of electrons is used to bom-
bard gaseous hydrogen at room temperature. Upto
30. Calculate the frequency of revolution of electron which energy level would the hydrogen be excited
in the second Bohr’s orbit of radius 2.12 Å. ? Calculate the wavelength of first member of
[Ans. 8.23 × 1014 Hz] Paschen series and first member of Balmer series.
31. Determine the radius of the first orbit of hy- [Ans. n = 4 ; 18750 Å; 6547 Å]
drogen atom. What would be the veloc- 36. Fig. shows energy level diagram of hydrogen atom.
ity and frequency of electron in this orbit ? Find out the transition which results in the emission of
[Ans. 0.53 Å; 2.19 × 106 m/s; 6.6 × 1015 Hz] a photon of wavelength 496 nm. Which transition cor-
32. The wavelength of Kα line for copper is 1.36 Å. Cal- responds to emission of radiation of maximum wave-
culate the ionisation potential of a K shell electron in length ? Justify your answer. [Ans. E4 —¿E2 : 4-¿3]

m
ra
u G
Ed

105 - Nikhil Kamboj — 8193989452 #edugramddun


Nuclie
Important Formulae and Notes

▶ Composition of nucleus
(a) Number of neutrons in the nucleus = A − Z Number of atoms present at given instant t is N =
where A = mass number N0 e−λt
Z = atomic number 1
(b) Mass of the nucleus = ZMp + (A − Z)Mn where ▶ Average life of radioactive element, Ta =
λ
Z = atomic number
(A − Z) = number of neutrons ▶ Einstein’s mass-energy equivalance
Mp = mass of proton
Mn = mass of the neutron E = mc2
▶ Mass defect, ∆m = [Zmp + (A − Z)Mn ] − MN
where MN = mass of nucleus
▶ Binding energy, BE = ∆m × 931.5(inM eV ) ▶ Relation between rest mass and mass of a particle in
BE motion is given by
▶ Binding energy per nucleon =
A
∆m m0
▶ Packing fraction = m= q
A 1− v2

m
c2
0.693
▶ Half-life period of radioactive element, T =
λ
where λ = decay constant where m0 = rest mass of particle
▶ Law of radioactive disintegration m = mass in motion
dN
dt
= −λN
ra v = velocity of a particle
c = velocity of light

Multiple Choice Questions


G
1. What will be the ratio of the radii of two nuclei of ∆m × c
c) ∆m × c2 A d)
mass numbers A = 1 and A = 2? A
 1/3  2
A1 A1 4. State the relation between the mean life (τ ) of a ra-
a) b)
u
A2 A2 dioactive element and its decay constant λ.
 1/3  
A2 A1 a) = 1/λ
c) d) τ
Ed

A1 A2 b) τ =λ
c) τ = 2/λ
2. Two nuclei have mass numbers in the ratio 1: 2. What
is the ratio of their nuclear densities? d) τ = 1/λ2

a) 1:2 b) 1:4 c) 1:1 d) 1:8 5. Four nuclei of an element undergo fusion to form a
heavier nucleus, with the release of energy. Which of
3. A nucleus of mass number A has a mass defect ∆m.
the two — the parent or the daughter nucleus – would
Give the formula, for the binding energy per nucleon
have higher binding energy per nucleon?
of this nucleus.
∆m × c2 ∆m × A a) Parent nucleus b) Daughter nucleus
a) b) c) Both same d) Data insufficient
A c2
Practice Questions and Numerical

1. A nucleus undergoes b – decay. How does 4. Can it be concluded from beta decay that electrons
(i) the mass number, exist inside the nucleus?
(ii) atomic number change? 5. Why is the ionizing power of a – parties greater than
that of g-rays?
2. Define the activity of a given radioactive substance.
Write its SI unit. [Ans. becquerel] 6. An electron and alpha particle have the same de-
Broglie wavelength associated with them. How
3. Why is it found experimentally difficult to detect neu- are their kinetic energies related to each other?
trinos in nuclear b-decay? [Ans. Ke > Kα ]
Nuclie

7. State three characteristic properties of nuclear force. 10−31 kg and mass of proton = 1.673 × 10−27 kg.
8. Two nuclei have mass numbers in the ratio 1: 2. What [Ans. 0.511 MeV, 941.1 MeV]
is the ratio of their nuclear densities? [Ans. 1:1] 19. Assuming that protons and neutrons have equal
9. Two nuclei have mass numbers in the ratio 1: 8. What masses; calculate how many times nuclear matter is
is the ratio of their nuclear radii? [Ans. R1:R2=1:2] denser than Water. [Ans. 2.307 × 1014 ]

10. Two nuclei have mass numbers in the ratio 20. What is the nuclear radius of F e125 ,if that of Al27 is
8:125. What is the ratio of their nuclear radii? 3.6 fermi. [Ans. 6 fermi]
[Ans. R1:R2=2:5] 21. The nuclear radius of 8 O16 is 3 × 10−15 m. What is
11. (a) The mass of a nucleus in its ground state is always its nuclear mass density? [Ans. 2.359 × 101 7 kg/m3 ]
less than the total mass of its constituents – neutrons MASS DEFECT AND NUCLEAR BINDING
and protons. Explain. ENERGY
(b) Plot a graph showing the variation of potential
energy of a pair of nucleons as a function of their sep- 22. The binding energy of 10 N e20 is 160.6 M eV . Find its
aration. Write two important conclusions which you atomic mass. Take mass of proton = 1.007325 u and
can draw regarding the nature of nuclear forces. mass of neutron = 1.003655 u. [Ans. 19.9924 u]
12. A heavy nucleus X of mass number 240 and binding 23. The binding energies of deuteron (1 H 2 ) and alpha
energy per nucleon 7.6 MeV is split into two fragments particle (2 He4 ) are 1.25 and 7.2 M eV /nucleon re-
Y and Z of mass numbers 110 and 130. The binding spectively. Which nucleus is more stable ? Cal-
energy of nucleons in Y and Z is 8.5 MeV per nucleon. culate binding energy per nucleon of 26 F e56 . Give;
mass of 26 F e56 = 55.934939 amu, mass of proton=

m
Calculate the energy Q released per fission in MeV.
[Ans. 216 MeV] 1.007825 amu, mass of neutron = 1.008665 amu.
[Ans. Helium; 8.79 MeV/nucleon]
13. Draw a plot of the binding energy per nucleon as a
function of mass number for a large number of nuclei, 24. What energy is needed to split an alpha particle to
2 ≤ A ≤ 240. How do you explain the constancy of ra pieces ? Given mass of α particle is 4.0029 amu.
binding energy per nucleon in the range 30 ¡ A ¡ 170 [Ans. 27.09 MeV]
using the property that nuclear force is short-ranged? 25. The binding energy per nucleon for 6 C 12 is 7.68
14. Using the curve for the binding energy per nucleon as MeV/N and that for 6 C 13 is 7.47 MeV/N. Calculate
the energy required to remove a neutron from 6 C 13 .
G
a function of mass number A, state clearly how the re-
lease of energy in the processes of nuclear fission and [Ans. 4.95 MeV]
nuclear fusion can be explained. 26. The mass of nucleus of 17 Cl35 is 34.9300 a.m.u. Calcu-
15. State the reason, why heavy water is generally used as late the average energy required to extract a nucleon
u
a moderator in a nuclear reactor. from this nucleus. Use 1 a.m.u. = 931.5 MeV. Given
mass of proton = 1.007825 u, and mass of neutron =
ATOMIC NUCLEUS, ITS SIZE AND DENSITY l.008665 u. [Ans. 8.22 MeV]
Ed

27. Calculate the binding energy per nucleon in the nuclei


16. How many electrons, protons and neu- of 15 P 31 . Given, m(15 P 31 ) = 30.97376 u;, m(0 n1 ) =
trons are there in 14 grams of 6C ?
14
1.00865 u; m(1 H 1 ) = 1.00782 u. [Ans. 8.47 MeV/N]
[Ans. 36 × 1023 , 36 × 1023 , 48 × 1023 ]
28. A nucleus of mass number 240 and having
17. Calculate the density of hydrogen nucleus in SI units, B.E./nuclear 7.6 MeV splits into two fragments Y,
given R0 = 1.2 fermi, mass of proton = 1.007825 Z of mass numbers 110 and 130 respectively. If the
a.m.u. [Ans. 2.295 × 1017 kg/m3 ] BE/nucleon of Y, Z is equal to 8.5 MeV each, cal-
18. Calculate the equivalent energy of electron and pro- culate the energy released in the nuclear reaction.
ton at rest. Given that mass of electron = 9.1 × [Ans. 216 MeV]

107 - Nikhil Kamboj — 8193989452 #edugramddun


Semiconductor
Important Formulae and Notes

▶ AC resistance (rd ) of the junction diode is defined as ▶ Transistors


the ratio of small change in voltage (∆V to the small (a) IE = IB + IV
change in current (∆I produced i.e., where IE = emitter current
∆V IB = base current
rd = IC = collector current
∆I
(b) In CB configuration the ratio of the collector cur-
▶ Half-wave rectifier rent to the emitter current is called current gain
Vm IC
(a) maximum current, Im = (α), α =
ri + RL IE
where Vm = maximum voltage (c) In CE configuraion the ratio of the colelctor cur-
ri = internal resistance of the diode IC
rent to the base curent is called beta (β) =
RL = load resistance IB
Im (d) AC voltage gain (A)
(b) Average current, IDC =
π
where Im = maximum current ∆Eout ∆Iout × RL RL
Av = = = β2
Im ∆Vin ∆Iin × Ri Ri
(c) Rms current, Irms = √
2

m
(d) AC power input, PAC = (Irms )2 × (r1 × RL ) (e) AC power gain
(e) DC power output, PDC = (IDC )2 × RL ∆Pout ∆Iout
2
× RL RL
where IDC = average current = = = β2
∆Pin ∆Iin
2 ×R
i Ri
(f) Efficiency of rectification, η =
where PDC = DC Power output
PAC = AC power iput
▶ Full wave rectifier
Pdc
PAC
ra ▶ Relation between α and β

β=
α
1−α
G
(a) Efficiency of full wave rectifier, ▶ Logic Gates
DC power output A, B are inputs and Y is output
η=
OR gate
u
AC power Input (a)
(b) AND gate
2Im
(b) Average current, IDC = (c) NOT gate
Ed

π
(c) Rms current, AC power input, DC power output (d) NOR gate
== similar to Half-wave rectifier. (e) NAND gate

Multiple Choice Questions

1. In a semiconductor, the number of valence electrons a) Only one direction b) Both directions
is:
a) 0 b) 1 c) 2 d) 3 5. In a semiconductor, the energy gap between the va-
lence band and the conduction band is also known as:
2. In an n-type semiconductor, the majority charge car-
riers are: a) Forbidden zone b) Energy gap
c) Bandwidth d) Fermi level
a) Electrons b) Holes
c) Protons d) Neutrons
6. When a p-n junction diode is forward-biased, the flow
3. The process of adding impurities to a semiconductor of current is primarily due to the movement of:
to increase its electrical conductivity is called:
a) Electrons b) Holes
a) Inversion b) Amplification c) Ions d) Neutrons
c) Doping d) Polarization

4. A p-n junction in a semiconductor device allows the 7. The process of recombination in a semiconductor oc-
flow of current in: curs when:
Semiconductor

a) Electrons move from the conduction band to the 16. The primary mechanism responsible for electron flow
valence band in a conductor is:
b) Electrons move from the valence band to the con- a) Electron movement within the crystal lattice
duction band b) Electron tunneling
c) Electrons move from one end of the crystal to c) Electron acceleration
the other d) Electron diffusion
d) Electrons remain stationary
17. The temperature coefficient of resistance for a typical
8. A semiconductor with a completely filled valence band semiconductor is:
and an empty conduction band is classified as a: a) Positive b) Negative
a) Metal b) Insulator c) None
c) Superconductor d) Conductor
18. What is the primary charge carrier in an n-type semi-
conductor?
9. Which of the following materials is commonly used in
the fabrication of light-emitting diodes (LEDs)? a) Electrons b) Holes
c) Ions d) Protons
a) Silicon (Si)
b) Germanium (Ge) 19. In an n-type semiconductor, which type of impurity
c) Gallium arsenide (GaAs) atoms are typically added to increase electron concen-
d) Indium antimonide (InSb) tration?
a) Boron b) Phosphorus
c) Aluminumd) Indium

m
10. The number of valence electrons in silicon (Si) is:
a) 1 b) 2 c) 3 d) 4 20. The phenomenon of ”avalanche breakdown” is most
closely associated with which semiconductor device?
11. A semiconductor diode allows current to flow in which a) Zener diode
direction when it is reverse-biased?
a)
b)
From the p-side to the n-side
From the n-side to the p-side
ra b)
c)
d)
Light-emitting diode (LED)
Bipolar junction transistor (BJT)
Schottky diode
c) In both directions
G
21. In a forward-biased p-n junction diode, which region
d) It does not allow any current to flow becomes narrower?
a) Depletion region b) Valence band
12. The term ”hole” in semiconductor physics refers to:
c) Conduction band d) N-type region
u
a) A physical gap in the crystal lattice
b) An electron with positive charge 22. The ”Fermi level” in a semiconductor represents the:
c) A vacancy in the valence band a) Highest energy level in the valence band
Ed

d) A particle with no charge b) Lowest energy level in the conduction band


c) Energy level at which electrons have a 50% prob-
13. In a p-n junction diode, the depletion region: ability of being occupied
a) Contains excess electrons d) Energy level of the most mobile charge carriers
b) Is an area of high electron concentration
c) Has a net positive charge 23. Which semiconductor device can be used as a voltage
d) Contains immobile ions regulator to maintain a constant output voltage?
a) Light-dependent resistor (LDR)
14. The term ”doping” in semiconductor physics refers to: b) Field-effect transistor (FET)
c) Varistor
a) The addition of impurities to change the electri-
d) Zener diode
cal properties
b) The process of heating the semiconductor 24. A material with a high electrical conductivity and low
material resistance is classified as a:
c) The removal of excess electrons a) Insulator b) Conductor
d) The creation of an insulating layer c) Semiconductor d) Superconductor

15. When a semiconductor is heated, its electrical conduc- 25. Which of the following materials is used as a semicon-
tivity: ductor in the fabrication of solar cells?
a) Increases b) Decreases a) Copper b) Silver
c) Remains unchanged d) Becomes unpredictable c) Silicon d) Aluminum

109 - Nikhil Kamboj — 8193989452 #edugramddun


Semiconductor

26. Which semiconductor property is responsible for the d) The voltage at which the diode oscillates
phenomenon of rectification in a diode?
a) Negative tempera- b) Positive temperature 29. A silicon (Si) crystal doped with boron atoms becomes
ture coefficient coefficient a:
c) Asymmetry of charge d) Superconductivity a) p-type semiconductor
carriers b) n-type semiconductor
c) Superconductor
27. Which type of semiconductor device emits light when d) Insulator
current flows through it?
a) Zener diode 30. The electrical conductivity of a semiconductor is pri-
b) Bipolar junction transistor (BJT) marily determined by the concentration of:
c) Light-emitting diode (LED) a) Electrons b) Holes
d) Schottky diode c) Ions d) Protons

28. The ”knee voltage” in a diode refers to: 31. In a semiconductor, when the temperature increases,
a) The voltage at which the diode turns on its electrical conductivity generally:
b) The voltage at which the diode turns off a) Increases b) Decreases
c) The breakdown voltage of the diode c) Remains unchanged d) Becomes unpredictable

Practice Questions and Numerical

m
DESCRIPTIVE QUES n-side is positive and p-side is negative, it is said to
1. What happens to the width of depletion layer of a p- be reverse biased. An ideal diode is one whose resis-
n junction when it is (i) forward biased, (ii) reverse tance in forward biasing is zero and the resistance is
biased?
2. What is the difference between intrinsic and extrinsic
semiconductors?
3. What is the difference between an n-type and a p-type
ra infinite in reverse biasing. When the diode is forward
biased, it is found that beyond forward voltage called
knee voltage, the conductivity is very high. When the
biasing voltage is more than the knee voltage the po-
tential barrier is overcome and the current increases
G
semiconductor? rapidly with increase in forward voltage. When the
4. What is Zener Diode? Explain its working. diode is reverse biased, the reverse bias voltage pro-
duces a very small current about a few microamperes
5. How is forward biasing different from reverse biasing
which almost remains constant with bias. This small
u
in a pn junction diode?
current is reverse saturation current.
6. Explain how a depletion region is formed in a junction
diode.
Ed

(i) In the given figure, a diode D is connected to an


7. The current in the forward bias is known to be more external resistance R = 100 ohms and an emf of 3.5
( mA) than the current in the reverse bias ( µA). What V. If the barrier potential developed across the diode
is the reason, then, to operate the photodiode in re- is 0.5 V, the current in the circuit will be:
verse bias?
8. Explain, with the help of a circuit diagram, the work-
ing of a p-n junction diode as a half-wave rectifier.
9. Name the important processes that occurs during the
formation of a p-n junction.Explain briefly, with the
help of a suitable diagram, how a p-n junction is
formed. Define the term ‘barrier potential’?
10. CASE STUDY : Read the following paragraph
and answer the questions that follow.
A semiconductor diode is basically a pn junction with
metallic contacts provided at the ends for the applica-
tion of an external voltage. It is a two terminal device.
When an external voltage is applied across a semicon- (a) 40 mA (b) 20 mA (c) 35 mA (d) 30 mA
ductor diode such that p-side is connected to the posi-
tive terminal of the battery and n-side to the negative (ii) In which of the following figures, the pn diode is
terminal, it is said to be forward biased. When an reverse biased?
external voltage is applied across the diode such that

110 - Nikhil Kamboj — 8193989452 #edugramddun


Semiconductor

11. A semiconductor has equal electron and hole concen-


tration of 6 × 108 /m3 . On doping with certain impu-
rity, electron conventration increases to 9 × 101 2/m3 .
(i) Identify the new semiconductor obtained after
doping. (ii) Calculate the new hole concentration.
(iii) How does the energy gap vary with doping?
[Ans. (i) n-type; (ii) 4 × 104 /m3 (iii) Energy gap
decreases with doping]
12. In a pure semiconductor, the number of conduction
electrons is 6 × 1019 per cubic metre. Find the total
number of current carriers (electrons and holes) in a
same semiconductor of size 1 cm X 1 cm X 1 mm.
(a) (b) (c) (d) [Ans. 12 × 1012 ]
(iii) Based on the V-I characteristics of the diode, we 13. The electrical conductivity of a semiconductor in-
can classify diode as creases when electromagnetic radiation of wavelength
(a) bilateral device (b) ohmic device (c) non-ohmic shorter than 2480 nm in incident on it. Find the band
device (d) passive element gap of the semiconductor. [Ans. 0.5 eV]
(iv) Two identical PN junctions can be connected in 14. A doped semiconductor has impurity levles 20 meV
series by three different methods as shown in the fig- below the conduction band. Is the material n-type
ure. If the potential difference in the junctions is the or p-type? What is the wavelength of light so that
same, then the correct connections will be

m
the electron of impurity level is just able to jump into
conduction band? [Ans. n-type, 6.22 × 10−5 m]
15. Determine the number density of donor atoms which
have to be added to an intrinsic germanium semi-
ra conductor to produce an n-type semiconductor to
conductivity 5 Ω−1 cm−1 , given that the mobility of
electron in n-type germanium is 2900 cm2 V −1 s−1 .
Neglect the contribution of holes to conductivity.
[Ans. 8.012 x 1015 cm−3 ]
G
16. On doping germanium with donor atoms of density
1017 cm−3 , find its conductivity if mobility of elec-
(a) in the circuits (1) and (2) trons is 3800 cm2 /V − s and intrinsic carrier concen-
(b) in the circuits (2) and (3) (c) in the circuits (1)
u
tration is 2.5 × 1013 cm−3 . Also find the ratio of con-
and (3) ductivity of doped germanium and pure germanium.
(d) only in the circuit (1) [Ans. 60.8 S/cm; 4 × 103 : 1]
Ed

(v) The V-I characteristic of a diode is shown in the 17. The energy of at photon of sodium light wavelength
figure. The ratio of the resistance of the diode at I = 5890 Åequals the energy gap of a semiconducting ma-
15 mA to the resistance at V = -10 V is terial. Find (a) the minimum energy E required to
create a hole-electron combination. (b) the value of
E/kT at a temperature of 27°C, where k = 8.62×10−5
eV/K. [Ans. 2.1 eV, 81.2]
18. Calculate the number of states per cubic metre of
sodium in 3s band. The density of sodium is
1013 kg/m3 . How many of them are empty ?
[Ans. 5.3 × 1028 ; 2.65 × 1028 ]
p − n JUNCTIONS

19. The V − I characteristics of a silicon diode is given


in Fig.. Calculate the diode resistance in (a) for-
ward bias at V = + 2 V and V = + 1 V and
(b) reverse bias V = - 1 V and V = - 2 V.
[Ans. (a) 20 Ω, 40 Ω (b) 8 × 106 Ω, 8 × 106 Ω]
(a) 100 (b) 106 (c) 10 (d) 10−6
20. A p − n junction is fabricated from a semiconductor
INTRINSIC AND EXTRINSIC
with band gap of 3.0 eV. Can it detect a wave-length
SEMICONDUCTOR
of (i) 600 nm (ii) 400 nm ? [Ans. (i) no (ii) Yes]

111 - Nikhil Kamboj — 8193989452 #edugramddun


Semiconductor

21. A potential barrier of 0.7 V exists across a p − n junc- [Ans. (a) 0.50 A (b) 0.33 A]
tion (a) If the depletion layer is 7.0 x 10−7 m thick,
24. A silicon p−n junction diode whose knee voltage is 0-7
what is the intensity of the electric filed in this region
V is connected to a battery with supply voltage 3 V
? (b) If an electron is approaching the p − n junction
and 100 mA in forward biasing. If a resistor R is used
from the n-side with a speed 6 × 105 m/s, determine
in series of the circuit, then a current of 20 mA passes
the speed with which it enters the p-side of p − n junc-
through the diode. Find the wattage of the resistor
tion. [Ans. (a) 1 × 106 V/m (b) 3.4 × 105 m/s]
and of the diode. [Ans. 4.6 × 10−2 W, 1.4 × 10−2 W]
22. A sinusoidal voltage of rms value 200 volt is connected
to a diode and a resistor R in the circuit as shown in 25. The applied input a.c. to a half wave rectifier is
Fig., so that half wave rectification occurs. If the diode 120 watt. The d.c. output is 50 watt. What
is ideal, what is the rms voltage across R ? If resistor R is the rectification efficiency and power efficiency?
is replaced by capacitor of capacitance√C, find the [Ans. 41.67% ; 83-3̇3%]
√ po-
tential difference across C. [Ans. 100 2 V; 200 2 V] 26. Find the average value of d.c. voltage that can be ob-
23. Find the current through the resistance R in tained from the half wave rectifier of Fig., assuming
Fig.. If (a) R =10 Ω and (b) R = 20 Ω. that the diode is ideal one. [Ans. 9.0 V]

m
ra
u G
Ed

112 - Nikhil Kamboj — 8193989452 #edugramddun

You might also like